| Additional Questions for each Class with Solution | ||||||
|---|---|---|---|---|---|---|
| 6th | 7th | 8th | 9th | 10th | 11th | 12th |
| Content On This Page | ||
|---|---|---|
| Objective Type Questions | Short Answer Type Questions | Long Answer Type Questions |
Chapter 2 Relations and Functions (Additional Questions)
Welcome to this essential supplementary practice resource dedicated to deepening your understanding of Relations and Functions, a cornerstone chapter in your Class 11 mathematics curriculum. This topic serves as a crucial bridge, connecting the abstract language of Set Theory to the dynamic world of Calculus and advanced mathematical analysis. While the main chapter meticulously laid out the foundational definitions and concepts – starting from the Cartesian product of sets, defining relations as subsets, and culminating in the specific type of relation known as a function – this collection of additional questions is designed to propel you towards a more robust and nuanced mastery. Prepare to engage with more challenging problems that test not just your knowledge but your analytical skills in identifying function properties, determining domains and ranges, and manipulating functions algebraically.
Recall that we began by defining the Cartesian Product $A \times B$ as the set of all possible ordered pairs $(a, b)$ where $a \in A$ and $b \in B$. A relation $R$ from set $A$ to set $B$ was then defined as any subset of $A \times B$. We learned to identify the Domain (set of all first elements of ordered pairs in $R$), Range (set of all second elements), and Codomain (the entire set $B$). The concept of a Function $f: A \to B$ emerged as a special type of relation where every element in the domain $A$ is associated with exactly one element in the codomain $B$. This uniqueness condition is paramount. This supplementary section provides rigorous practice in identifying whether given rules, sets of ordered pairs, or graphs truly represent functions, perhaps including more complex piecewise-defined functions.
A significant portion of the chapter, and indeed this practice section, focuses on various types of real-valued functions (where the codomain is a subset of the real numbers $\mathbb{R}$) and understanding their behavior. You were introduced to fundamental functions like the Identity function ($f(x)=x$), Constant function ($f(x)=k$), Polynomial functions, Rational functions ($f(x)=\frac{p(x)}{q(x)}$, where $p(x), q(x)$ are polynomials and $q(x) \neq 0$), the Modulus function ($f(x)=|x|$), the Signum function, and the Greatest Integer Function ($f(x)=[x]$). For each, understanding their characteristic graphs, determining their Domain (the set of all possible input values $x$) and Range (the set of all possible output values $f(x)$) is critical. This practice section heavily emphasizes finding the domain and range of more complicated functions, particularly those involving square roots (requiring the expression under the root to be non-negative, $\text{radicand} \ge 0$) or rational expressions (requiring the denominator to be non-zero).
Furthermore, you explored the algebra of real functions, learning how to combine functions through addition ($(f+g)(x) = f(x)+g(x)$), subtraction ($(f-g)(x) = f(x)-g(x)$), multiplication ($(fg)(x) = f(x)g(x)$), and division ($(\frac{f}{g})(x) = \frac{f(x)}{g(x)}$, provided $g(x) \neq 0$). A crucial aspect, reinforced heavily here, is determining the domain of these combined functions. Generally, the domain is the intersection of the domains of the original functions ($ \text{dom}(f) \cap \text{dom}(g) $), with the additional constraint for division that $g(x)$ cannot be zero. Problems here might involve finding the algebraic form and the precise domain of combined functions. While function composition is formally detailed in Class 12, understanding these basics prepares you for it. Engaging thoroughly with this practice is vital for building a strong conceptual foundation, mastering domain and range calculations, and preparing for the deeper analysis of function properties, inverses, and calculus in Class 12.
Objective Type Questions
Question 1. If $(2a+1, 3b-2) = (5, 4)$, find the values of $a$ and $b$.
(A) $a=2, b=2$
(B) $a=3, b=2$
(C) $a=2, b=3$
(D) $a=3, b=3$
Answer:
Solution:
Given the equality of two ordered pairs:
$(2a+1, 3b-2) = (5, 4)$
For two ordered pairs to be equal, their corresponding components must be equal.
Equating the first components:
$2a+1 = 5$
Solving for $a$:
$2a = 5 - 1$
$2a = 4$
$a = \frac{4}{2}$
$a = 2$
Equating the second components:
$3b-2 = 4$
Solving for $b$:
$3b = 4 + 2$
$3b = 6$
$b = \frac{6}{3}$
$b = 2$
Thus, the values of $a$ and $b$ are $a=2$ and $b=2$.
This matches option (A).
The final answer is $\boxed{a=2, b=2}$.
Therefore, the correct option is (A) $a=2, b=2$.
Question 2. Let $A = \{1, 2\}$ and $B = \{3, 4\}$. Find $A \times B$.
(A) $\{(1,3), (2,4)\}$
(B) $\{(1,3), (1,4), (2,3), (2,4)\}$
(C) $\{(3,1), (4,1), (3,2), (4,2)\}$
(D) $\{1, 2, 3, 4\}$
Answer:
Solution:
Given:
Set $A = \{1, 2\}$
Set $B = \{3, 4\}$
To Find:
The Cartesian product $A \times B$.
The Cartesian product $A \times B$ is the set of all ordered pairs $(a, b)$ where $a \in A$ and $b \in B$.
To find $A \times B$, we take each element from set $A$ and pair it with each element from set $B$ to form an ordered pair.
For the element $1 \in A$, the pairs are $(1, 3)$ and $(1, 4)$.
For the element $2 \in A$, the pairs are $(2, 3)$ and $(2, 4)$.
Combining all these pairs, we get the set $A \times B$:
$A \times B = \{(1, 3), (1, 4), (2, 3), (2, 4)\}$
Comparing this result with the given options, we find that it matches option (B).
The final answer is $\boxed{\{(1,3), (1,4), (2,3), (2,4)\}}$.
Therefore, the correct option is (B) $\{(1,3), (1,4), (2,3), (2,4)\}$.
Question 3. If $n(A) = 3$ and $n(B) = 2$, then the number of elements in $A \times A \times B$ is:
(A) 18
(B) 12
(C) 8
(D) 27
Answer:
Solution:
Given:
The number of elements in set A, $n(A) = 3$.
The number of elements in set B, $n(B) = 2$.
To Find:
The number of elements in the Cartesian product $A \times A \times B$, denoted as $n(A \times A \times B)$.
The number of elements in the Cartesian product of multiple finite sets is the product of the number of elements in each set.
For sets $S_1, S_2, ..., S_k$, the number of elements in their Cartesian product is given by:
$n(S_1 \times S_2 \times ... \times S_k) = n(S_1) \times n(S_2) \times ... \times n(S_k)$.
In this case, we need to find $n(A \times A \times B)$. Using the formula:
$n(A \times A \times B) = n(A) \times n(A) \times n(B)$
Substitute the given values of $n(A)$ and $n(B)$ into the formula:
$n(A \times A \times B) = 3 \times 3 \times 2$
$n(A \times A \times B) = 9 \times 2$
$n(A \times A \times B) = 18$
Thus, the number of elements in $A \times A \times B$ is 18.
The final answer is $\boxed{18}$.
Comparing this result with the given options, we find that it matches option (A).
Therefore, the correct option is (A) 18.
Question 4. Let $A = \{1, 2, 3, 4, 5\}$. A relation $R$ is defined on $A$ as $R = \{(x, y) : x, y \in A, y = x+1\}$. The range of $R$ is:
(A) $\{1, 2, 3, 4\}$
(B) $\{2, 3, 4, 5\}$
(C) $\{1, 2, 3, 4, 5\}$
(D) $\{1, 2, 3, 4, 5, 6\}$
Answer:
Solution:
Given:
Set $A = \{1, 2, 3, 4, 5\}$.
Relation $R$ on $A$ is defined as $R = \{(x, y) : x, y \in A, y = x+1\}$.
To Find:
The range of the relation $R$.
The relation $R$ consists of ordered pairs $(x, y)$ such that both $x$ and $y$ are elements of set $A$, and $y$ is one more than $x$. We need to find the elements of $R$ by checking which pairs $(x, y)$ satisfy these conditions.
Let's list the possible ordered pairs $(x, y)$ where $x \in A$, $y = x+1$, and we also require $y \in A$.
If $x = 1$, then $y = 1+1 = 2$. Since $1 \in A$ and $2 \in A$, the pair $(1, 2)$ is in $R$.
If $x = 2$, then $y = 2+1 = 3$. Since $2 \in A$ and $3 \in A$, the pair $(2, 3)$ is in $R$.
If $x = 3$, then $y = 3+1 = 4$. Since $3 \in A$ and $4 \in A$, the pair $(3, 4)$ is in $R$.
If $x = 4$, then $y = 4+1 = 5$. Since $4 \in A$ and $5 \in A$, the pair $(4, 5)$ is in $R$.
If $x = 5$, then $y = 5+1 = 6$. Since $6 \notin A$, the pair $(5, 6)$ is not in $R$.
So, the relation $R$ is the set of ordered pairs:
$R = \{(1, 2), (2, 3), (3, 4), (4, 5)\}$
The range of a relation is the set of all second components (y-values) of the ordered pairs in the relation.
From the ordered pairs in $R$, the second components are $2, 3, 4, 5$.
Therefore, the range of $R$ is $\{2, 3, 4, 5\}$.
Comparing this result with the given options, we find that it matches option (B).
The final answer is $\boxed{\{2, 3, 4, 5\}}$.
Therefore, the correct option is (B) $\{2, 3, 4, 5\}$.
Question 5. Which of the following relations is a function from $A = \{1, 2, 3\}$ to $B = \{a, b, c, d\}$?
(A) $R_1 = \{(1, a), (2, b), (3, c), (1, d)\}$
(B) $R_2 = \{(1, a), (2, b)\}$
(C) $R_3 = \{(1, a), (2, a), (3, a)\}$
(D) $R_4 = \{(1, a), (2, b), (3, c), (4, d)\}$
Answer:
Solution:
Given:
Set $A = \{1, 2, 3\}$.
Set $B = \{a, b, c, d\}$.
Four relations $R_1, R_2, R_3, R_4$.
To Determine:
Which of the given relations is a function from set $A$ to set $B$.
A relation $f$ from a set $A$ to a set $B$ is called a function if every element of set $A$ has one and only one image in set $B$. This means two conditions must be satisfied:
1. Every element in set $A$ must be the first element of at least one ordered pair in the relation.
2. No element in set $A$ can be the first element of more than one ordered pair in the relation (i.e., no element in $A$ can be mapped to more than one element in $B$).
Let's examine each relation:
(A) $R_1 = \{(1, a), (2, b), (3, c), (1, d)\}$
The elements of set $A$ are $1, 2, 3$.
In $R_1$, the first elements of the ordered pairs are $1, 2, 3, 1$.
The element $1 \in A$ appears as the first element in two ordered pairs: $(1, a)$ and $(1, d)$. This means that the element $1$ in set $A$ is associated with two different elements ($a$ and $d$) in set $B$. This violates the second condition for a function.
Therefore, $R_1$ is not a function from $A$ to $B$.
(B) $R_2 = \{(1, a), (2, b)\}$
The elements of set $A$ are $1, 2, 3$.
In $R_2$, the first elements of the ordered pairs are $1, 2$.
The element $3 \in A$ does not appear as the first element of any ordered pair in $R_2$. This means that the element $3$ in set $A$ does not have an image in set $B$ under the relation $R_2$. This violates the first condition for a function.
Therefore, $R_2$ is not a function from $A$ to $B$.
(C) $R_3 = \{(1, a), (2, a), (3, a)\}$
The elements of set $A$ are $1, 2, 3$.
In $R_3$, the first elements of the ordered pairs are $1, 2, 3$. Every element of set $A$ is the first element of exactly one ordered pair.
- Element $1 \in A$ is mapped to $a \in B$. (One image)
- Element $2 \in A$ is mapped to $a \in B$. (One image)
- Element $3 \in A$ is mapped to $a \in B$. (One image)
Both conditions for a function are satisfied: every element in $A$ has an image, and each element has exactly one image (even though different elements in $A$ can map to the same element in $B$).
Therefore, $R_3$ is a function from $A$ to $B$.
(D) $R_4 = \{(1, a), (2, b), (3, c), (4, d)\}$
The elements of set $A$ are $1, 2, 3$.
In $R_4$, the first elements of the ordered pairs are $1, 2, 3, 4$.
The element $4$ is the first element of the pair $(4, d)$, but $4$ is not an element of set $A = \{1, 2, 3\}$. Thus, $R_4$ is not a relation from $A$ to $B$. For a relation to be a function from $A$ to $B$, its domain must be exactly $A$. The domain of $R_4$ is $\{1, 2, 3, 4\}$, which is not equal to $A$.
Therefore, $R_4$ is not a function from $A$ to $B$.
Based on the analysis, only relation $R_3$ satisfies the conditions to be a function from $A$ to $B$.
The final answer is $\boxed{R_3 = \{(1, a), (2, a), (3, a)\}}$.
Therefore, the correct option is (C) $R_3 = \{(1, a), (2, a), (3, a)\}$.
Question 6. The domain of the real function $f(x) = \frac{1}{x^2 - 4}$ is:
(A) $\mathbb{R}$
(B) $\mathbb{R} - \{2\}$
(C) $\mathbb{R} - \{-2\}$
(D) $\mathbb{R} - \{-2, 2\}$
Answer:
Solution:
Given:
The real function $f(x) = \frac{1}{x^2 - 4}$.
To Find:
The domain of the function $f(x)$.
For a real function defined as a fraction, the denominator cannot be equal to zero, because division by zero is undefined in the set of real numbers.
Therefore, for $f(x) = \frac{1}{x^2 - 4}$ to be defined, the denominator $x^2 - 4$ must not be equal to zero.
We need to find the values of $x$ for which the denominator is zero:
$x^2 - 4 = 0$
This is a difference of squares, which can be factored as $(x-2)(x+2)$.
So, $(x-2)(x+2) = 0$
For the product of two factors to be zero, at least one of the factors must be zero.
Either $x-2 = 0$ or $x+2 = 0$.
Solving for $x$ in the first equation:
$x = 2$
Solving for $x$ in the second equation:
$x = -2$
Thus, the denominator $x^2 - 4$ is equal to zero when $x = 2$ or $x = -2$. These are the values of $x$ for which the function is undefined.
The domain of the function is the set of all real numbers except the values that make the denominator zero.
Domain $= \mathbb{R} - \{-2, 2\}$
Comparing this result with the given options, we find that it matches option (D).
The final answer is $\boxed{\mathbb{R} - \{-2, 2\}}$.
Therefore, the correct option is (D) $\mathbb{R} - \{-2, 2\}$.
Question 7. The range of the function $f(x) = |x - 1|$ is:
(A) $[1, \infty)$
(B) $(-\infty, \infty)$
(C) $[0, \infty)$
(D) $(-\infty, 1]$
Answer:
Solution:
Given:
The function $f(x) = |x - 1|$.
To Find:
The range of the function $f(x)$.
The function is defined as $f(x) = |x - 1|$. The absolute value of any real number is always non-negative. That is, for any real number $y$, $|y| \geq 0$.
In this function, the expression inside the absolute value is $(x - 1)$. As $x$ takes any real value, $(x - 1)$ can take any real value (positive, negative, or zero).
However, the absolute value of $(x - 1)$, which is $|x - 1|$, will always be greater than or equal to zero.
$|x - 1| \geq 0$ for all real values of $x$.
The minimum value of $|x - 1|$ is $0$, which occurs when $x - 1 = 0$, i.e., when $x = 1$.
$f(1) = |1 - 1| = |0| = 0$.
For any other real value of $x$, $|x - 1|$ will be a positive real number. As $x$ moves away from $1$ (in either direction), $|x - 1|$ increases and can take any positive real value.
Therefore, the set of all possible values that $f(x)$ can take is the set of all non-negative real numbers.
The range of $f(x)$ is $[0, \infty)$.
Comparing this result with the given options, we find that it matches option (C).
The final answer is $\boxed{[0, \infty)}$.
Therefore, the correct option is (C) $[0, \infty)$.
Question 8. Assertion (A): The relation $R = \{(x, x^2) : x \text{ is a prime number less than } 10\}$ is a function.
Reason (R): Every element in the domain of $R$ has a unique image.
(A) Both A and R are true and R is the correct explanation of A.
(B) Both A and R are true but R is not the correct explanation of A.
(C) A is true but R is false.
(D) A is false but R is true.
Answer:
Solution:
Given:
Assertion (A): The relation $R = \{(x, x^2) : x \text{ is a prime number less than } 10\}$ is a function.
Reason (R): Every element in the domain of $R$ has a unique image.
Let's first determine the relation $R$.
The prime numbers less than 10 are 2, 3, 5, and 7.
So, the set of $x$ values in the relation is $\{2, 3, 5, 7\}$. This set is the domain of the relation $R$.
Now, let's find the ordered pairs $(x, x^2)$ for each $x$ in the domain:
- For $x=2$, $x^2 = 2^2 = 4$. The ordered pair is $(2, 4)$.
- For $x=3$, $x^2 = 3^2 = 9$. The ordered pair is $(3, 9)$.
- For $x=5$, $x^2 = 5^2 = 25$. The ordered pair is $(5, 25)$.
- For $x=7$, $x^2 = 7^2 = 49$. The ordered pair is $(7, 49)$.
The relation $R$ is therefore $R = \{(2, 4), (3, 9), (5, 25), (7, 49)\}$.
Checking Assertion (A):
A relation is a function if and only if every element in its domain has exactly one image.
The domain of $R$ is $\{2, 3, 5, 7\}$.
Let's check if each element in the domain has a unique image:
- The element 2 is related only to 4. Its image is unique.
- The element 3 is related only to 9. Its image is unique.
- The element 5 is related only to 25. Its image is unique.
- The element 7 is related only to 49. Its image is unique.
Every element in the domain $\{2, 3, 5, 7\}$ appears exactly once as the first component in the ordered pairs of $R$. This means that every element in the domain has one and only one image.
Therefore, the relation $R$ is a function.
Assertion (A) is True.
Checking Reason (R):
Reason (R) states: Every element in the domain of $R$ has a unique image.
This statement is precisely the definition of a function from its domain to its range.
From our analysis above, we found that every element in the domain of $R$ (which is $\{2, 3, 5, 7\}$) does indeed have a unique image in the range (which is $\{4, 9, 25, 49\}$).
Therefore, Reason (R) is True.
Relation between A and R:
Assertion (A) claims that $R$ is a function. Reason (R) provides the condition that defines a function: every element in the domain has a unique image.
Our process for checking if $R$ is a function directly used the condition stated in Reason (R). We verified that every element in the domain of $R$ has a unique image, and based on this, we concluded that $R$ is a function.
Therefore, Reason (R) is the correct explanation for Assertion (A).
Both Assertion (A) and Reason (R) are true, and Reason (R) is the correct explanation for Assertion (A).
The final answer is $\boxed{\text{Both A and R are true and R is the correct explanation of A}}$.
Therefore, the correct option is (A) Both A and R are true and R is the correct explanation of A.
Question 9. If $f(x) = x^2$ and $g(x) = 2x+1$, find $(f+g)(x)$.
(A) $x^2(2x+1)$
(B) $x^2 + 2x + 1$
(C) $x^2 - 2x - 1$
(D) $(2x+1)^2$
Answer:
Solution:
Given:
The function $f(x) = x^2$.
The function $g(x) = 2x+1$.
To Find:
The sum of the functions $(f+g)(x)$.
The sum of two functions $f$ and $g$, denoted by $(f+g)$, is defined for all $x$ in the intersection of the domains of $f$ and $g$ as:
$(f+g)(x) = f(x) + g(x)$
Substitute the given expressions for $f(x)$ and $g(x)$ into the definition:
$(f+g)(x) = (x^2) + (2x+1)$
Simplify the expression by removing the parentheses:
$(f+g)(x) = x^2 + 2x + 1$
Thus, the sum of the functions $f(x)$ and $g(x)$ is $x^2 + 2x + 1$.
The final answer is $\boxed{x^2 + 2x + 1}$.
Comparing this result with the given options, we find that it matches option (B).
Therefore, the correct option is (B) $x^2 + 2x + 1$.
Question 10. Match the following functions with their types:
(i) $f(x) = 5$
(ii) $f(x) = x$
(iii) $f(x) = |x|$
(iv) $f(x) = \lfloor x \rfloor$
(a) Identity Function
(b) Constant Function
(c) Modulus Function
(d) Greatest Integer Function
(A) (i)-(a), (ii)-(b), (iii)-(c), (iv)-(d)
(B) (i)-(b), (ii)-(a), (iii)-(c), (iv)-(d)
(C) (i)-(b), (ii)-(a), (iii)-(d), (iv)-(c)
(D) (i)-(a), (ii)-(b), (iii)-(d), (iv)-(c)
Answer:
Solution:
We need to match each given function with its correct type.
(i) The function $f(x) = 5$. This function assigns the same constant value (5) to every input $x$. This is the definition of a Constant Function.
So, (i) matches with (b).
(ii) The function $f(x) = x$. This function assigns the input value itself as the output value. This is the definition of an Identity Function.
So, (ii) matches with (a).
(iii) The function $f(x) = |x|$. This function assigns the absolute value of the input $x$ as the output value. This is the definition of a Modulus Function.
So, (iii) matches with (c).
(iv) The function $f(x) = \lfloor x \rfloor$. This function assigns the greatest integer less than or equal to the input $x$ as the output value. This is the definition of the Greatest Integer Function.
So, (iv) matches with (d).
Based on the matching:
- (i) - (b)
- (ii) - (a)
- (iii) - (c)
- (iv) - (d)
Now let's check the given options:
(A) (i)-(a), (ii)-(b), (iii)-(c), (iv)-(d) - Incorrect
(B) (i)-(b), (ii)-(a), (iii)-(c), (iv)-(d) - Correct
(C) (i)-(b), (ii)-(a), (iii)-(d), (iv)-(c) - Incorrect
(D) (i)-(a), (ii)-(b), (iii)-(d), (iv)-(c) - Incorrect
The final answer is $\boxed{(i)-(b), (ii)-(a), (iii)-(c), (iv)-(d)}$.
Therefore, the correct option is (B) (i)-(b), (ii)-(a), (iii)-(c), (iv)-(d).
Question 11. The domain of the function $f(x) = \sqrt{9 - x^2}$ is:
(A) $(-3, 3)$
(B) $[-3, 3]$
(C) $(-\infty, -3] \cup [3, \infty)$
(D) $(-\infty, \infty)$
Answer:
Solution:
Given:
The real function $f(x) = \sqrt{9 - x^2}$.
To Find:
The domain of the function $f(x)$.
For the function $f(x) = \sqrt{9 - x^2}$ to yield a real number, the expression under the square root must be non-negative (greater than or equal to zero).
So, we must have:
$9 - x^2 \geq 0$
We can rewrite the inequality as:
$-x^2 \geq -9$
Multiplying both sides by -1 and reversing the inequality sign:
$x^2 \leq 9$
Taking the square root of both sides gives:
$\sqrt{x^2} \leq \sqrt{9}$
$|x| \leq 3$
The inequality $|x| \leq k$ for $k > 0$ is equivalent to $-k \leq x \leq k$.
In this case, $k=3$. So, the inequality $|x| \leq 3$ is equivalent to:
$-3 \leq x \leq 3$
This means that $x$ must be a real number between -3 and 3, including -3 and 3.
In interval notation, the set of all such $x$ is $[-3, 3]$.
Thus, the domain of the function $f(x) = \sqrt{9 - x^2}$ is the closed interval $[-3, 3]$.
The final answer is $\boxed{[-3, 3]}$.
Comparing this result with the given options, we find that it matches option (B).
Therefore, the correct option is (B) $[-3, 3]$.
Question 12. If $f(x) = x+1$ and $g(x) = x-1$, find $(f \cdot g)(x)$.
(A) $x^2 - 1$
(B) $x^2 + 1$
(C) $2x$
(D) 2
Answer:
Solution:
Given:
The function $f(x) = x+1$.
The function $g(x) = x-1$.
To Find:
The product of the functions $(f \cdot g)(x)$.
The product of two functions $f$ and $g$, denoted by $(f \cdot g)$, is defined for all $x$ in the intersection of the domains of $f$ and $g$ as:
$(f \cdot g)(x) = f(x) \cdot g(x)$
Substitute the given expressions for $f(x)$ and $g(x)$ into the definition:
$(f \cdot g)(x) = (x+1) \cdot (x-1)$
Use the difference of squares formula, which states that $(a+b)(a-b) = a^2 - b^2$. In this case, $a=x$ and $b=1$.
$(f \cdot g)(x) = x^2 - 1^2$
$(f \cdot g)(x) = x^2 - 1$
Thus, the product of the functions $f(x)$ and $g(x)$ is $x^2 - 1$.
The final answer is $\boxed{x^2 - 1}$.
Comparing this result with the given options, we find that it matches option (A).
Therefore, the correct option is (A) $x^2 - 1$.
Question 13. Which of the following is NOT true for functions $f$ and $g$?
(A) $(f+g)(x) = f(x) + g(x)$
(B) $(f-g)(x) = f(x) - g(x)$
(C) $(f \cdot g)(x) = f(x) \cdot g(x)$
(D) $(\frac{f}{g})(x) = \frac{f(x)}{g(x)}$, where $g(x) = 0$
Answer:
Solution:
We are asked to identify the statement that is NOT true among the given options regarding operations on functions $f$ and $g$. Let $D_f$ be the domain of $f$ and $D_g$ be the domain of $g$.
Let's review the definitions of the basic operations on functions:
(A) Sum of functions: The sum of $f$ and $g$, denoted by $f+g$, is defined by:
$(f+g)(x) = f(x) + g(x)$
This definition is valid for all $x$ in the intersection of the domains of $f$ and $g$, i.e., for $x \in D_f \cap D_g$.
Statement (A) is the standard definition and is True.
(B) Difference of functions: The difference of $f$ and $g$, denoted by $f-g$, is defined by:
$(f-g)(x) = f(x) - g(x)$
This definition is valid for all $x$ in the intersection of the domains of $f$ and $g$, i.e., for $x \in D_f \cap D_g$.
Statement (B) is the standard definition and is True.
(C) Product of functions: The product of $f$ and $g$, denoted by $f \cdot g$, is defined by:
$(f \cdot g)(x) = f(x) \cdot g(x)$
This definition is valid for all $x$ in the intersection of the domains of $f$ and $g$, i.e., for $x \in D_f \cap D_g$.
Statement (C) is the standard definition and is True.
(D) Quotient of functions: The quotient of $f$ and $g$, denoted by $\frac{f}{g}$, is defined by:
$(\frac{f}{g})(x) = \frac{f(x)}{g(x)}$
This definition is valid for all $x$ in the intersection of the domains of $f$ and $g$, such that $g(x) \neq 0$. The domain of $(\frac{f}{g})(x)$ is $\{x \in D_f \cap D_g \mid g(x) \neq 0\}$.
Statement (D) says $(\frac{f}{g})(x) = \frac{f(x)}{g(x)}$, where $g(x) = 0$. This statement implies that the quotient function is defined even when the denominator $g(x)$ is zero. However, division by zero is undefined in mathematics.
Therefore, the equation $(\frac{f}{g})(x) = \frac{f(x)}{g(x)}$ is not defined for values of $x$ where $g(x) = 0$. The statement claiming it holds "where $g(x) = 0$" is fundamentally incorrect because the function itself is not defined at such points.
Statement (D) is False.
Based on the analysis, the statement that is NOT true is (D).
The final answer is $\boxed{(\frac{f}{g})(x) = \frac{f(x)}{g(x)}, \text{ where } g(x) = 0}$.
Therefore, the correct option is (D) $(\frac{f}{g})(x) = \frac{f(x)}{g(x)}$, where $g(x) = 0$.
Question 14. The range of the Signum function, $f(x) = \begin{cases} 1 & , & x > 0 \\ 0 & , & x = 0 \\ -1 & , & x < 0 \end{cases}$ is:
(A) $\mathbb{R}$
(B) $\{1, 0, -1\}$
(C) $\{-1, 1\}$
(D) $(-\infty, -1] \cup [1, \infty)$
Answer:
Solution:
Given:
The Signum function, $f(x) = \begin{cases} 1 & , & x > 0 \\ 0 & , & x = 0 \\ -1 & , & x < 0 \end{cases}$.
To Find:
The range of the function $f(x)$.
The range of a function is the set of all possible output values ($f(x)$) that the function can produce for inputs in its domain.
The domain of the Signum function is all real numbers, $\mathbb{R}$.
We examine the definition of the function for different intervals of $x$:
1. When $x > 0$ (e.g., $x=5$, $x=0.1$), the function definition states that $f(x) = 1$. So, 1 is a possible output value.
2. When $x = 0$, the function definition states that $f(x) = 0$. So, 0 is a possible output value.
3. When $x < 0$ (e.g., $x=-2$, $x=-0.001$), the function definition states that $f(x) = -1$. So, -1 is a possible output value.
These are the only three cases for any real number $x$. Therefore, the only possible output values of the function $f(x)$ are 1, 0, and -1.
The set of all possible output values is $\{-1, 0, 1\}$.
The range of the Signum function is the set $\{-1, 0, 1\}$.
The final answer is $\boxed{\{1, 0, -1\}}$.
Comparing this result with the given options, we find that it matches option (B).
Therefore, the correct option is (B) $\{1, 0, -1\}$.
Question 15. Case Study:
In a class of students, let $A$ be the set of students and $B$ be the set of marks obtained in a test out of 50. Consider the relation $R$ from $A$ to $B$ defined by $(s, m) \in R$ if student $s$ obtained $m$ marks. The following data shows the marks of 5 students:
| Student Name | Marks |
| Amit | 45 |
| Bina | 38 |
| Chirag | 45 |
| Divya | 40 |
| Eshan | 38 |
Which of the following statements is true about the relation $R$ represented by this data?
(A) It is a function because each student is mapped to a unique mark.
(B) It is a function because each mark is associated with a unique student.
(C) It is a relation but not a function because Chirag and Amit have the same mark.
(D) It is a relation but not a function because two students have the same mark.
Answer:
Solution:
Given:
Set of students $A = \{\text{Amit, Bina, Chirag, Divya, Eshan}\}$.
Relation $R$ from $A$ to $B$ defined by $(s, m) \in R$ if student $s$ obtained $m$ marks.
The given data is: Amit (45), Bina (38), Chirag (45), Divya (40), Eshan (38).
To Determine:
Whether the relation $R$ is a function and identify the correct statement about it.
The relation $R$ is a set of ordered pairs $(s, m)$, where $s$ is a student from set $A$ and $m$ is the mark obtained. Based on the given data, we can write the relation $R$ as:
$R = \{(\text{Amit}, 45), (\text{Bina}, 38), (\text{Chirag}, 45), (\text{Divya}, 40), (\text{Eshan}, 38)\}$
For a relation from set $A$ to set $B$ to be a function, two conditions must be satisfied:
1. Every element in the domain (set $A$) must be associated with some element in the codomain (set $B$). In terms of ordered pairs, every element of $A$ must appear as the first component in at least one ordered pair in the relation.
2. Every element in the domain (set $A$) must be associated with a unique element in the codomain (set $B$). In terms of ordered pairs, no element of $A$ can appear as the first component in more than one ordered pair.
Let's check these conditions for the given relation $R$ with domain $A = \{\text{Amit, Bina, Chirag, Divya, Eshan}\}$.
1. Check if every student in $A$ is in the domain of $R$ (appears as a first component):
- Amit is the first component in $(\text{Amit}, 45)$.
- Bina is the first component in $(\text{Bina}, 38)$.
- Chirag is the first component in $(\text{Chirag}, 45)$.
- Divya is the first component in $(\text{Divya}, 40)$.
- Eshan is the first component in $(\text{Eshan}, 38)$.
All elements in set $A$ appear as the first component. So, condition 1 is satisfied.
2. Check if each student in $A$ is associated with a unique mark (appears as a first component in only one ordered pair):
- Amit appears only in $(\text{Amit}, 45)$. Unique image (45).
- Bina appears only in $(\text{Bina}, 38)$. Unique image (38).
- Chirag appears only in $(\text{Chirag}, 45)$. Unique image (45).
- Divya appears only in $(\text{Divya}, 40)$. Unique image (40).
- Eshan appears only in $(\text{Eshan}, 38)$. Unique image (38).
Each student in set $A$ appears exactly once as the first component. This means each student has one and only one mark associated with them according to the relation $R$. So, condition 2 is satisfied.
Since both conditions are satisfied, the relation $R$ is a function from $A$ to $B$.
It is important to note that having different students getting the same mark (e.g., Amit and Chirag both got 45) does not violate the definition of a function. This is a characteristic of a many-to-one function, which is a valid type of function.
Now let's evaluate the given statements:
(A) It is a function because each student is mapped to a unique mark.
This statement correctly identifies the relation as a function and provides the correct reason based on the definition of a function. Every element in the domain (student) is mapped to exactly one element in the codomain (mark).
(B) It is a function because each mark is associated with a unique student.
This statement describes a one-to-one function. The relation $R$ is not one-to-one because the marks 45 and 38 are associated with more than one student. Thus, this statement is incorrect.
(C) It is a relation but not a function because Chirag and Amit have the same mark.
As explained earlier, having the same mark for different students does not prevent a relation from being a function. This statement is incorrect.
(D) It is a relation but not a function because two students have the same mark.
Similar to (C), this statement provides an incorrect reason for the relation not being a function. This statement is incorrect.
Based on our analysis, statement (A) is the true statement.
The final answer is $\boxed{\text{It is a function because each student is mapped to a unique mark}}$.
Therefore, the correct option is (A) It is a function because each student is mapped to a unique mark.
Question 16. (Continuation of Case Study) What is the domain of the relation $R$?
(A) $\{$Amit, Bina, Chirag, Divya, Eshan$\}$
(B) $\{38, 40, 45\}$
(C) $\{38, 40, 45\}$, $\{$Amit, Bina, Chirag, Divya, Eshan$\}$
(D) The set of all students in the class.
Answer:
Solution:
Given:
The relation $R$ is from the set of students $A$ to the set of marks $B$, defined by $(s, m) \in R$ if student $s$ obtained $m$ marks.
The relation is represented by the following data for 5 students:
| Student Name | Marks |
| Amit | 45 |
| Bina | 38 |
| Chirag | 45 |
| Divya | 40 |
| Eshan | 38 |
To Find:
The domain of the relation $R$.
The relation $R$ can be written as a set of ordered pairs $(s, m)$:
$R = \{(\text{Amit}, 45), (\text{Bina}, 38), (\text{Chirag}, 45), (\text{Divya}, 40), (\text{Eshan}, 38)\}$
The domain of a relation is the set of all first components of the ordered pairs in the relation.
In the ordered pairs of $R$, the first components are the names of the students: Amit, Bina, Chirag, Divya, Eshan.
Therefore, the domain of the relation $R$ is the set containing these names.
Domain of $R = \{\text{Amit, Bina, Chirag, Divya, Eshan}\}$
Comparing this set with the given options:
(A) $\{$Amit, Bina, Chirag, Divya, Eshan$\}$ - This matches our calculated domain.
(B) $\{38, 40, 45\}$ - This is the set of all second components, which is the range of the relation.
(C) $\{38, 40, 45\}$, $\{$Amit, Bina, Chirag, Divya, Eshan$\}$ - This lists both the range and the domain, not just the domain.
(D) The set of all students in the class. The problem states that $A$ is the set of students in the class, and based on the data, the domain of $R$ is equal to $A$. Option (A) lists the elements of $A$. Both (A) and (D) represent the same set, but (A) explicitly lists the elements derived from the relation data.
Option (A) is the explicit set of elements that constitute the domain of the relation $R$ as given by the data.
The final answer is $\boxed{\{\text{Amit, Bina, Chirag, Divya, Eshan}\}}$.
Therefore, the correct option is (A) $\{$Amit, Bina, Chirag, Divya, Eshan$\}$.
Question 17. (Continuation of Case Study) What is the range of the relation $R$?
(A) $\{$Amit, Bina, Chirag, Divya, Eshan$\}$
(B) $\{38, 40, 45\}$
(C) $\{38, 40, 45\}$, $\{$Amit, Bina, Chirag, Divya, Eshan$\}$
(D) The set of all possible marks out of 50.
Answer:
Solution:
Given:
The relation $R$ from the set of students $A$ to the set of marks $B$, defined by $(s, m) \in R$ if student $s$ obtained $m$ marks.
The relation is represented by the following data for 5 students:
| Student Name | Marks |
| Amit | 45 |
| Bina | 38 |
| Chirag | 45 |
| Divya | 40 |
| Eshan | 38 |
To Find:
The range of the relation $R$.
The relation $R$ can be written as a set of ordered pairs $(s, m)$, where $s$ is a student and $m$ is their mark:
$R = \{(\text{Amit}, 45), (\text{Bina}, 38), (\text{Chirag}, 45), (\text{Divya}, 40), (\text{Eshan}, 38)\}$
The range of a relation is the set of all second components of the ordered pairs in the relation.
In the ordered pairs of $R$, the second components (the marks obtained) are: 45, 38, 45, 40, 38.
To find the range, we collect all the unique second components into a set.
The unique second components are 38, 40, and 45.
Therefore, the range of the relation $R$ is the set $\{38, 40, 45\}$.
Comparing this set with the given options:
(A) $\{$Amit, Bina, Chirag, Divya, Eshan$\}$ - This is the domain of the relation.
(B) $\{38, 40, 45\}$ - This matches our calculated range.
(C) $\{38, 40, 45\}$, $\{$Amit, Bina, Chirag, Divya, Eshan$\}$ - This lists both the range and the domain.
(D) The set of all possible marks out of 50. This is the codomain, which could be $\{0, 1, 2, ..., 50\}$. The range is a subset of the codomain containing only the marks actually obtained by the students in the relation.
Based on our calculation, the range of the relation $R$ is $\{38, 40, 45\}$.
The final answer is $\boxed{\{38, 40, 45\}}$.
Therefore, the correct option is (B) $\{38, 40, 45\}$.
Question 18. If $A = \{p, q, r\}$ and $B = \{1, 2\}$. Which of the following is a relation from $A$ to $B$?
(A) $\{(p, 1), (q, 2), (r, 1), (p, 2)\}$
(B) $\{(1, p), (2, q)\}$
(C) $\{(p, a), (q, b)\}$ where $a, b \in B$
(D) $\{(1, p), (2, q), (3, r)\}$
Answer:
Solution:
Given:
Set $A = \{p, q, r\}$.
Set $B = \{1, 2\}$.
To Determine:
Which of the given options is a relation from set $A$ to set $B$.
A relation from a set $A$ to a set $B$ is defined as any subset of the Cartesian product $A \times B$.
First, let's find the Cartesian product $A \times B$. This is the set of all ordered pairs $(a, b)$ where the first element $a$ is from set $A$, and the second element $b$ is from set $B$.
$A \times B = \{(x, y) \mid x \in A \text{ and } y \in B\}$
Using the given sets $A = \{p, q, r\}$ and $B = \{1, 2\}$:
$A \times B = \{(p, 1), (p, 2), (q, 1), (q, 2), (r, 1), (r, 2)\}$
Now, we check each given option to see if it is a subset of $A \times B$. A set is a subset of another if all its elements are contained within the other set.
(A) $\{(p, 1), (q, 2), (r, 1), (p, 2)\}$
Let's check each ordered pair in this set:
- $(p, 1)$: $p \in A$ and $1 \in B$. So, $(p, 1) \in A \times B$.
- $(q, 2)$: $q \in A$ and $2 \in B$. So, $(q, 2) \in A \times B$.
- $(r, 1)$: $r \in A$ and $1 \in B$. So, $(r, 1) \in A \times B$.
- $(p, 2)$: $p \in A$ and $2 \in B$. So, $(p, 2) \in A \times B$.
Since all the ordered pairs in option (A) are elements of $A \times B$, the set in option (A) is a subset of $A \times B$.
Therefore, option (A) is a relation from $A$ to $B$.
(B) $\{(1, p), (2, q)\}$
Let's check the first ordered pair:
- $(1, p)$: $1 \in B$ but $1 \notin A$. The first element must be from $A$ for the pair to be in $A \times B$. So, $(1, p) \notin A \times B$.
Since not all elements are in $A \times B$, option (B) is not a relation from $A$ to $B$. (Note: This is a relation from $B$ to $A$).
(C) $\{(p, a), (q, b)\}$ where $a, b \in B$
This option describes a set of ordered pairs where the first components are $p$ and $q$ (which are in $A$), and the second components $a$ and $b$ are from $B = \{1, 2\}$. Any pair $(p, a)$ with $a \in B$ is in $A \times B$, and any pair $(q, b)$ with $b \in B$ is in $A \times B$. Thus, a set of the form $\{(p, a), (q, b)\}$ where $a, b \in B$ is indeed a subset of $A \times B$. Examples include $\{(p, 1), (q, 1)\}$, $\{(p, 1), (q, 2)\}$, $\{(p, 2), (q, 1)\}$, $\{(p, 2), (q, 2)\}$. While this describes a form of relation from A to B, option (A) provides a specific, concrete set that is a relation.
(D) $\{(1, p), (2, q), (3, r)\}$
Let's check the ordered pairs:
- $(1, p)$: $1 \notin A$. So, $(1, p) \notin A \times B$.
- $(2, q)$: $2 \notin A$. So, $(2, q) \notin A \times B$.
- $(3, r)$: $3$ is not an element of $A$ or $B$. So, $(3, r) \notin A \times B$.
Since none of the elements are in $A \times B$, option (D) is not a relation from $A$ to $B$.
Comparing the options, only option (A) provides a set where all ordered pairs have the first component from $A$ and the second component from $B$. Thus, only (A) is a subset of $A \times B$ and is a relation from $A$ to $B$.
The final answer is $\boxed{\{(p, 1), (q, 2), (r, 1), (p, 2)\}}$.
Therefore, the correct option is (A) $\{(p, 1), (q, 2), (r, 1), (p, 2)\}$.
Question 19. Let $f(x) = \frac{x+1}{x-1}$. The domain of $f$ is:
(A) $\mathbb{R} - \{1\}$
(B) $\mathbb{R} - \{-1\}$
(C) $\mathbb{R} - \{0\}$
(D) $\mathbb{R}$
Answer:
Solution:
Given:
The function $f(x) = \frac{x+1}{x-1}$.
To Find:
The domain of the function $f(x)$.
The domain of a real function is the set of all real numbers for which the function is defined.
For a rational function (a function expressed as a ratio of two polynomials), the function is defined everywhere except where the denominator is equal to zero, because division by zero is undefined.
The function is $f(x) = \frac{x+1}{x-1}$. The denominator is $x-1$.
The function is undefined when the denominator is zero.
Set the denominator equal to zero to find the values of $x$ that are excluded from the domain:
$x - 1 = 0$
Solving for $x$:
$x = 1$
This means that the function $f(x)$ is undefined when $x=1$. For all other real values of $x$, the denominator $x-1$ is non-zero, and the function is defined.
Therefore, the domain of the function $f(x)$ is the set of all real numbers ($\mathbb{R}$) excluding the value $1$.
Domain of $f = \mathbb{R} - \{1\}$
Comparing this result with the given options, we find that it matches option (A).
The final answer is $\boxed{\mathbb{R} - \{1\}}$.
Therefore, the correct option is (A) $\mathbb{R} - \{1\}$.
Question 20. Which of the following graphs represents a function?
(A) Graph A (Vertical line passing through two points on the curve)
(B) Graph B (Circle)
(C) Graph C (Parabola opening upwards)
(D) Graph D (Parabola opening sideways)
Answer:
Solution:
To Determine:
Which of the given graphs represents a function.
A graph represents a function if and only if every vertical line intersects the graph at most once. This is known as the Vertical Line Test.
Let's analyze each option based on the Vertical Line Test:
(A) Graph A (Vertical line passing through two points on the curve)
The description states that a vertical line passes through two points on the curve. This means there exists at least one vertical line that intersects the graph at more than one point. According to the Vertical Line Test, this graph does not represent a function.
(B) Graph B (Circle)
For a circle that is not a single point, it is possible to draw a vertical line that intersects the circle at two distinct points (an upper half and a lower half for the same x-coordinate, within the diameter). For example, the graph of $x^2 + y^2 = r^2$ (a circle centered at the origin with radius $r$) fails the vertical line test for any $x$ such that $-r < x < r$, where $y = \pm\sqrt{r^2 - x^2}$. Thus, a circle does not represent a function of $x$.
(C) Graph C (Parabola opening upwards)
A parabola opening upwards, like the graph of $y = x^2$, has the property that for every $x$ value in its domain, there is exactly one corresponding $y$ value. Graphically, any vertical line drawn across a parabola opening upwards will intersect the parabola at exactly one point. Therefore, a parabola opening upwards represents a function.
(D) Graph D (Parabola opening sideways)
A parabola opening sideways, like the graph of $x = y^2$, has the property that for a given $x$ value (specifically, for $x > 0$ in the case of $x=y^2$), there can be two corresponding $y$ values (a positive and a negative value, e.g., for $x=4$, $y=\pm 2$). Graphically, a vertical line can intersect a parabola opening sideways at two distinct points. Therefore, a parabola opening sideways does not represent a function of $x$.
Based on the Vertical Line Test, only the graph of a parabola opening upwards represents a function among the described options.
The final answer is $\boxed{\text{Graph C (Parabola opening upwards)}}$.
Therefore, the correct option is (C) Graph C (Parabola opening upwards).
Question 21. If $A = \{1, 2, 3\}$ and $B = \{a, b\}$. The number of relations from $A$ to $B$ is:
(A) $2^{3+2}$
(B) $2^{3 \times 2}$
(C) $3^2$
(D) $2^3$
Answer:
Solution:
Given:
Set $A$ with $n(A) = 3$ elements.
Set $B$ with $n(B) = 2$ elements.
To Find:
The number of relations from set $A$ to set $B$.
A relation from set $A$ to set $B$ is defined as any subset of the Cartesian product $A \times B$.
First, we find the number of elements in the Cartesian product $A \times B$. If set $A$ has $m$ elements and set $B$ has $n$ elements, then the number of elements in $A \times B$ is $m \times n$.
$n(A \times B) = n(A) \times n(B)$
Substitute the given values $n(A)=3$ and $n(B)=2$:
$n(A \times B) = 3 \times 2 = 6$
Now, the number of relations from $A$ to $B$ is equal to the number of possible subsets of $A \times B$. The number of subsets of a set with $k$ elements is $2^k$.
Number of relations from $A$ to $B = \text{Number of subsets of } A \times B = 2^{n(A \times B)}$
Substitute the calculated value of $n(A \times B)$:
Number of relations = $2^6$
Comparing this result with the given options:
(A) $2^{3+2} = 2^5$
(B) $2^{3 \times 2} = 2^6$
(C) $3^2 = 9$
(D) $2^3 = 8$
The calculated number of relations, $2^6$, matches option (B) because $3 \times 2 = 6$.
The final answer is $\boxed{2^{3 \times 2}}$.
Therefore, the correct option is (B) $2^{3 \times 2}$.
Question 22. If $f(x) = x^2$ and $g(x) = \sqrt{x}$, find the domain of $(\frac{f}{g})(x)$.
(A) $[0, \infty)$
(B) $(0, \infty)$
(C) $\mathbb{R}$
(D) $\mathbb{R} - \{0\}$
Answer:
Solution:
Given:
Function $f(x) = x^2$.
Function $g(x) = \sqrt{x}$.
To Find:
The domain of the function $(\frac{f}{g})(x)$.
The quotient of two functions $f$ and $g$, denoted by $(\frac{f}{g})$, is defined by:
$(\frac{f}{g})(x) = \frac{f(x)}{g(x)}$
The domain of $(\frac{f}{g})(x)$ is the set of all $x$ that are in the domain of $f$ AND in the domain of $g$, with the additional condition that $g(x) \neq 0$.
Domain$(\frac{f}{g}) = \{x \mid x \in \text{Domain}(f) \text{ and } x \in \text{Domain}(g) \text{ and } g(x) \neq 0\}$
First, find the domain of $f(x) = x^2$.
Since $f(x)$ is a polynomial, it is defined for all real numbers.
Domain$(f) = \mathbb{R}$ or $(-\infty, \infty)$.
Next, find the domain of $g(x) = \sqrt{x}$.
For $\sqrt{x}$ to be a real number, the expression under the square root must be non-negative.
So, $x \geq 0$.
Domain$(g) = \{x \in \mathbb{R} \mid x \geq 0\}$ or $[0, \infty)$.
Now, consider the condition $g(x) \neq 0$.
$g(x) = \sqrt{x}$
We need $\sqrt{x} \neq 0$.
$\sqrt{x} = 0$ only when $x = 0$.
So, we must exclude $x=0$ from the domain.
The domain of $(\frac{f}{g})(x)$ is the intersection of Domain$(f)$ and Domain$(g)$, excluding $x=0$.
Domain$(\frac{f}{g}) = \text{Domain}(f) \cap \text{Domain}(g) \cap \{x \mid g(x) \neq 0\}$
Domain$(\frac{f}{g}) = (-\infty, \infty) \cap [0, \infty) \cap \{x \mid x \neq 0\}$
The intersection of $(-\infty, \infty)$ and $[0, \infty)$ is $[0, \infty)$.
Now, we exclude $x=0$ from $[0, \infty)$.
$[0, \infty) - \{0\} = (0, \infty)$
Thus, the domain of $(\frac{f}{g})(x)$ is $(0, \infty)$.
The final answer is $\boxed{(0, \infty)}$.
Comparing this result with the given options, we find that it matches option (B).
Therefore, the correct option is (B) $(0, \infty)$.
Question 23. The domain of the function $f(x) = \frac{1}{\sqrt{x-5}}$ is:
(A) $[5, \infty)$
(B) $(5, \infty)$
(C) $(-\infty, 5)$
(D) $(-\infty, 5]$
Answer:
Solution:
Given:
The real function $f(x) = \frac{1}{\sqrt{x-5}}$.
To Find:
The domain of the function $f(x)$.
For the function $f(x)$ to be defined as a real number, two conditions must be met:
1. The expression under the square root must be non-negative.
2. The denominator cannot be equal to zero.
From condition 1, the expression inside the square root, $x-5$, must be greater than or equal to zero:
$x - 5 \geq 0$
Solving for $x$:
$x \geq 5$
From condition 2, the denominator $\sqrt{x-5}$ must not be equal to zero.
$\sqrt{x-5} \neq 0$
This means $x-5 \neq 0$.
Solving for $x$:
$x \neq 5$
We need to satisfy both conditions simultaneously. We need $x \geq 5$ AND $x \neq 5$.
Combining these two conditions, we get $x > 5$.
In interval notation, the set of all real numbers $x$ such that $x > 5$ is $(5, \infty)$.
Thus, the domain of the function $f(x) = \frac{1}{\sqrt{x-5}}$ is $(5, \infty)$.
The final answer is $\boxed{(5, \infty)}$.
Comparing this result with the given options, we find that it matches option (B).
Therefore, the correct option is (B) $(5, \infty)$.
Question 24. If $f(x) = 3x-2$, then $f(0)$ is:
(A) 0
(B) 3
(C) -2
(D) 1
Answer:
Solution:
Given:
The function $f(x) = 3x-2$.
To Find:
The value of $f(0)$.
To find the value of the function at $x=0$, we need to substitute $x=0$ into the expression for $f(x)$.
$f(x) = 3x - 2$
Substitute $x=0$ into the equation:
$f(0) = 3(0) - 2$
Perform the multiplication and subtraction:
$f(0) = 0 - 2$
$f(0) = -2$
Thus, the value of $f(0)$ is -2.
The final answer is $\boxed{-2}$.
Comparing this result with the given options, we find that it matches option (C).
Therefore, the correct option is (C) -2.
Question 25. Which of the following statements is true?
(A) Every relation is a function.
(B) Every function is a relation.
(C) A relation from $A$ to $B$ is a subset of $B \times A$.
(D) The range of a relation is the set of all first elements of the ordered pairs.
Answer:
Solution:
To Determine:
Which of the given statements is true.
Let's analyze each statement:
(A) Every relation is a function.
A relation is any subset of the Cartesian product of two sets. A function is a special type of relation where every element in the domain is mapped to exactly one element in the codomain.
Consider the relation $R = \{(1, a), (1, b)\}$ from set $A = \{1\}$ to set $B = \{a, b\}$. This is a relation because it is a subset of $A \times B = \{(1, a), (1, b)\}$. However, it is not a function because the element $1 \in A$ is related to two different elements, $a$ and $b$, in $B$.
Therefore, this statement is false.
(B) Every function is a relation.
By definition, a function $f$ from a set $A$ to a set $B$ is a relation from $A$ to $B$ such that every element of $A$ is related to exactly one element of $B$. This means that the set of ordered pairs defining the function is a subset of the Cartesian product $A \times B$, which is the definition of a relation from $A$ to $B$.
Therefore, every function is indeed a relation.
This statement is true.
(C) A relation from $A$ to $B$ is a subset of $B \times A$.
A relation from set $A$ to set $B$ is defined as any subset of the Cartesian product $A \times B$. The ordered pairs in $A \times B$ are of the form $(a, b)$ where $a \in A$ and $b \in B$. The ordered pairs in $B \times A$ are of the form $(b, a)$ where $b \in B$ and $a \in A$.
Unless $A=B$ or $A$ or $B$ is empty, $A \times B$ and $B \times A$ are different sets. A relation from $A$ to $B$ is a subset of $A \times B$, not $B \times A$.
Therefore, this statement is false.
(D) The range of a relation is the set of all first elements of the ordered pairs.
The domain of a relation is the set of all first components (elements from the first set) of the ordered pairs in the relation.
The range of a relation is the set of all second components (elements from the second set) of the ordered pairs in the relation.
Therefore, this statement incorrectly defines the range; it describes the domain.
This statement is false.
Based on the analysis, only statement (B) is true.
The final answer is $\boxed{\text{Every function is a relation}}$.
Therefore, the correct option is (B) Every function is a relation.
Question 26. Let $A = \{1, 2, 3\}$ and $R = \{(1, 1), (2, 2), (3, 3)\}$ be a relation on $A$. The domain and range of $R$ are:
(A) Domain = $\{1, 2, 3\}$, Range = $\{1, 2, 3\}$
(B) Domain = $\{1, 2, 3\}$, Range = $\{1, 2, 3, 4, 5, ...\}$
(C) Domain = $A \times A$, Range = $A \times A$
(D) Domain = $\{1, 2, 3\}$, Range = $\{1\}$
Answer:
Solution:
Given:
Set $A = \{1, 2, 3\}$.
Relation $R = \{(1, 1), (2, 2), (3, 3)\}$ on $A$. A relation on $A$ means a relation from $A$ to $A$.
To Find:
The domain and the range of the relation $R$.
The domain of a relation is the set of all first components of the ordered pairs in the relation.
The range of a relation is the set of all second components of the ordered pairs in the relation.
The given relation is $R = \{(1, 1), (2, 2), (3, 3)\}$.
To find the domain, we list all the unique first components from the ordered pairs in $R$:
The first components are 1, 2, and 3.
Domain of $R = \{1, 2, 3\}$
To find the range, we list all the unique second components from the ordered pairs in $R$:
The second components are 1, 2, and 3.
Range of $R = \{1, 2, 3\}$
So, the domain of $R$ is $\{1, 2, 3\}$ and the range of $R$ is $\{1, 2, 3\}$.
Note that in this specific case, the domain and range are equal to the set $A$.
Comparing our results with the given options:
(A) Domain = $\{1, 2, 3\}$, Range = $\{1, 2, 3\}$ - This matches our calculation.
(B) Domain = $\{1, 2, 3\}$, Range = $\{1, 2, 3, 4, 5, ...\}$ - The range is incorrect.
(C) Domain = $A \times A$, Range = $A \times A$ - $A \times A$ is the Cartesian product, not the domain or range.
(D) Domain = $\{1, 2, 3\}$, Range = $\{1\}$ - The range is incorrect.
The final answer is $\boxed{\text{Domain = } \{1, 2, 3\}, \text{ Range = } \{1, 2, 3\}}$.
Therefore, the correct option is (A) Domain = $\{1, 2, 3\}$, Range = $\{1, 2, 3\}$.
Question 27. If $f(x) = x^2 + 2x - 1$, find the value of $f(-1)$.
(A) 0
(B) -2
(C) 2
(D) -4
Answer:
Solution:
Given:
The function $f(x) = x^2 + 2x - 1$.
To Find:
The value of $f(-1)$.
To find the value of the function at $x=-1$, we substitute $x=-1$ into the expression for $f(x)$.
$f(x) = x^2 + 2x - 1$
Substitute $x=-1$ into the equation:
$f(-1) = (-1)^2 + 2(-1) - 1$
Evaluate the terms:
$(-1)^2 = 1$
$2(-1) = -2$
So, the expression becomes:
$f(-1) = 1 + (-2) - 1$
$f(-1) = 1 - 2 - 1$
Perform the subtraction:
$f(-1) = -1 - 1$
$f(-1) = -2$
Thus, the value of $f(-1)$ is -2.
The final answer is $\boxed{-2}$.
Comparing this result with the given options, we find that it matches option (B).
Therefore, the correct option is (B) -2.
Question 28. The graph of the identity function $f(x) = x$ is a straight line:
(A) Parallel to the x-axis
(B) Parallel to the y-axis
(C) Passing through the origin with slope 1
(D) Passing through the origin with slope -1
Answer:
Solution:
Given:
The identity function $f(x) = x$.
To Determine:
The characteristic of the graph of the identity function $f(x) = x$.
The graph of a function $f(x)$ is the set of all points $(x, y)$ in the Cartesian plane such that $y = f(x)$. For the identity function, $y = f(x) = x$. So the graph consists of all points $(x, y)$ where $y = x$.
Let's consider some points $(x, y)$ that satisfy $y=x$:
- If $x=0$, $y=0$. Point is $(0, 0)$. This point is the origin.
- If $x=1$, $y=1$. Point is $(1, 1)$.
- If $x=2$, $y=2$. Point is $(2, 2)$.
- If $x=-1$, $y=-1$. Point is $(-1, -1)$.
Plotting these points suggests that they lie on a straight line. The equation $y = x$ is the equation of a straight line in the form $y = mx + c$, where $m$ is the slope and $c$ is the y-intercept.
In the equation $y = x$, the slope $m = 1$ and the y-intercept $c = 0$.
A y-intercept of 0 means the line passes through the point where $x=0$ and $y=0$, which is the origin $(0, 0)$.
A slope of 1 means that for every 1 unit increase in $x$, the value of $y$ increases by 1 unit. The angle the line makes with the positive x-axis is $45^\circ$ since $\tan(45^\circ) = 1$.
Let's evaluate the options:
(A) Parallel to the x-axis: A line parallel to the x-axis has an equation of the form $y = c$ (where $c$ is a constant, and the slope is 0). This is not $y=x$. So, this is incorrect.
(B) Parallel to the y-axis: A line parallel to the y-axis has an equation of the form $x = c$ (where $c$ is a constant, and the slope is undefined). This is not $y=x$. So, this is incorrect.
(C) Passing through the origin with slope 1: As we found, the line $y=x$ has a y-intercept of 0, meaning it passes through the origin $(0, 0)$, and its slope is 1. This matches our analysis.
(D) Passing through the origin with slope -1: The line passing through the origin with slope -1 has the equation $y = -x$. This is not $y=x$. So, this is incorrect.
Therefore, the graph of the identity function $f(x) = x$ is a straight line passing through the origin with slope 1.
The final answer is $\boxed{\text{Passing through the origin with slope 1}}$.
Therefore, the correct option is (C) Passing through the origin with slope 1.
Question 29. The domain of the function $f(x) = \frac{1}{|x|}$ is:
(A) $\mathbb{R}$
(B) $\mathbb{R} - \{0\}$
(C) $(0, \infty)$
(D) $(-\infty, 0)$
Answer:
Solution:
Given:
The real function $f(x) = \frac{1}{|x|}$.
To Find:
The domain of the function $f(x)$.
The domain of a real function is the set of all real numbers for which the function is defined as a real number.
For the function $f(x) = \frac{1}{|x|}$ to be defined, the denominator $|x|$ cannot be equal to zero, because division by zero is undefined.
So, we must have:
$|x| \neq 0$
The absolute value of a real number $x$, denoted by $|x|$, is defined as:
$|x| = x$ if $x \geq 0$
$|x| = -x$ if $x < 0$
The absolute value $|x|$ is equal to zero if and only if $x$ is equal to zero.
$|x| = 0 \iff x = 0$
Since we require $|x| \neq 0$, this means we must exclude the value of $x$ for which $|x|$ is zero.
Therefore, $x \neq 0$.
The domain of the function $f(x) = \frac{1}{|x|}$ is the set of all real numbers ($\mathbb{R}$) except for the value $0$.
Domain of $f = \mathbb{R} - \{0\}$
Comparing this result with the given options, we find that it matches option (B).
The final answer is $\boxed{\mathbb{R} - \{0\}}$.
Therefore, the correct option is (B) $\mathbb{R} - \{0\}$.
Question 30. If $f(x) = c$ where $c$ is a constant, the range of the function is:
(A) $\mathbb{R}$
(B) $\{c\}$
(C) $[c, \infty)$
(D) $(-\infty, c]$
Answer:
Solution:
Given:
The function $f(x) = c$, where $c$ is a constant.
To Find:
The range of the function $f(x)$.
The range of a function is the set of all possible output values that the function can produce for inputs in its domain.
The given function is a constant function. The definition $f(x) = c$ means that for any input value of $x$ from the domain, the output value $f(x)$ is always the same constant value, $c$.
For example, if the domain is all real numbers $\mathbb{R}$:
- $f(1) = c$
- $f(0) = c$
- $f(-5) = c$
- $f(\sqrt{2}) = c$
Regardless of what real number we substitute for $x$, the function always returns the value $c$.
Therefore, the set of all possible output values of the function $f(x) = c$ is the set containing only the element $c$.
Range of $f = \{c\}$
Comparing this result with the given options:
(A) $\mathbb{R}$ - This represents all real numbers, which is the domain of a constant function, not its range (unless $c$ is defined in a way that encompasses all real numbers, which is not implied here).
(B) $\{c\}$ - This is the set containing the single constant value $c$, which is our calculated range.
(C) $[c, \infty)$ - This represents all real numbers greater than or equal to $c$. This is incorrect for a constant function.
(D) $(-\infty, c]$ - This represents all real numbers less than or equal to $c$. This is incorrect for a constant function.
The final answer is $\boxed{\{c\}}$.
Therefore, the correct option is (B) $\{c\}$.
Question 31. Completion Question: If the relation $R$ on $\{1, 2, 3, 4\}$ is given by $R = \{(1, 2), (2, 3), (3, 4)\}$, then the range of $R$ is ________.
(A) $\{1, 2, 3, 4\}$
(B) $\{1, 2, 3\}$
(C) $\{2, 3, 4\}$
(D) $\{4\}$
Answer:
Solution:
Given:
The relation $R$ on the set $\{1, 2, 3, 4\}$ is given by $R = \{(1, 2), (2, 3), (3, 4)\}$.
To Find:
The range of the relation $R$.
The range of a relation is the set of all second components of the ordered pairs in the relation.
The given relation $R$ consists of the following ordered pairs:
- $(1, 2)$
- $(2, 3)$
- $(3, 4)$
The second components of these ordered pairs are the values $2$, $3$, and $4$.
Collecting the unique second components into a set gives the range of $R$:
Range of $R = \{2, 3, 4\}$
Comparing this result with the given options, we find that it matches option (C).
The final answer is $\boxed{\{2, 3, 4\}}$.
Therefore, the correct option is (C) $\{2, 3, 4\}$.
Question 32. Let $f(x) = 2x+3$ and $g(x) = x-2$. The value of $(f-g)(4)$ is:
(A) 13
(B) 7
(C) 5
(D) 4
Answer:
Solution:
Given:
The function $f(x) = 2x+3$.
The function $g(x) = x-2$.
To Find:
The value of $(f-g)(4)$.
The difference of two functions $f$ and $g$, denoted by $(f-g)$, is defined for all $x$ in the intersection of the domains of $f$ and $g$ as:
$(f-g)(x) = f(x) - g(x)$
Substitute the given expressions for $f(x)$ and $g(x)$:
$(f-g)(x) = (2x+3) - (x-2)$
Remove the parentheses and simplify the expression:
$(f-g)(x) = 2x + 3 - x + 2$
$(f-g)(x) = (2x - x) + (3 + 2)$
$(f-g)(x) = x + 5$
Now, we need to find the value of $(f-g)(4)$. Substitute $x=4$ into the expression $x+5$:
$(f-g)(4) = 4 + 5$
$(f-g)(4) = 9$
The calculated value of $(f-g)(4)$ is 9.
Let's compare this result with the given options:
(A) 13
(B) 7
(C) 5
(D) 4
The calculated result (9) is not present among the given options. There appears to be an error in the provided options for the question as stated.
However, if the question intended to ask for the value of the composite function $(f \circ g)(4)$, the calculation would be:
$(f \circ g)(x) = f(g(x)) = f(x-2) = 2(x-2) + 3 = 2x - 4 + 3 = 2x - 1$
$(f \circ g)(4) = 2(4) - 1 = 8 - 1 = 7$.
This result (7) matches option (B).
Given that a multiple-choice answer is expected, it is probable that the intended operation was composition $(f \circ g)$ instead of difference $(f-g)$. Assuming this likely typographical error in the question's notation, the intended answer would be 7.
Based on the literal reading of the question, the answer is 9, which is not in the options. Based on the options provided, the question likely intended to ask for $(f \circ g)(4)$. Assuming the latter intention, the correct option is (B).
The final answer calculated as per the question is 9.
Assuming a typo in the question and that $(f \circ g)(4)$ was intended, the result is 7.
Assuming the question meant to ask for $(f \circ g)(4)$ to align with the options, the correct option is (B) 7.
Question 33. Data Interpretation:
A small shop recorded the number of cold drinks (in bottles) sold over 5 days. The data is as follows:
| Day | Monday | Tuesday | Wednesday | Thursday | Friday |
| Bottles Sold | 20 | 15 | 20 | 25 | 30 |
Let $D$ be the set of days and $N$ be the number of bottles sold. The relation $R$ from $D$ to $N$ is given by $R = \{(d, n) : \text{on day } d, n \text{ bottles were sold}\}$. Is this relation a function? If yes, why?
(A) Yes, because each day is mapped to a unique number of bottles sold.
(B) Yes, because each number of bottles sold is mapped to a unique day.
(C) No, because the same number of bottles (20) was sold on Monday and Wednesday.
(D) No, because the number of bottles sold is a numerical value.
Answer:
Solution:
Given:
Set of days $D = \{\text{Monday, Tuesday, Wednesday, Thursday, Friday}\}$.
Set $N$ containing the number of bottles sold on these days.
Relation $R$ from $D$ to $N$ is given by $R = \{(d, n) : \text{on day } d, n \text{ bottles were sold}\}$.
The data is:
| Day | Bottles Sold |
| Monday | 20 |
| Tuesday | 15 |
| Wednesday | 20 |
| Thursday | 25 |
| Friday | 30 |
To Determine:
Whether the relation $R$ is a function and the correct reasoning.
The relation $R$ can be written as a set of ordered pairs $(d, n)$, where $d$ is a day from set $D$ and $n$ is the number of bottles sold on that day.
$R = \{(\text{Monday}, 20), (\text{Tuesday}, 15), (\text{Wednesday}, 20), (\text{Thursday}, 25), (\text{Friday}, 30)\}$
For a relation from set $D$ to set $N$ to be a function, two conditions must be satisfied:
1. Every element in the domain (set $D$) must be associated with some element in the codomain (set $N$). Every day in $D$ must appear as the first component in at least one ordered pair in $R$.
2. Every element in the domain (set $D$) must be associated with a unique element in the codomain (set $N$). No day in $D$ can appear as the first component in more than one ordered pair.
Let's check these conditions for the given relation $R$ with domain $D = \{\text{Monday, Tuesday, Wednesday, Thursday, Friday}\}$.
1. Check if every day in $D$ is in the domain of $R$ (appears as a first component):
- Monday is the first component in $(\text{Monday}, 20)$.
- Tuesday is the first component in $(\text{Tuesday}, 15)$.
- Wednesday is the first component in $(\text{Wednesday}, 20)$.
- Thursday is the first component in $(\text{Thursday}, 25)$.
- Friday is the first component in $(\text{Friday}, 30)$.
All elements in set $D$ appear as the first component. So, condition 1 is satisfied.
2. Check if each day in $D$ is associated with a unique number of bottles sold (appears as a first component in only one ordered pair):
- Monday appears only in $(\text{Monday}, 20)$. Unique image (20).
- Tuesday appears only in $(\text{Tuesday}, 15)$. Unique image (15).
- Wednesday appears only in $(\text{Wednesday}, 20)$. Unique image (20).
- Thursday appears only in $(\text{Thursday}, 25)$. Unique image (25).
- Friday appears only in $(\text{Friday}, 30)$. Unique image (30).
Each day in set $D$ appears exactly once as the first component. This means each day is associated with one and only one number of bottles sold according to the relation $R$. So, condition 2 is satisfied.
Since both conditions are satisfied, the relation $R$ is a function from $D$ to $N$.
The fact that the same number of bottles (20) was sold on different days (Monday and Wednesday) does not violate the definition of a function. This means that different elements in the domain (Monday, Wednesday) are mapped to the same element in the codomain (20). This is characteristic of a many-to-one function, which is a valid type of function.
Now let's evaluate the given statements:
(A) Yes, because each day is mapped to a unique number of bottles sold.
This statement correctly identifies the relation as a function. The reason given, "each day is mapped to a unique number of bottles sold," is the correct reason why it is a function. 'Unique image' means each element in the domain has only one corresponding element in the codomain, which is true here.
(B) Yes, because each number of bottles sold is mapped to a unique day.
This statement describes a one-to-one function. The relation is not one-to-one because the number 20 is associated with two different days (Monday and Wednesday), and 38 is associated with two different days (Bina and Eshan in the previous problem, if we were considering that data). This statement is incorrect.
(C) No, because the same number of bottles (20) was sold on Monday and Wednesday.
This describes a situation where different elements in the domain map to the same element in the codomain, which is allowed in a function (specifically, a many-to-one function). This is not a reason for a relation to NOT be a function. This statement is incorrect.
(D) No, because the number of bottles sold is a numerical value.
The type of value in the codomain (numerical or otherwise) does not determine whether a relation is a function. This statement is incorrect.
Based on our analysis, statement (A) is the true statement.
The final answer is $\boxed{\text{Yes, because each day is mapped to a unique number of bottles sold}}$.
Therefore, the correct option is (A) Yes, because each day is mapped to a unique number of bottles sold.
Question 34. The domain of the function $f(x) = \frac{1}{\sqrt{1 - x^2}}$ is:
(A) $(-1, 1)$
(B) $[-1, 1]$
(C) $(-\infty, -1) \cup (1, \infty)$
(D) $\mathbb{R} - \{-1, 1\}$
Answer:
Solution:
Given:
The real function $f(x) = \frac{1}{\sqrt{1 - x^2}}$.
To Find:
The domain of the function $f(x)$.
For the function $f(x)$ to be defined as a real number, two conditions must be met:
1. The expression under the square root must be non-negative.
2. The denominator cannot be equal to zero.
From condition 1, the expression inside the square root, $1 - x^2$, must be greater than or equal to zero:
$1 - x^2 \geq 0$
Rearrange the inequality:
$1 \geq x^2$
Or, $x^2 \leq 1$
Taking the square root of both sides gives:
$\sqrt{x^2} \leq \sqrt{1}$
$|x| \leq 1$
The inequality $|x| \leq k$ for $k > 0$ is equivalent to $-k \leq x \leq k$.
In this case, $k=1$. So, the inequality $|x| \leq 1$ is equivalent to:
$-1 \leq x \leq 1$
From condition 2, the denominator $\sqrt{1 - x^2}$ must not be equal to zero.
$\sqrt{1 - x^2} \neq 0$
This implies $1 - x^2 \neq 0$.
We set $1 - x^2 = 0$ to find the values to exclude:
$x^2 = 1$
$x = \pm \sqrt{1}$
$x = \pm 1$
So, we must have $x \neq 1$ and $x \neq -1$.
We need to satisfy both sets of conditions: $-1 \leq x \leq 1$ AND ($x \neq -1$ and $x \neq 1$).
Combining these conditions means that $x$ must be strictly between -1 and 1.
$-1 < x < 1$
In interval notation, the set of all real numbers $x$ such that $-1 < x < 1$ is the open interval $(-1, 1)$.
Thus, the domain of the function $f(x) = \frac{1}{\sqrt{1 - x^2}}$ is $(-1, 1)$.
The final answer is $\boxed{(-1, 1)}$.
Comparing this result with the given options, we find that it matches option (A).
Therefore, the correct option is (A) $(-1, 1)$.
Question 35. Which of the following is the graph of the modulus function $f(x) = |x|$?
(A) A straight line passing through the origin.
(B) A V-shaped graph with vertex at the origin, opening upwards.
(C) A parabola opening upwards.
(D) A horizontal line.
Answer:
Solution:
Given:
The modulus function $f(x) = |x|$.
To Determine:
The shape of the graph of the modulus function.
The modulus function is defined as $f(x) = |x|$, which means:
$f(x) = x$ if $x \geq 0$
$f(x) = -x$ if $x < 0$
Let's consider the graph by plotting some points:
- If $x=0$, $f(0) = |0| = 0$. Point is $(0, 0)$. (Origin)
- If $x=1$, $f(1) = |1| = 1$. Point is $(1, 1)$.
- If $x=2$, $f(2) = |2| = 2$. Point is $(2, 2)$.
- If $x=-1$, $f(-1) = |-1| = 1$. Point is $(-1, 1)$.
- If $x=-2$, $f(-2) = |-2| = 2$. Point is $(-2, 2)$.
For $x \geq 0$, the graph is $y = x$, which is a straight line passing through the origin with slope 1.
For $x < 0$, the graph is $y = -x$, which is a straight line passing through the origin with slope -1.
When plotted, these two parts form a graph that consists of two rays starting from the origin $(0, 0)$ and extending upwards. The part for $x \geq 0$ goes up and to the right (slope 1), and the part for $x < 0$ goes up and to the left (slope -1). This creates a V-shape with its vertex at the origin.
Let's evaluate the options:
(A) A straight line passing through the origin: This only describes one half of the graph (either $y=x$ or $y=-x$, but not the entire graph of $|x|$). So, this is incorrect.
(B) A V-shaped graph with vertex at the origin, opening upwards: This description perfectly matches the shape formed by the two linear pieces of the modulus function graph.
(C) A parabola opening upwards: A parabola opening upwards has the equation $y = ax^2 + bx + c$ with $a > 0$. For example, $y=x^2$. The graph of $|x|$ is made of straight line segments, not a curve like a parabola. So, this is incorrect.
(D) A horizontal line: A horizontal line has the equation $y = c$ (a constant function). The value of $|x|$ changes with $x$, so it is not a horizontal line (unless restricted to a single point). So, this is incorrect.
Therefore, the graph of the modulus function $f(x) = |x|$ is a V-shaped graph with its vertex at the origin, opening upwards.
The final answer is $\boxed{\text{A V-shaped graph with vertex at the origin, opening upwards}}$.
Therefore, the correct option is (B) A V-shaped graph with vertex at the origin, opening upwards.
Question 36. If $f(x) = x$ for $x \in \mathbb{Q}$ and $f(x) = -x$ for $x \notin \mathbb{Q}$, then $f(\sqrt{2})$ is:
(A) $\sqrt{2}$
(B) $-\sqrt{2}$
(C) 2
(D) $-2$
Answer:
Solution:
Given:
The function $f(x)$ is defined as:
$f(x) = \begin{cases} x & , & x \in \mathbb{Q} \\ -x & , & x \notin \mathbb{Q} \end{cases}$
To Find:
The value of $f(\sqrt{2})$.
The definition of the function $f(x)$ depends on whether the input value $x$ is a rational number ($x \in \mathbb{Q}$) or an irrational number ($x \notin \mathbb{Q}$).
We need to evaluate the function at $x = \sqrt{2}$. Therefore, we must determine if $\sqrt{2}$ is a rational number or an irrational number.
A rational number is any number that can be expressed as the quotient or fraction $\frac{p}{q}$ of two integers, a numerator $p$ and a non-zero denominator $q$. An irrational number is a real number that cannot be expressed as a simple fraction.
It is a fundamental result in mathematics that $\sqrt{2}$ cannot be expressed as a fraction of two integers, and thus $\sqrt{2}$ is an irrational number.
This means $\sqrt{2} \notin \mathbb{Q}$.
According to the definition of the function $f(x)$, if $x \notin \mathbb{Q}$ (i.e., $x$ is irrational), then $f(x) = -x$.
Since $\sqrt{2}$ is irrational, we use the second case of the function definition:
$f(x) = -x$ when $x \notin \mathbb{Q}$
Substitute $x = \sqrt{2}$ into this part of the definition:
$f(\sqrt{2}) = -(\sqrt{2})$
$f(\sqrt{2}) = -\sqrt{2}$
Thus, the value of $f(\sqrt{2})$ is $-\sqrt{2}$.
The final answer is $\boxed{-\sqrt{2}}$.
Comparing this result with the given options, we find that it matches option (B).
Therefore, the correct option is (B) $-\sqrt{2}$.
Question 37. Let $A = \{1, 2\}$ and $B = \{a, b, c\}$. Which of the following is a multiple correct answer question?
Which of the following are relations from $A$ to $B$?
(A) $\{(1, a), (2, b)\}$
(B) $\{(a, 1), (b, 2)\}$
(C) $\{(1, c), (2, a), (1, b)\}$
(D) $\emptyset$
Answer:
Solution:
Given:
Set $A = \{1, 2\}$.
Set $B = \{a, b, c\}$.
To Determine:
Which of the given options are relations from set $A$ to set $B$. (Multiple correct answers are possible).
A relation from a set $A$ to a set $B$ is defined as any subset of the Cartesian product $A \times B$.
First, let's find the Cartesian product $A \times B$. This is the set of all ordered pairs $(x, y)$ where the first element $x$ is from set $A$, and the second element $y$ is from set $B$.
$A \times B = \{(x, y) \mid x \in A \text{ and } y \in B\}$
Using the given sets $A = \{1, 2\}$ and $B = \{a, b, c\}$:
$A \times B = \{(1, a), (1, b), (1, c), (2, a), (2, b), (2, c)\}$
Now, we check each given option to see if it is a subset of $A \times B$. A set is a subset of another if all its elements are contained within the other set.
(A) $\{(1, a), (2, b)\}$
Let's check each ordered pair in this set:
- $(1, a)$: $1 \in A$ and $a \in B$. So, $(1, a) \in A \times B$.
- $(2, b)$: $2 \in A$ and $b \in B$. So, $(2, b) \in A \times B$.
Since all the ordered pairs in option (A) are elements of $A \times B$, the set in option (A) is a subset of $A \times B$.
Therefore, option (A) is a relation from $A$ to $B$.
(B) $\{(a, 1), (b, 2)\}$
Let's check the ordered pairs:
- $(a, 1)$: $a \in B$ but $a \notin A$. For the pair to be in $A \times B$, the first element must be from $A$. So, $(a, 1) \notin A \times B$.
Since not all elements are in $A \times B$, option (B) is not a relation from $A$ to $B$. (It is a relation from $B$ to $A$).
(C) $\{(1, c), (2, a), (1, b)\}$
Let's check each ordered pair in this set:
- $(1, c)$: $1 \in A$ and $c \in B$. So, $(1, c) \in A \times B$.
- $(2, a)$: $2 \in A$ and $a \in B$. So, $(2, a) \in A \times B$.
- $(1, b)$: $1 \in A$ and $b \in B$. So, $(1, b) \in A \times B$.
Since all the ordered pairs in option (C) are elements of $A \times B$, the set in option (C) is a subset of $A \times B$.
Therefore, option (C) is a relation from $A$ to $B$. (Note that this relation is not a function because the element 1 from set A is related to both c and b in set B).
(D) $\emptyset$
The empty set ($\emptyset$) is considered a subset of every set, including $A \times B$.
Therefore, the empty set is always a relation from $A$ to $B$. This relation is called the empty relation.
Therefore, option (D) is a relation from $A$ to $B$.
Based on the analysis, options (A), (C), and (D) are all subsets of $A \times B$, and thus are relations from $A$ to $B$.
The final answer is $\boxed{\text{(A), (C), (D)}}$.
Therefore, the correct options are (A) $\{(1, a), (2, b)\}$, (C) $\{(1, c), (2, a), (1, b)\}$, and (D) $\emptyset$.
Question 38. The range of the function $f(x) = x^2 + 2$ is:
(A) $[0, \infty)$
(B) $[2, \infty)$
(C) $(-\infty, \infty)$
(D) $(-\infty, 2]$
Answer:
Solution:
Given:
The function $f(x) = x^2 + 2$.
To Find:
The range of the function $f(x)$.
The range of a function is the set of all possible output values ($f(x)$ or $y$) that the function can produce for inputs in its domain.
The domain of the function $f(x) = x^2 + 2$ is all real numbers, $\mathbb{R}$, since it is a polynomial function.
Let's analyze the expression for $f(x)$. The term $x^2$ is the square of a real number. For any real number $x$, the square $x^2$ is always non-negative (greater than or equal to 0).
$x^2 \geq 0$ for all $x \in \mathbb{R}$
Now, consider the entire expression $f(x) = x^2 + 2$. Since $x^2 \geq 0$, adding 2 to both sides of the inequality gives us the possible values for $f(x)$:
$x^2 + 2 \geq 0 + 2$
$f(x) \geq 2$
This inequality tells us that the value of the function $f(x)$ is always greater than or equal to 2.
The minimum value of $f(x)$ is 2, which occurs when $x^2 = 0$, i.e., when $x=0$.
$f(0) = 0^2 + 2 = 2$.
As $|x|$ increases (either positive or negative), $x^2$ increases, and consequently $f(x) = x^2 + 2$ increases. There is no upper bound for the values of $f(x)$.
Therefore, the set of all possible output values is the set of all real numbers greater than or equal to 2.
In interval notation, the range is $[2, \infty)$.
Comparing this result with the given options, we find that it matches option (B).
The final answer is $\boxed{[2, \infty)}$.
Therefore, the correct option is (B) $[2, \infty)$.
Question 39. If the graph of a relation on $\mathbb{R}$ passes the vertical line test, the relation is:
(A) A constant function
(B) An identity function
(C) A relation, but not a function
(D) A function
Answer:
Solution:
Given:
The graph of a relation on $\mathbb{R}$ passes the vertical line test.
To Determine:
What conclusion can be drawn about the relation.
The Vertical Line Test is a graphical method used to determine whether a given graph represents a function. The test states that:
A graph in the Cartesian plane represents a function if and only if no vertical line intersects the graph at more than one point.
If a graph passes the vertical line test, it means that for every value on the x-axis (which represents the input or the first element of the ordered pair), there is at most one corresponding value on the y-axis (which represents the output or the second element of the ordered pair).
Let's relate this to the definition of a function:
A relation is a function if every element in the domain is associated with a unique element in the codomain. In terms of a graph, the domain is represented by the x-values, and the output (image) is represented by the y-values.
Passing the vertical line test means that for each valid x-value, there is only one corresponding y-value. This is exactly the condition required for a relation to be a function.
Let's evaluate the options:
(A) A constant function: A constant function ($f(x) = c$) is a specific type of function. Its graph is a horizontal line. A horizontal line passes the vertical line test. However, passing the vertical line test only guarantees that it is *a* function, not necessarily a constant function (it could be $f(x) = x$, $f(x) = x^2$, etc.). So, this is not the most general conclusion.
(B) An identity function: An identity function ($f(x) = x$) is also a specific type of function. Its graph is a straight line through the origin with slope 1. This graph also passes the vertical line test. However, passing the vertical line test does not mean it *must* be the identity function. So, this is not the most general conclusion.
(C) A relation, but not a function: Since the graph passes the vertical line test, it satisfies the definition of a function. A function is a special type of relation. Therefore, it is a function, which implies it is also a relation. The statement "but not a function" makes this option incorrect.
(D) A function: As explained above, the vertical line test is the graphical criterion for determining if a relation is a function. If the graph of a relation passes the test, the relation is a function.
The most accurate and direct conclusion from a graph passing the vertical line test is that the relation represented by the graph is a function.
The final answer is $\boxed{\text{A function}}$.
Therefore, the correct option is (D) A function.
Question 40. Let $f = \{(1, 2), (2, 3), (3, 4)\}$ be a function from $\{1, 2, 3\}$ to $\{2, 3, 4, 5\}$. Which element in the domain has the image 3?
(A) 1
(B) 2
(C) 3
(D) 4
Answer:
Solution:
Given:
The function $f = \{(1, 2), (2, 3), (3, 4)\}$.
The domain of the function is $\{1, 2, 3\}$.
The codomain of the function is $\{2, 3, 4, 5\}$.
To Find:
The element in the domain that has the image 3.
A function, when represented as a set of ordered pairs $(x, y)$, relates each element $x$ from the domain to its corresponding image $y$ in the codomain. In the ordered pair $(x, y)$, $x$ is the input (an element from the domain) and $y$ is the output or image of $x$ (an element from the codomain or range).
The given function is $f = \{(1, 2), (2, 3), (3, 4)\}$. Let's examine each ordered pair:
- The ordered pair $(1, 2)$ means that the element 1 from the domain has the image 2. So, $f(1) = 2$.
- The ordered pair $(2, 3)$ means that the element 2 from the domain has the image 3. So, $f(2) = 3$.
- The ordered pair $(3, 4)$ means that the element 3 from the domain has the image 4. So, $f(3) = 4$.
We are looking for the element in the domain that has the image 3. From the ordered pairs, we see that the pair $(2, 3)$ has 3 as the image (the second component).
The corresponding element in the domain (the first component) is 2.
Therefore, the element in the domain $\{1, 2, 3\}$ that has the image 3 is 2.
The final answer is $\boxed{2}$.
Comparing this result with the given options, we find that it matches option (B).
Therefore, the correct option is (B) 2.
Question 41. Assertion (A): The domain of the function $f(x) = \frac{1}{x}$ is $\mathbb{R} - \{0\}$.
Reason (R): Division by zero is undefined.
(A) Both A and R are true and R is the correct explanation of A.
(B) Both A and R are true but R is not the correct explanation of A.
(C) A is true but R is false.
(D) A is false but R is true.
Answer:
Solution:
Given:
Assertion (A): The domain of the function $f(x) = \frac{1}{x}$ is $\mathbb{R} - \{0\}$.
Reason (R): Division by zero is undefined.
Let's evaluate the truthfulness of the Assertion and the Reason, and then check if the Reason correctly explains the Assertion.
Checking Assertion (A):
The function is $f(x) = \frac{1}{x}$. For a real function to be defined, any operation must result in a real number. In this case, the operation is division.
Division is defined for all real numbers except when the divisor (denominator) is zero.
The denominator of $f(x) = \frac{1}{x}$ is $x$. The function is undefined when $x = 0$.
For all other real values of $x$ (i.e., $x \neq 0$), the value of $\frac{1}{x}$ is a well-defined real number.
Therefore, the domain of $f(x) = \frac{1}{x}$ is the set of all real numbers excluding 0, which is $\mathbb{R} - \{0\}$.
Assertion (A) is True.
Checking Reason (R):
Reason (R) states: Division by zero is undefined.
This is a fundamental rule in arithmetic and algebra. In the set of real numbers (and complex numbers), division by zero is an undefined operation.
Reason (R) is True.
Relation between A and R:
Assertion (A) claims the domain of $f(x) = \frac{1}{x}$ is $\mathbb{R} - \{0\}$. Reason (R) states that division by zero is undefined.
The reason why the domain of $f(x) = \frac{1}{x}$ excludes $x=0$ is precisely because division by zero is undefined. The restriction on the domain is a direct consequence of the fact stated in Reason (R).
Therefore, Reason (R) provides the correct explanation for Assertion (A).
Both Assertion (A) and Reason (R) are true, and Reason (R) is the correct explanation for Assertion (A).
The final answer is $\boxed{\text{Both A and R are true and R is the correct explanation of A}}$.
Therefore, the correct option is (A) Both A and R are true and R is the correct explanation of A.
Question 42. The range of the function $f(x) = \frac{|x|}{x}$ is:
(A) $\{-1, 1\}$
(B) $\mathbb{R} - \{0\}$
(C) $\{-1, 0, 1\}$
(D) $(-1, 1)$
Answer:
Solution:
Given:
The function $f(x) = \frac{|x|}{x}$.
To Find:
The range of the function $f(x)$.
The domain of the function $f(x) = \frac{|x|}{x}$ is all real numbers except where the denominator is zero. The denominator is $x$, so the function is defined for all $x \in \mathbb{R}$ such that $x \neq 0$. The domain is $\mathbb{R} - \{0\}$.
Now let's analyze the possible output values for $f(x)$ for $x$ in the domain.
We consider two cases for $x \neq 0$ based on the definition of the absolute value function $|x|$:
Case 1: $x > 0$
If $x > 0$, then $|x| = x$.
So, $f(x) = \frac{|x|}{x} = \frac{x}{x}$.
For any $x > 0$, $\frac{x}{x} = 1$.
Thus, when $x > 0$, the output of the function is always 1.
Case 2: $x < 0$
If $x < 0$, then $|x| = -x$.
So, $f(x) = \frac{|x|}{x} = \frac{-x}{x}$.
For any $x < 0$, $\frac{-x}{x} = -1$.
Thus, when $x < 0$, the output of the function is always -1.
The function is not defined at $x=0$. Therefore, the only possible output values for $f(x)$ are 1 (when $x>0$) and -1 (when $x<0$).
The set of all possible output values is $\{-1, 1\}$.
Note that this function is also known as the Signum function, $\text{sgn}(x)$, excluding the value at $x=0$. The standard Signum function includes $f(0)=0$ in its definition.
The range of the function $f(x) = \frac{|x|}{x}$ is the set $\{-1, 1\}$.
The final answer is $\boxed{\{-1, 1\}}$.
Comparing this result with the given options, we find that it matches option (A).
Therefore, the correct option is (A) $\{-1, 1\}$.
Question 43. If $f(x) = x^3$, find the value of $\frac{f(2) - f(1)}{2 - 1}$.
(A) 1
(B) 7
(C) 9
(D) 6
Answer:
Solution:
Given:
The function $f(x) = x^3$.
To Find:
The value of the expression $\frac{f(2) - f(1)}{2 - 1}$.
First, we need to evaluate the function $f(x)$ at $x=2$ and $x=1$.
For $f(2)$, substitute $x=2$ into $f(x) = x^3$:
$f(2) = (2)^3$
$f(2) = 2 \times 2 \times 2$
$f(2) = 8$
For $f(1)$, substitute $x=1$ into $f(x) = x^3$:
$f(1) = (1)^3$
$f(1) = 1 \times 1 \times 1$
$f(1) = 1$
Now, substitute the values of $f(2)$ and $f(1)$ into the given expression $\frac{f(2) - f(1)}{2 - 1}$.
$\frac{f(2) - f(1)}{2 - 1} = \frac{8 - 1}{2 - 1}$
Perform the subtraction in the numerator and the denominator:
Numerator: $8 - 1 = 7$
Denominator: $2 - 1 = 1$
Substitute these values back into the fraction:
$\frac{7}{1} = 7$
Thus, the value of $\frac{f(2) - f(1)}{2 - 1}$ is 7.
The final answer is $\boxed{7}$.
Comparing this result with the given options, we find that it matches option (B).
Therefore, the correct option is (B) 7.
Question 44. Which of the following is a rational function?
(A) $f(x) = x^2 + \sqrt{x}$
(B) $f(x) = \frac{x^2+1}{x-3}$
(C) $f(x) = |x| + 5$
(D) $f(x) = 2^x$
Answer:
Solution:
Given:
Four different functions.
To Determine:
Which of the given functions is a rational function.
A rational function is a function that can be written as the ratio of two polynomial functions, $f(x) = \frac{P(x)}{Q(x)}$, where $P(x)$ and $Q(x)$ are polynomials and $Q(x)$ is not the zero polynomial.
Let's examine each option:
(A) $f(x) = x^2 + \sqrt{x}$
This function involves the term $\sqrt{x}$, which is $x^{1/2}$. A polynomial function can only have non-negative integer exponents for the variable. Since the exponent is $\frac{1}{2}$, $\sqrt{x}$ is not a polynomial term (over the domain where $\sqrt{x}$ is defined for real numbers). Therefore, $x^2 + \sqrt{x}$ is not a ratio of polynomials.
This is not a rational function.
(B) $f(x) = \frac{x^2+1}{x-3}$
The numerator is $P(x) = x^2+1$, which is a polynomial.
The denominator is $Q(x) = x-3$, which is a polynomial and is not the zero polynomial.
This function is expressed as the ratio of two polynomials.
Therefore, this is a rational function.
(C) $f(x) = |x| + 5$
This function involves the absolute value function $|x|$. The absolute value function is defined piecewise and is not a polynomial function for all real numbers. For example, its graph is V-shaped, while a polynomial graph is a smooth curve. Therefore, $|x| + 5$ is not a ratio of polynomials.
This is not a rational function.
(D) $f(x) = 2^x$
This function is an exponential function where the variable $x$ is in the exponent. This is not a polynomial function, nor can it generally be expressed as a ratio of polynomials.
This is not a rational function.
Based on the definition, only option (B) fits the criteria of a rational function.
The final answer is $\boxed{f(x) = \frac{x^2+1}{x-3}}$.
Therefore, the correct option is (B) $f(x) = \frac{x^2+1}{x-3}$.
Question 45. If $f(x) = ax+b$ is a linear function, and $f(1) = 5$, $f(2) = 8$, find the values of $a$ and $b$.
(A) $a=3, b=2$
(B) $a=2, b=3$
(C) $a=3, b=5$
(D) $a=5, b=3$
Answer:
Solution:
Given:
The linear function $f(x) = ax+b$.
The function satisfies $f(1) = 5$ and $f(2) = 8$.
To Find:
The values of the constants $a$ and $b$.
We are given two conditions based on the function's value at specific points. We can substitute these points into the function's equation to form a system of linear equations in terms of $a$ and $b$.
Using the condition $f(1) = 5$:
Substitute $x=1$ into $f(x) = ax+b$:
$f(1) = a(1) + b$
Since $f(1) = 5$, we have:
$a + b = 5$
$a + b = 5$
... (i)
Using the condition $f(2) = 8$:
Substitute $x=2$ into $f(x) = ax+b$:
$f(2) = a(2) + b$
Since $f(2) = 8$, we have:
$2a + b = 8$
$2a + b = 8$
... (ii)
Now we have a system of two linear equations with two variables $a$ and $b$:
1) $a + b = 5$
2) $2a + b = 8$
We can solve this system using the elimination method. Subtract equation (i) from equation (ii):
$(2a + b) - (a + b) = 8 - 5$
$2a + b - a - b = 3$
$(2a - a) + (b - b) = 3$
$a + 0 = 3$
$a = 3$
Now that we have the value of $a$, we can substitute $a=3$ into either equation (i) or equation (ii) to find the value of $b$. Using equation (i):
$a + b = 5$
$3 + b = 5$
Solving for $b$:
$b = 5 - 3$
$b = 2$
So, the values of $a$ and $b$ are $a=3$ and $b=2$.
Let's verify these values using equation (ii):
$2a + b = 2(3) + 2 = 6 + 2 = 8$. This matches the given condition $f(2)=8$. The values are correct.
Thus, the values of $a$ and $b$ are 3 and 2, respectively.
The final answer is $\boxed{a=3, b=2}$.
Comparing this result with the given options, we find that it matches option (A).
Therefore, the correct option is (A) $a=3, b=2$.
Question 46. Consider the relation $R = \{(1, 2), (1, 3), (2, 4), (3, 5)\}$ from $A = \{1, 2, 3\}$ to $B = \{2, 3, 4, 5\}$.
Which of the following statements is/are correct?
(A) $R$ is a function.
(B) The domain of $R$ is $\{1, 2, 3\}$.
(C) The range of $R$ is $\{2, 3, 4, 5\}$.
(D) The element 1 in the domain has unique image.
Answer:
Solution:
Given:
Set $A = \{1, 2, 3\}$.
Set $B = \{2, 3, 4, 5\}$.
Relation $R = \{(1, 2), (1, 3), (2, 4), (3, 5)\}$ from $A$ to $B$.
To Determine:
Which of the given statements about the relation $R$ are correct.
Let's analyze the relation $R$ and evaluate each statement.
$R = \{(1, 2), (1, 3), (2, 4), (3, 5)\}$.
(A) $R$ is a function.
A relation $R$ from set $A$ to set $B$ is a function if and only if every element in set $A$ has one and only one image in set $B$ under the relation $R$. This means that for every element $x \in A$, there must be exactly one ordered pair $(x, y)$ in $R$.
Let's check the elements of set $A = \{1, 2, 3\}$:
- For the element 1: The ordered pairs in $R$ starting with 1 are $(1, 2)$ and $(1, 3)$. The element 1 is related to both 2 and 3 in set $B$. This means 1 has two images (2 and 3).
Since the element 1 from the domain is related to more than one element in the codomain, the relation $R$ is not a function.
Statement (A) is False.
(B) The domain of $R$ is $\{1, 2, 3\}$.
The domain of a relation is the set of all first components of the ordered pairs in the relation.
The ordered pairs in $R$ are $(1, 2), (1, 3), (2, 4), (3, 5)$.
The first components are 1, 1, 2, and 3.
The set of unique first components is $\{1, 2, 3\}$.
This set is equal to set $A$.
Statement (B) is True.
(C) The range of $R$ is $\{2, 3, 4, 5\}$.
The range of a relation is the set of all second components of the ordered pairs in the relation.
The ordered pairs in $R$ are $(1, 2), (1, 3), (2, 4), (3, 5)$.
The second components are 2, 3, 4, and 5.
The set of unique second components is $\{2, 3, 4, 5\}$.
This set is equal to set $B$.
Statement (C) is True.
(D) The element 1 in the domain has unique image.
The image of an element $x$ in the domain under a relation $R$ is the set of all elements $y$ in the codomain such that $(x, y) \in R$. A unique image means this set contains exactly one element.
For the element 1 in the domain, the ordered pairs starting with 1 are $(1, 2)$ and $(1, 3)$.
The set of images of 1 is $\{2, 3\}$.
Since the set $\{2, 3\}$ contains more than one element, the element 1 does not have a unique image.
Statement (D) is False.
Based on the analysis, statements (B) and (C) are correct.
The final answer is $\boxed{\text{(B) and (C)}}$.
Therefore, the correct options are (B) The domain of $R$ is $\{1, 2, 3\}$ and (C) The range of $R$ is $\{2, 3, 4, 5\}$.
Question 47. If $A = \{x \in \mathbb{N} : x \leq 3\}$ and $B = \{y \in \mathbb{Z} : -1 \leq y \leq 1\}$. How many elements are there in $A \times B$?
(A) 6
(B) 9
(C) 12
(D) 15
Answer:
Solution:
Given:
Set $A = \{x \in \mathbb{N} : x \leq 3\}$.
Set $B = \{y \in \mathbb{Z} : -1 \leq y \leq 1\}$.
To Find:
The number of elements in the Cartesian product $A \times B$, denoted as $n(A \times B)$.
First, let's determine the elements of set $A$. The set $A$ consists of natural numbers ($x \in \mathbb{N}$) such that $x$ is less than or equal to 3.
Assuming natural numbers start from 1, $\mathbb{N} = \{1, 2, 3, 4, ...\}$.
So, the elements of $A$ that satisfy $x \leq 3$ are 1, 2, and 3.
$A = \{1, 2, 3\}$
The number of elements in set $A$ is $n(A) = 3$.
Next, let's determine the elements of set $B$. The set $B$ consists of integers ($y \in \mathbb{Z}$) such that $y$ is greater than or equal to -1 and less than or equal to 1.
The integers satisfying $-1 \leq y \leq 1$ are -1, 0, and 1.
$B = \{-1, 0, 1\}$
The number of elements in set $B$ is $n(B) = 3$.
The number of elements in the Cartesian product $A \times B$ is the product of the number of elements in set $A$ and the number of elements in set $B$.
$n(A \times B) = n(A) \times n(B)$
Substitute the calculated values of $n(A)$ and $n(B)$:
$n(A \times B) = 3 \times 3$
$n(A \times B) = 9$
Thus, there are 9 elements in $A \times B$. The elements are the ordered pairs $(x, y)$ where $x \in A$ and $y \in B$.
$A \times B = \{(1, -1), (1, 0), (1, 1), (2, -1), (2, 0), (2, 1), (3, -1), (3, 0), (3, 1)\}$
Comparing this result with the given options, we find that it matches option (B).
The final answer is $\boxed{9}$.
Therefore, the correct option is (B) 9.
Question 48. The domain of the function $f(x) = \frac{1}{x^2+1}$ is:
(A) $\mathbb{R} - \{-1\}$
(B) $\mathbb{R} - \{1\}$
(C) $\mathbb{R}$
(D) $\emptyset$
Answer:
Solution:
Given:
The real function $f(x) = \frac{1}{x^2+1}$.
To Find:
The domain of the function $f(x)$.
For the function $f(x)$ to be defined as a real number, the denominator cannot be equal to zero, because division by zero is undefined.
So, we must have:
$x^2 + 1 \neq 0$
Let's try to find the values of $x$ for which the denominator would be zero:
$x^2 + 1 = 0$
Subtract 1 from both sides:
$x^2 = -1$
In the set of real numbers ($\mathbb{R}$), the square of any real number ($x^2$) is always non-negative ($x^2 \geq 0$). Therefore, there is no real number $x$ whose square is equal to -1.
The equation $x^2 = -1$ has no real solutions.
This means that the denominator $x^2 + 1$ is never equal to zero for any real value of $x$. Since the denominator is never zero, the function $f(x) = \frac{1}{x^2+1}$ is defined for all real numbers.
Therefore, the domain of the function $f(x) = \frac{1}{x^2+1}$ is the set of all real numbers.
Domain of $f = \mathbb{R}$ or $(-\infty, \infty)$.
Comparing this result with the given options, we find that it matches option (C).
The final answer is $\boxed{\mathbb{R}}$.
Therefore, the correct option is (C) $\mathbb{R}$.
Question 49. If $f(x) = \text{cosec}(x)$, the domain of $f(x)$ is:
(A) $\mathbb{R}$
(B) $\mathbb{R} - \{n\pi : n \in \mathbb{Z}\}$
(C) $\mathbb{R} - \{(2n+1)\frac{\pi}{2} : n \in \mathbb{Z}\}$
(D) $(-\infty, -1] \cup [1, \infty)$
Answer:
Solution:
Given:
The function $f(x) = \text{cosec}(x)$.
To Find:
The domain of the function $f(x)$.
The cosecant function is defined as the reciprocal of the sine function:
$f(x) = \text{cosec}(x) = \frac{1}{\sin(x)}$
For $f(x)$ to be a real number, the denominator cannot be equal to zero. Therefore, we must find the values of $x$ for which $\sin(x) = 0$ and exclude them from the domain of real numbers $\mathbb{R}$.
The sine function, $\sin(x)$, is equal to zero at integer multiples of $\pi$. That is, the values of $x$ for which $\sin(x) = 0$ are:
$x = n\pi$, where $n$ is any integer ($n \in \mathbb{Z}$).
These are the values of $x$ where the function $\text{cosec}(x)$ is undefined.
The domain of $f(x) = \text{cosec}(x)$ is the set of all real numbers excluding these values.
Domain of $f = \mathbb{R} - \{x \in \mathbb{R} \mid x = n\pi \text{ for some } n \in \mathbb{Z}\}$
Domain of $f = \mathbb{R} - \{n\pi : n \in \mathbb{Z}\}$
Comparing this result with the given options, we find that it matches option (B).
The final answer is $\boxed{\mathbb{R} - \{n\pi : n \in \mathbb{Z}\}}$.
Therefore, the correct option is (B) $\mathbb{R} - \{n\pi : n \in \mathbb{Z}\}$.
Question 50. Let $f(x) = 2x$ and $g(x) = x+5$. Find $(f \circ g)(x)$.
(A) $2x+5$
(B) $2x+10$
(C) $2(x+5)$
(D) $x+5/2$
Answer:
Solution:
Given:
The function $f(x) = 2x$.
The function $g(x) = x+5$.
To Find:
The composite function $(f \circ g)(x)$.
The composition of two functions $f$ and $g$, denoted by $(f \circ g)$, is defined as:
$(f \circ g)(x) = f(g(x))$
This means we first apply the function $g$ to $x$, and then apply the function $f$ to the result of $g(x)$.
We are given $g(x) = x+5$.
Now, substitute $g(x)$ into the expression for $f(x)$. The expression for $f(x)$ is $2x$. We replace the variable $x$ in $f(x)$ with the entire expression for $g(x)$.
$f(g(x)) = f(x+5)$
Since $f(x) = 2x$, replacing $x$ with $(x+5)$ gives:
$f(x+5) = 2(x+5)$
Distribute the 2:
$f(x+5) = 2x + 10$
Thus, the composite function $(f \circ g)(x)$ is $2x + 10$.
$(f \circ g)(x) = 2x + 10$
Comparing this result with the given options, we find that it matches option (B).
While option (C) $2(x+5)$ is an intermediate step before simplification, the question asks to "find $(f \circ g)(x)$", and the simplified form is usually expected unless otherwise specified. Option (B) provides the simplified form.
The final answer is $\boxed{2x+10}$.
Therefore, the correct option is (B) $2x+10$.
Question 51. The range of the greatest integer function $f(x) = \lfloor x \rfloor$ is:
(A) $\mathbb{R}$
(B) $\mathbb{Z}$
(C) $[0, \infty)$
(D) $(-\infty, \infty)$
Answer:
Solution:
Given:
The greatest integer function $f(x) = \lfloor x \rfloor$.
To Find:
The range of the function $f(x)$.
The range of a function is the set of all possible output values that the function can produce for inputs in its domain.
The greatest integer function $\lfloor x \rfloor$ gives the largest integer less than or equal to $x$. The domain of this function is all real numbers, $\mathbb{R}$.
Let's consider some examples of the output values of $f(x)$ for different real number inputs $x$:
- If $x = 0.5$, $f(0.5) = \lfloor 0.5 \rfloor = 0$.
- If $x = 1.9$, $f(1.9) = \lfloor 1.9 \rfloor = 1$.
- If $x = 3$, $f(3) = \lfloor 3 \rfloor = 3$.
- If $x = -0.7$, $f(-0.7) = \lfloor -0.7 \rfloor = -1$.
- If $x = -2.4$, $f(-2.4) = \lfloor -2.4 \rfloor = -3$.
- If $x = \pi \approx 3.14$, $f(\pi) = \lfloor \pi \rfloor = 3$.
- If $x = -\sqrt{3} \approx -1.73$, $f(-\sqrt{3}) = \lfloor -\sqrt{3} \rfloor = -2$.
From these examples, we can observe that the output value of the greatest integer function is always an integer.
Furthermore, for any integer $n$, we can choose an input value of $x$ such that $\lfloor x \rfloor = n$. For example, if we want the output to be $n$, we can choose any real number $x$ such that $n \leq x < n+1$. For instance, choosing $x=n$ gives $\lfloor n \rfloor = n$.
This means that any integer can be an output value of the function $f(x) = \lfloor x \rfloor$.
Since the output is always an integer, and any integer can be an output, the set of all possible output values (the range) is the set of all integers.
The set of all integers is denoted by $\mathbb{Z}$.
Range of $f = \mathbb{Z}$
Comparing this result with the given options, we find that it matches option (B).
The final answer is $\boxed{\mathbb{Z}}$.
Therefore, the correct option is (B) $\mathbb{Z}$.
Question 52. If $A = \{1, 2, 3\}$ and $B = \{a, b\}$, the number of functions from $A$ to $B$ is:
(A) $2^3$
(B) $3^2$
(C) $2 \times 3$
(D) $3 \times 2$
Answer:
Solution:
Given:
Set $A = \{1, 2, 3\}$, so the number of elements in A is $n(A) = 3$.
Set $B = \{a, b\}$, so the number of elements in B is $n(B) = 2$.
To Find:
The number of functions from set $A$ to set $B$.
A function from set $A$ to set $B$ requires that each element in $A$ is mapped to exactly one element in $B$.
Let $m = n(A)$ be the number of elements in set $A$, and $n = n(B)$ be the number of elements in set $B$.
For each element in set $A$, there are $n(B)$ choices for its image in set $B$.
Since there are $n(A)$ elements in set $A$, and the choice of the image for each element is independent, the total number of ways to map the elements of $A$ to the elements of $B$ is the product of the number of choices for each element in $A$.
Number of functions from $A$ to $B = \underbrace{n(B) \times n(B) \times \cdots \times n(B)}_{n(A) \text{ times}}$
Number of functions from $A$ to $B = (n(B))^{n(A)}$
Number of functions from $A$ to $B = n^m$
Substitute the given values $m = n(A) = 3$ and $n = n(B) = 2$:
Number of functions from $A$ to $B = 2^3$
Calculate the value:
$2^3 = 2 \times 2 \times 2 = 8$
Comparing this result with the given options:
(A) $2^3$
(B) $3^2$
(C) $2 \times 3$
(D) $3 \times 2$
The calculated number of functions, $2^3$, matches option (A).
The final answer is $\boxed{2^3}$.
Therefore, the correct option is (A) $2^3$.
Question 53. Which of the following is NOT a real function?
(A) $f: \mathbb{N} \to \mathbb{R}$ defined by $f(x) = x^2$
(B) $f: \mathbb{R} \to \mathbb{R}$ defined by $f(x) = \sqrt{x}$
(C) $f: \mathbb{C} \to \mathbb{R}$ defined by $f(z) = |z|$
(D) $f: \mathbb{R} \to \mathbb{Z}$ defined by $f(x) = \lfloor x \rfloor$
Answer:
Solution:
Given:
Four different functions with their domains and codomains specified.
To Determine:
Which of the given functions is NOT a real function.
A real function is typically defined as a function whose domain is a subset of the set of real numbers ($\mathbb{R}$). Some definitions also require the codomain to be a subset of $\mathbb{R}$ (making it a real-valued function of a real variable). Let's use the standard definition that the domain must be a subset of $\mathbb{R}$.
Let's examine each option based on its domain:
(A) $f: \mathbb{N} \to \mathbb{R}$ defined by $f(x) = x^2$
The domain of this function is $\mathbb{N}$, the set of natural numbers. The set of natural numbers $\mathbb{N}$ is a subset of the set of real numbers $\mathbb{R}$ ($\mathbb{N} \subset \mathbb{R}$).
The codomain is $\mathbb{R}$, which is a subset of $\mathbb{R}$.
This function satisfies the condition for being a real function.
(B) $f: \mathbb{R} \to \mathbb{R}$ defined by $f(x) = \sqrt{x}$
The domain of this function is $\mathbb{R}$, the set of real numbers. The set of real numbers $\mathbb{R}$ is a subset of $\mathbb{R}$ ($\mathbb{R} \subseteq \mathbb{R}$).
The codomain is $\mathbb{R}$, which is a subset of $\mathbb{R}$.
Based solely on the domain being a subset of $\mathbb{R}$, this fits the definition of a real function. However, it's worth noting that the function rule $\sqrt{x}$ does not map all elements of the stated domain $\mathbb{R}$ to the stated codomain $\mathbb{R}$ (e.g., $\sqrt{-1} = i \notin \mathbb{R}$). But the question asks about the function *as defined* by its stated domain and codomain. The domain itself is a subset of $\mathbb{R}$.
(C) $f: \mathbb{C} \to \mathbb{R}$ defined by $f(z) = |z|$
The domain of this function is $\mathbb{C}$, the set of complex numbers. The set of complex numbers $\mathbb{C}$ is not a subset of the set of real numbers $\mathbb{R}$.
The codomain is $\mathbb{R}$, which is a subset of $\mathbb{R}$.
Since the domain of this function is not a subset of $\mathbb{R}$, it does not satisfy the standard definition of a real function.
(D) $f: \mathbb{R} \to \mathbb{Z}$ defined by $f(x) = \lfloor x \rfloor$
The domain of this function is $\mathbb{R}$, the set of real numbers. $\mathbb{R}$ is a subset of $\mathbb{R}$.
The codomain is $\mathbb{Z}$, the set of integers. The set of integers $\mathbb{Z}$ is a subset of the set of real numbers $\mathbb{R}$ ($\mathbb{Z} \subset \mathbb{R}$).
This function satisfies the condition for being a real function (both domain and codomain are subsets of $\mathbb{R}$).
Based on the standard definition that a real function must have a domain which is a subset of $\mathbb{R}$, option (C) is the only function whose domain is not a subset of $\mathbb{R}$.
The final answer is $\boxed{f: \mathbb{C} \to \mathbb{R} \text{ defined by } f(z) = |z|}$.
Therefore, the correct option is (C) $f: \mathbb{C} \to \mathbb{R}$ defined by $f(z) = |z|$.
Question 54. Consider the relation $R$ on the set of all lines in a plane defined by $l_1 R l_2$ if $l_1$ is perpendicular to $l_2$. The relation $R$ is:
(A) Reflexive
(B) Symmetric
(C) Transitive
(D) An equivalence relation
Answer:
Solution:
Given:
Set $L$, the set of all lines in a plane.
Relation $R$ on $L$ defined by $l_1 R l_2$ if $l_1$ is perpendicular to $l_2$. We can write this as $l_1 R l_2 \iff l_1 \perp l_2$.
To Determine:
Whether the relation $R$ is Reflexive, Symmetric, Transitive, or an Equivalence relation.
Let's check the properties of the relation $R$:
1. Reflexivity:
A relation $R$ on a set $L$ is reflexive if $(l, l) \in R$ for every $l \in L$.
In this case, we need to check if every line $l$ is perpendicular to itself ($l \perp l$).
A line is parallel to itself, not perpendicular. The angle a line makes with itself is $0^\circ$, not $90^\circ$.
Thus, $l \not\perp l$ for any line $l$.
Therefore, the relation $R$ is not reflexive.
2. Symmetry:
A relation $R$ on a set $L$ is symmetric if whenever $(l_1, l_2) \in R$, then $(l_2, l_1) \in R$.
In this case, we need to check if whenever $l_1 \perp l_2$, it implies $l_2 \perp l_1$.
If line $l_1$ is perpendicular to line $l_2$, it means the angle between them is $90^\circ$. The statement "$l_2$ is perpendicular to $l_1$" also means the angle between them is $90^\circ$. These two statements are equivalent.
Thus, $l_1 \perp l_2 \implies l_2 \perp l_1$.
Therefore, the relation $R$ is symmetric.
3. Transitivity:
A relation $R$ on a set $L$ is transitive if whenever $(l_1, l_2) \in R$ and $(l_2, l_3) \in R$, then $(l_1, l_3) \in R$.
In this case, we need to check if whenever $l_1 \perp l_2$ and $l_2 \perp l_3$, it implies $l_1 \perp l_3$.
Consider three distinct lines in a plane. If $l_1 \perp l_2$ and $l_2 \perp l_3$, then line $l_1$ must be parallel to line $l_3$. For example, the x-axis ($l_1$) is perpendicular to the y-axis ($l_2$), and the line $x=5$ ($l_3$) is perpendicular to the y-axis ($l_2$). The x-axis ($l_1$) and the line $x=5$ ($l_3$) are parallel, not perpendicular.
Thus, $l_1 \perp l_2$ and $l_2 \perp l_3$ does not imply $l_1 \perp l_3$.
Therefore, the relation $R$ is not transitive.
4. Equivalence Relation:
A relation is an equivalence relation if it is reflexive, symmetric, and transitive.
Since $R$ is not reflexive and not transitive, it is not an equivalence relation.
Based on the analysis of the properties, the relation $R$ is symmetric, but not reflexive and not transitive.
The final answer is $\boxed{\text{Symmetric}}$.
Therefore, the correct option is (B) Symmetric.
Question 55. If $f(x) = x^2$ and $g(x) = 5x$, for what value(s) of $x$ is $f(x) = g(x)$?
(A) $x=0$ only
(B) $x=5$ only
(C) $x=0$ or $x=5$
(D) No real value of $x$
Answer:
Solution:
Given:
The function $f(x) = x^2$.
The function $g(x) = 5x$.
To Find:
The value(s) of $x$ for which $f(x) = g(x)$.
We are looking for the values of $x$ where the output of the function $f(x)$ is equal to the output of the function $g(x)$. Set the two function expressions equal to each other:
$f(x) = g(x)$
$x^2 = 5x$
To solve this equation, we move all terms to one side to get a quadratic equation:
$x^2 - 5x = 0$
Factor out the common term $x$ from the expression on the left side:
$x(x - 5) = 0$
For the product of two factors to be zero, at least one of the factors must be zero.
So, either the first factor is zero or the second factor is zero:
Case 1: $x = 0$
Case 2: $x - 5 = 0$
Solving for $x$ in Case 2:
$x = 5$
Thus, the values of $x$ for which $f(x) = g(x)$ are $x=0$ and $x=5$.
We can check these values:
- If $x=0$: $f(0) = 0^2 = 0$. $g(0) = 5(0) = 0$. $f(0) = g(0)$.
- If $x=5$: $f(5) = 5^2 = 25$. $g(5) = 5(5) = 25$. $f(5) = g(5)$.
The values $x=0$ and $x=5$ satisfy the condition $f(x) = g(x)$.
Comparing this result with the given options, we find that it matches option (C).
The final answer is $\boxed{x=0 \text{ or } x=5}$.
Therefore, the correct option is (C) $x=0$ or $x=5$.
Question 56. Let $A = \{x, y, z\}$. The number of relations on $A$ is:
(A) $3^3$
(B) $2^3$
(C) $2^{3 \times 3}$
(D) $2^6$
Answer:
Solution:
Given:
Set $A = \{x, y, z\}$.
To Find:
The number of relations on set $A$.
A relation on set $A$ is defined as any relation from set $A$ to set $A$. In other words, it is any subset of the Cartesian product $A \times A$.
First, find the number of elements in set $A$.
$n(A) = 3$.
Next, find the number of elements in the Cartesian product $A \times A$. If a set has $m$ elements, the number of elements in its Cartesian product with itself is $m \times m = m^2$.
$n(A \times A) = n(A) \times n(A) = 3 \times 3 = 9$.
The Cartesian product $A \times A$ is the set of all ordered pairs $(a_1, a_2)$ where $a_1 \in A$ and $a_2 \in A$.
$A \times A = \{(x, x), (x, y), (x, z), (y, x), (y, y), (y, z), (z, x), (z, y), (z, z)\}$
Indeed, there are 9 elements in $A \times A$.
The number of relations on $A$ is equal to the number of possible subsets of $A \times A$. The number of subsets of a set with $k$ elements is $2^k$.
Number of relations on $A = \text{Number of subsets of } A \times A = 2^{n(A \times A)}$
Substitute the calculated value of $n(A \times A)$:
Number of relations = $2^9$
Let's evaluate the options based on this result:
(A) $3^3 = 27$
(B) $2^3 = 8$
(C) $2^{3 \times 3} = 2^9$
(D) $2^6 = 64$
The calculated number of relations, $2^9$, matches option (C) because $3 \times 3 = 9$.
The final answer is $\boxed{2^{3 \times 3}}$.
Therefore, the correct option is (C) $2^{3 \times 3}$.
Question 57. The graph of the constant function $f(x) = c$ is:
(A) A horizontal line
(B) A vertical line
(C) A parabola
(D) A V-shaped curve
Answer:
Solution:
Given:
The constant function $f(x) = c$, where $c$ is a constant.
To Determine:
The shape of the graph of the constant function $f(x) = c$.
The graph of a function $f(x)$ is the set of all points $(x, y)$ in the Cartesian plane such that $y = f(x)$.
For the constant function $f(x) = c$, the equation of the graph is $y = c$.
The equation $y = c$ means that for any value of $x$ in the domain of the function (which is typically $\mathbb{R}$ for a constant function), the corresponding $y$-coordinate is always the same constant value $c$.
Let's consider some points on the graph:
- If $x=0$, $y=c$. Point is $(0, c)$.
- If $x=1$, $y=c$. Point is $(1, c)$.
- If $x=-5$, $y=c$. Point is $(-5, c)$.
- If $x=100$, $y=c$. Point is $(100, c)$.
When plotted on a graph, all these points lie on a straight line where the y-coordinate is always $c$. This line is parallel to the x-axis and passes through the point $(0, c)$ on the y-axis.
A line that is parallel to the x-axis is called a horizontal line.
Let's evaluate the options:
(A) A horizontal line: This matches our conclusion.
(B) A vertical line: A vertical line has an equation of the form $x = k$ (where $k$ is a constant). This is not the form $y=c$.
(C) A parabola: A parabola is a curved graph, typically represented by a quadratic equation like $y = ax^2 + bx + c$ or $x = ay^2 + by + c$. The graph of $y=c$ is a straight line.
(D) A V-shaped curve: A V-shaped curve is characteristic of the modulus function $f(x) = |x|$, not a constant function.
Therefore, the graph of the constant function $f(x) = c$ is a horizontal line.
The final answer is $\boxed{\text{A horizontal line}}$.
Therefore, the correct option is (A) A horizontal line.
Question 58. If $f(x) = 2x-1$, then the range of $f$ for the domain $\{1, 2, 3\}$ is:
(A) $\{1, 3, 5\}$
(B) $\{2, 4, 6\}$
(C) $\{1, 2, 3\}$
(D) $\{-1, 1, 3\}$
Answer:
Solution:
Given:
The function $f(x) = 2x-1$.
The domain of the function is the set $\{1, 2, 3\}$.
To Find:
The range of the function $f$ for the given domain.
The range of a function for a specific domain is the set of all output values obtained by applying the function rule to each element in the domain.
The domain is $\{1, 2, 3\}$. We need to find $f(x)$ for each value in this set.
For $x=1$:
$f(1) = 2(1) - 1$
$f(1) = 2 - 1$
$f(1) = 1$
For $x=2$:
$f(2) = 2(2) - 1$
$f(2) = 4 - 1$
$f(2) = 3$
For $x=3$:
$f(3) = 2(3) - 1$
$f(3) = 6 - 1$
$f(3) = 5$
The output values obtained are 1, 3, and 5 when the inputs are 1, 2, and 3 respectively.
The range of the function for the domain $\{1, 2, 3\}$ is the set of these output values.
Range of $f = \{1, 3, 5\}$
Comparing this result with the given options, we find that it matches option (A).
The final answer is $\boxed{\{1, 3, 5\}}$.
Therefore, the correct option is (A) $\{1, 3, 5\}$.
Question 59. Which of the following is a representation of a relation?
(A) Set-builder form
(B) Roster form
(C) Arrow diagram
(D) All of the above
Answer:
Solution:
To Determine:
Which of the given options is a representation of a relation.
A relation is a set of ordered pairs that describes a relationship between elements of two sets (or elements within the same set). Relations can be represented in various ways.
Let's examine each option:
(A) Set-builder form:
A relation can be described using set-builder notation, which specifies the rule or conditions that define the ordered pairs in the relation.
Example: $R = \{(x, y) : x \in A, y \in B, y = x+1\}$. This form describes the set of ordered pairs $(x, y)$ that satisfy the given condition.
Therefore, set-builder form is a representation of a relation.
(B) Roster form:
A relation can be listed explicitly by enumerating all the ordered pairs that belong to the relation. This is known as the roster form.
Example: If $A = \{1, 2\}$ and $B = \{2, 3\}$, the relation $R = \{(1, 2), (2, 3)\}$ lists the specific ordered pairs.
Therefore, roster form is a representation of a relation.
(C) Arrow diagram:
An arrow diagram is a visual representation of a relation. It involves drawing ovals or regions to represent the sets, and drawing arrows from an element in the first set to an element in the second set if they are related by the given relation.
Example: For the relation $R = \{(1, 2), (2, 3)\}$ from $A$ to $B$, we draw an arrow from 1 in $A$ to 2 in $B$, and an arrow from 2 in $A$ to 3 in $B$.
Therefore, an arrow diagram is a representation of a relation.
Since set-builder form, roster form, and arrow diagrams are all valid ways to represent a relation, the option that includes all of them is correct.
The final answer is $\boxed{\text{All of the above}}$.
Therefore, the correct option is (D) All of the above.
Question 60. The range of the function $f(x) = \sin(x)$ is:
(A) $(-1, 1)$
(B) $[-1, 1]$
(C) $\mathbb{R}$
(D) $[0, \infty)$
Answer:
Solution:
Given:
The function $f(x) = \sin(x)$.
To Find:
The range of the function $f(x)$.
The range of a function is the set of all possible output values that the function can produce for inputs in its domain.
The domain of the sine function $f(x) = \sin(x)$ is all real numbers, $\mathbb{R}$.
The sine function is a periodic function that oscillates between a minimum value and a maximum value.
The maximum value that $\sin(x)$ can attain is 1.
The minimum value that $\sin(x)$ can attain is -1.
For any real number $x$, the value of $\sin(x)$ is always between -1 and 1, inclusive.
So, for all $x \in \mathbb{R}$, we have:
$-1 \leq \sin(x) \leq 1$
The set of all possible output values of the function $f(x) = \sin(x)$ is the set of all real numbers $y$ such that $-1 \leq y \leq 1$.
In interval notation, this set is represented by the closed interval $[-1, 1]$.
Range of $f = [-1, 1]$
Comparing this result with the given options:
(A) $(-1, 1)$ - This is an open interval, excluding -1 and 1.
(B) $[-1, 1]$ - This is a closed interval, including -1 and 1.
(C) $\mathbb{R}$ - This is the set of all real numbers, which is incorrect as the sine function's output is bounded.
(D) $[0, \infty)$ - This is the set of non-negative real numbers, which is incorrect as the sine function can output negative values.
The final answer is $\boxed{[-1, 1]}$.
Therefore, the correct option is (B) $[-1, 1]$.
Question 61. If $f(x) = 5$ for all $x \in \mathbb{R}$, this is an example of:
(A) Identity function
(B) Constant function
(C) Polynomial function
(D) Both (B) and (C)
Answer:
Solution:
Given:
The function $f(x) = 5$ for all $x \in \mathbb{R}$.
To Determine:
Which type of function is $f(x) = 5$.
Let's analyze the properties and classifications of functions to match the given function $f(x) = 5$.
A Constant function is a function whose output value is the same for every input value. The general form is $f(x) = c$, where $c$ is a constant.
The given function $f(x) = 5$ fits this definition exactly, with the constant $c = 5$. The output is always 5, regardless of the input $x$.
A Polynomial function is a function that can be expressed in the form $f(x) = a_n x^n + a_{n-1} x^{n-1} + \dots + a_1 x + a_0$, where $a_0, a_1, \dots, a_n$ are constants and $n$ is a non-negative integer (the degree of the polynomial).
The given function $f(x) = 5$ can be written as $f(x) = 5 \cdot x^0$. This fits the general form of a polynomial where $n=0$ and $a_0 = 5$. A constant function (that is not the zero function $f(x)=0$) is a polynomial of degree zero.
An Identity function is a function that always returns the input value as the output value. The general form is $f(x) = x$.
The given function $f(x) = 5$ is not an identity function because $f(x)$ is always 5, whereas the identity function's output changes with the input $x$ (e.g., $f(1)=1$, $f(5)=5$).
Based on the analysis, the function $f(x) = 5$ is both a constant function and a polynomial function (specifically, a polynomial of degree 0).
Let's evaluate the given options:
(A) Identity function: Incorrect, as $f(x) \neq x$.
(B) Constant function: Correct, as $f(x)$ has a constant value 5 for all $x$.
(C) Polynomial function: Correct, as $f(x) = 5$ is a polynomial of degree 0.
(D) Both (B) and (C): This option combines the two correct classifications identified above.
Since the function $f(x)=5$ is indeed a constant function and a polynomial function, option (D) is the most comprehensive and correct answer among the choices.
The final answer is $\boxed{\text{Both (B) and (C)}}$.
Therefore, the correct option is (D) Both (B) and (C).
Question 62. Let $A = \{1, 2, 3, 4\}$. The relation $R = \{(x, y) : x \text{ divides } y, x, y \in A\}$. Which of the following pairs is NOT in $R$?
(A) $(1, 4)$
(B) $(2, 3)$
(C) $(3, 3)$
(D) $(4, 4)$
Answer:
Solution:
Given:
Set $A = \{1, 2, 3, 4\}$.
The relation $R$ on $A$ is defined as $R = \{(x, y) : x, y \in A, \text{ and } x \text{ divides } y\}$.
To Determine:
Which of the given ordered pairs is NOT in the relation $R$.
According to the definition of the relation $R$, an ordered pair $(x, y)$ is in $R$ if and only if both $x$ and $y$ are elements of the set $A = \{1, 2, 3, 4\}$, and $x$ is a divisor of $y$ (or $y$ is a multiple of $x$).
Let's list the elements of the relation $R$ by considering all possible pairs $(x, y)$ where $x, y \in A$ and checking if $x$ divides $y$:
- For $x=1$: 1 divides 1, 2, 3, 4. Pairs are $(1, 1), (1, 2), (1, 3), (1, 4)$.
- For $x=2$: 2 divides 2, 4. Pairs are $(2, 2), (2, 4)$.
- For $x=3$: 3 divides 3. Pair is $(3, 3)$. (3 does not divide 1, 2, or 4 in A)
- For $x=4$: 4 divides 4. Pair is $(4, 4)$. (4 does not divide 1, 2, or 3 in A)
The relation $R$ in roster form is:
$R = \{(1, 1), (1, 2), (1, 3), (1, 4), (2, 2), (2, 4), (3, 3), (4, 4)\}$
Now, we check each of the given options to see if it is present in the set $R$ or not.
(A) $(1, 4)$: Is $(1, 4) \in R$? Yes, $1 \in A$, $4 \in A$, and 1 divides 4. So, $(1, 4)$ is in $R$.
(B) $(2, 3)$: Is $(2, 3) \in R$? $2 \in A$ and $3 \in A$. Does 2 divide 3? No, 3 is not an integer multiple of 2.
So, $(2, 3)$ is NOT in $R$.
(C) $(3, 3)$: Is $(3, 3) \in R$? Yes, $3 \in A$, $3 \in A$, and 3 divides 3. So, $(3, 3)$ is in $R$.
(D) $(4, 4)$: Is $(4, 4) \in R$? Yes, $4 \in A$, $4 \in A$, and 4 divides 4. So, $(4, 4)$ is in $R$.
The only pair among the options that is not an element of the relation $R$ is $(2, 3)$.
The final answer is $\boxed{(2, 3)}$.
Therefore, the correct option is (B) $(2, 3)$.
Question 63. Consider the piecewise function $f(x) = \begin{cases} x+2 & , & x < 0 \\ x^2 & , & x \geq 0 \end{cases}$. Find $f(-3)$ and $f(3)$.
(A) $f(-3) = -1, f(3) = 9$
(B) $f(-3) = 5, f(3) = 6$
(C) $f(-3) = -1, f(3) = 6$
(D) $f(-3) = 5, f(3) = 9$
Answer:
Solution:
Given:
The piecewise function $f(x) = \begin{cases} x+2 & , & x < 0 \\ x^2 & , & x \geq 0 \end{cases}$.
To Find:
The values of $f(-3)$ and $f(3)$.
To evaluate the function at a specific value of $x$, we first need to determine which part of the piecewise definition applies based on the value of $x$.
Find $f(-3)$:**
The value we are interested in is $x = -3$.
We compare this value with the conditions given in the piecewise definition:
- Is $-3 < 0$? Yes, $-3$ is less than 0.
- Is $-3 \geq 0$? No, $-3$ is not greater than or equal to 0.
Since $-3 < 0$, we use the first part of the definition, $f(x) = x+2$.
Substitute $x=-3$ into this expression:
$f(-3) = (-3) + 2$
$f(-3) = -1$
Find $f(3)$:**
The value we are interested in is $x = 3$.
We compare this value with the conditions given in the piecewise definition:
- Is $3 < 0$? No, 3 is not less than 0.
- Is $3 \geq 0$? Yes, 3 is greater than or equal to 0.
Since $3 \geq 0$, we use the second part of the definition, $f(x) = x^2$.
Substitute $x=3$ into this expression:
$f(3) = (3)^2$
$f(3) = 3 \times 3$
$f(3) = 9$
So, we have $f(-3) = -1$ and $f(3) = 9$.
The final answer is $\boxed{f(-3) = -1, f(3) = 9}$.
Comparing this result with the given options, we find that it matches option (A).
Therefore, the correct option is (A) $f(-3) = -1, f(3) = 9$.
Short Answer Type Questions
Question 1. If $(x+1, y-2) = (3, 1)$, find the values of $x$ and $y$. Explain the concept of equality of ordered pairs.
Answer:
Given:
The equality of two ordered pairs: $(x+1, y-2) = (3, 1)$.
To Find:
The values of $x$ and $y$.
Solution:
According to the concept of equality of ordered pairs, if two ordered pairs are equal, their corresponding components must be equal.
Given $(x+1, y-2) = (3, 1)$, we can equate the first components and the second components separately.
Equating the first components:
$x+1 = 3$
Subtracting 1 from both sides:
$x = 3 - 1$
$x = 2$
Equating the second components:
$y-2 = 1$
Adding 2 to both sides:
$y = 1 + 2$
$y = 3$
Thus, the values are $x = 2$ and $y = 3$.
Concept of Equality of Ordered Pairs:
Two ordered pairs $(a, b)$ and $(c, d)$ are said to be equal if and only if their corresponding components are equal. This means the first component of the first pair must be equal to the first component of the second pair ($a = c$), and the second component of the first pair must be equal to the second component of the second pair ($b = d$).
Mathematically, $(a, b) = (c, d)$ if and only if $a = c$ and $b = d$.
This concept is fundamental in various areas of mathematics, including coordinate geometry (where points are represented by ordered pairs), functions (where elements are mapped from a domain to a range, often represented as ordered pairs), and set theory (in the definition of Cartesian products).
Question 2. Let A = $\{1, 2\}$ and B = $\{3, 4\}$. Find $A \times B$ and $B \times A$. Is $A \times B = B \times A$? Justify your answer.
Answer:
Given:
Set A = $\{1, 2\}$
Set B = $\{3, 4\}$
To Find:
$A \times B$
$B \times A$
Whether $A \times B = B \times A$ and justification.
Solution:
The Cartesian product of two sets A and B, denoted by $A \times B$, is the set of all ordered pairs $(a, b)$ where $a$ is an element of A and $b$ is an element of B. Mathematically, $A \times B = \{(a, b) \mid a \in A \text{ and } b \in B\}$.
Similarly, the Cartesian product of set B and set A, denoted by $B \times A$, is the set of all ordered pairs $(b, a)$ where $b$ is an element of B and $a$ is an element of A. Mathematically, $B \times A = \{(b, a) \mid b \in B \text{ and } a \in A\}$.
Let's find $A \times B$. We form all ordered pairs $(a, b)$ where $a \in \{1, 2\}$ and $b \in \{3, 4\}$.
- Take $a=1$. The possible pairs are $(1, 3)$ and $(1, 4)$.
- Take $a=2$. The possible pairs are $(2, 3)$ and $(2, 4)$.
So, $A \times B = \{(1, 3), (1, 4), (2, 3), (2, 4)\}$.
Now, let's find $B \times A$. We form all ordered pairs $(b, a)$ where $b \in \{3, 4\}$ and $a \in \{1, 2\}$.
- Take $b=3$. The possible pairs are $(3, 1)$ and $(3, 2)$.
- Take $b=4$. The possible pairs are $(4, 1)$ and $(4, 2)$.
So, $B \times A = \{(3, 1), (3, 2), (4, 1), (4, 2)\}$.
Comparison and Justification:
We have $A \times B = \{(1, 3), (1, 4), (2, 3), (2, 4)\}$ and $B \times A = \{(3, 1), (3, 2), (4, 1), (4, 2)\}$.
Two sets are equal if and only if they contain exactly the same elements. In this case, the elements are ordered pairs.
The ordered pairs in $A \times B$ are $(1, 3), (1, 4), (2, 3), (2, 4)$.
The ordered pairs in $B \times A$ are $(3, 1), (3, 2), (4, 1), (4, 2)$.
According to the definition of equality of ordered pairs, $(a, b) = (c, d)$ if and only if $a = c$ and $b = d$. For example, the ordered pair $(1, 3)$ in $A \times B$ is not equal to the ordered pair $(3, 1)$ in $B \times A$ because $1 \neq 3$ and $3 \neq 1$. None of the ordered pairs in $A \times B$ are present in $B \times A$, and vice versa.
Therefore, $A \times B$ and $B \times A$ do not contain the same ordered pairs.
So, $A \times B \neq B \times A$.
In general, for non-empty sets A and B, $A \times B = B \times A$ if and only if $A = B$. Since $A \neq B$ in this question, we expect $A \times B \neq B \times A$.
Question 3. If $A = \{1, 2, 3\}$ and $B = \{x, y\}$, list all possible relations from A to B. What is the total number of relations possible?
Answer:
Given:
Set A = $\{1, 2, 3\}$
Set B = $\{x, y\}$
To Find:
All possible relations from A to B.
The total number of possible relations.
Solution:
A relation from a set A to a set B is defined as any subset of the Cartesian product $A \times B$.
First, let's find the Cartesian product $A \times B$. The Cartesian product $A \times B$ is the set of all ordered pairs $(a, b)$ where $a \in A$ and $b \in B$.
$A \times B = \{(a, b) \mid a \in A \text{ and } b \in B\}$
$A \times B = \{(1, x), (1, y), (2, x), (2, y), (3, x), (3, y)\}$
The total number of elements in set A is $|A| = 3$.
The total number of elements in set B is $|B| = 2$.
The total number of elements in the Cartesian product $A \times B$ is $|A \times B| = |A| \times |B| = 3 \times 2 = 6$.
Since a relation from A to B is any subset of $A \times B$, the total number of possible relations is equal to the total number of subsets of $A \times B$.
The number of subsets of a set with $n$ elements is given by $2^n$.
Here, the set $A \times B$ has $n = 6$ elements.
Total number of possible relations = $2^{|A \times B|} = 2^6$.
Calculating $2^6$:
$2^6 = 2 \times 2 \times 2 \times 2 \times 2 \times 2 = 64$.
So, the total number of possible relations from A to B is 64.
Listing all possible relations:
Every subset of the Cartesian product $A \times B = \{(1, x), (1, y), (2, x), (2, y), (3, x), (3, y)\}$ is a relation from A to B. Listing all 64 possible subsets explicitly is extensive.
These relations range from the empty relation (containing no ordered pairs) to the universal relation (containing all ordered pairs from $A \times B$).
Examples of some possible relations are:
- The empty relation: $R_1 = \emptyset = \{\}$
- A relation containing only one ordered pair: $R_2 = \{(1, x)\}$, $R_3 = \{(2, y)\}$, etc.
- A relation containing some ordered pairs: $R_4 = \{(1, x), (2, x), (3, x)\}$
- A relation containing all ordered pairs (the universal relation): $R_{64} = \{(1, x), (1, y), (2, x), (2, y), (3, x), (3, y)\}$
Each of the 64 subsets formed by taking combinations of the 6 elements in $A \times B$ represents a distinct relation from A to B.
Question 4. Let R be a relation from $\mathbb{N}$ to $\mathbb{N}$ defined by $R = \{(x, y) : y = x+5, x < 4, x, y \in \mathbb{N}\}$. Write R in roster form. Find the domain and range of R.
Answer:
Given:
A relation R from $\mathbb{N}$ (natural numbers) to $\mathbb{N}$ defined by the rule $R = \{(x, y) : y = x+5, x < 4, x, y \in \mathbb{N}\}$.
Recall that the set of natural numbers $\mathbb{N} = \{1, 2, 3, 4, ...\}$.
To Find:
Roster form of the relation R.
Domain of R.
Range of R.
Solution:
The relation R consists of all ordered pairs $(x, y)$ such that $y$ is obtained by adding 5 to $x$, and $x$ is a natural number less than 4, with both $x$ and $y$ being natural numbers.
The condition on $x$ is $x \in \mathbb{N}$ and $x < 4$. The natural numbers less than 4 are 1, 2, and 3.
So, the possible values for $x$ are 1, 2, and 3.
Now, we find the corresponding values of $y$ using the rule $y = x+5$ for each valid $x$ and check if $y \in \mathbb{N}$.
- When $x = 1$: $y = 1 + 5 = 6$. Since $6 \in \mathbb{N}$, the ordered pair $(1, 6)$ is in R.
- When $x = 2$: $y = 2 + 5 = 7$. Since $7 \in \mathbb{N}$, the ordered pair $(2, 7)$ is in R.
- When $x = 3$: $y = 3 + 5 = 8$. Since $8 \in \mathbb{N}$, the ordered pair $(3, 8)$ is in R.
Since $x$ must be less than 4, these are the only possible values for $x$. Thus, these are all the ordered pairs in the relation R.
The roster form of the relation R is the set of these ordered pairs:
$R = \{(1, 6), (2, 7), (3, 8)\}$
The domain of a relation is the set of all first components (x-coordinates) of the ordered pairs in the relation.
Domain of R = $\{1, 2, 3\}$
The range of a relation is the set of all second components (y-coordinates) of the ordered pairs in the relation.
Range of R = $\{6, 7, 8\}$
Question 5. Determine whether the relation $R = \{(1, 2), (2, 3), (3, 4), (4, 5)\}$ is a function. Justify your answer.
Answer:
Given:
The relation $R = \{(1, 2), (2, 3), (3, 4), (4, 5)\}$.
To Determine:
Whether the relation R is a function.
Solution:
A relation is a function if every element in its domain is associated with exactly one element in its range (or codomain).
In other words, in the ordered pairs $(x, y)$ of a function, for any given value of $x$, there must be only one corresponding value of $y$. If a relation contains two distinct ordered pairs with the same first component but different second components, then it is not a function.
Let's examine the given relation $R = \{(1, 2), (2, 3), (3, 4), (4, 5)\}$.
The domain of the relation R consists of the first components of the ordered pairs: Domain(R) = $\{1, 2, 3, 4\}$.
Now let's check if each element in the domain is mapped to exactly one element:
- For the element 1 in the domain, the ordered pair is $(1, 2)$. The element 1 is associated only with 2.
- For the element 2 in the domain, the ordered pair is $(2, 3)$. The element 2 is associated only with 3.
- For the element 3 in the domain, the ordered pair is $(3, 4)$. The element 3 is associated only with 4.
- For the element 4 in the domain, the ordered pair is $(4, 5)$. The element 4 is associated only with 5.
In this relation, there are no two distinct ordered pairs that have the same first component. Each element in the domain $\{1, 2, 3, 4\}$ is associated with a unique element in the set $\{2, 3, 4, 5\}$.
Therefore, based on the definition of a function, the relation R is a function.
Justification:
The relation $R = \{(1, 2), (2, 3), (3, 4), (4, 5)\}$ is a function because for every element $x$ in the domain $\{1, 2, 3, 4\}$, there exists one and only one element $y$ such that $(x, y) \in R$. There are no cases where an element from the domain is paired with more than one element from the range.
Question 6. Find the domain and range of the real function $f(x) = x^2 + 2$.
Answer:
Given:
A real function $f(x) = x^2 + 2$.
To Find:
The domain of $f(x)$.
The range of $f(x)$.
Solution:
The function is given by $f(x) = x^2 + 2$. Since it is a real function, the domain and range are subsets of the set of real numbers, denoted by $\mathbb{R}$.
Domain of the function:
The domain of a function is the set of all possible input values ($x$) for which the function is defined in the set of real numbers. For the function $f(x) = x^2 + 2$, there are no restrictions on the values of $x$. Any real number can be squared, and adding 2 to the result will also be a real number.
Therefore, the domain of $f(x)$ is the set of all real numbers.
Domain($f$) = $\mathbb{R}$
In interval notation, the domain is $(-\infty, \infty)$.
Range of the function:
The range of a function is the set of all possible output values ($f(x)$) for the given domain. For $f(x) = x^2 + 2$, we analyze the possible values of $x^2$ and then $x^2 + 2$.
For any real number $x$, the square of $x$, $x^2$, is always greater than or equal to zero.
$x^2 \ge 0$
Now, we add 2 to both sides of the inequality:
$x^2 + 2 \ge 0 + 2$
$x^2 + 2 \ge 2$
Since $f(x) = x^2 + 2$, this means $f(x) \ge 2$.
The smallest possible value for $f(x)$ is 2, which occurs when $x = 0$ ($0^2 + 2 = 2$). As $x$ moves away from 0 (either positive or negative), $x^2$ increases, and consequently, $f(x)$ increases.
Therefore, the range of $f(x)$ is the set of all real numbers greater than or equal to 2.
Range($f$) = $\{y \in \mathbb{R} \mid y \ge 2\}$
In interval notation, the range is $[2, \infty)$.
Question 7. Define the Modulus function. Write its domain and range. Sketch its graph.
Answer:
Definition:
The Modulus function (or Absolute Value function) is a real function, denoted by $f(x) = |x|$. It is defined for every real number $x$ and gives the magnitude or non-negative value of $x$.
The definition is given by:
$|x| = \begin{cases} x & , \text{ if } x \ge 0 \\ -x & , \text{ if } x < 0 \end{cases}$
...
Domain:
The domain of the modulus function $f(x) = |x|$ is the set of all real numbers, because $|x|$ is defined for every real number $x$.
Domain($f$) = $\mathbb{R}$
In interval notation, the domain is $(-\infty, \infty)$.
Range:
The range of the modulus function $f(x) = |x|$ is the set of all non-negative real numbers, because the absolute value of any real number is always greater than or equal to zero.
$|x| \ge 0$ for all $x \in \mathbb{R}$.
The minimum value of $|x|$ is 0, which occurs at $x=0$. For any positive real number $y$, there exist values of $x$ (namely $x=y$ and $x=-y$) such that $|x|=y$.
Range($f$) = $\{y \in \mathbb{R} \mid y \ge 0\}$
In interval notation, the range is $[0, \infty)$.
Sketch of the Graph:
The graph of the modulus function $f(x) = |x|$ is a V-shaped curve with its vertex at the origin $(0, 0)$.
For $x \ge 0$, the function is $f(x) = x$, which is a straight line passing through the origin with a slope of 1. This part of the graph lies in the first quadrant.
For $x < 0$, the function is $f(x) = -x$, which is a straight line passing through the origin with a slope of -1. This part of the graph lies in the second quadrant.
Both lines meet at the origin $(0, 0)$, forming the vertex of the V-shape.
Key points on the graph include: $(0, 0)$, $(1, 1)$, $(-1, 1)$, $(2, 2)$, $(-2, 2)$, etc.
The graph is symmetric about the y-axis.
Question 8. Let $f(x) = x^2$ and $g(x) = 2x + 1$. Find $(f+g)(x)$, $(f-g)(x)$, and $(fg)(x)$.
Answer:
Given:
Function $f(x) = x^2$
Function $g(x) = 2x + 1$
To Find:
$(f+g)(x)$
$(f-g)(x)$
$(fg)(x)$
Solution:
We use the definitions for the sum, difference, and product of two functions $f$ and $g$.
1. Sum of functions $(f+g)(x)$:
The sum of two functions $f$ and $g$, denoted by $(f+g)(x)$, is defined as:
$(f+g)(x) = f(x) + g(x)$
Substitute the given expressions for $f(x)$ and $g(x)$:
$(f+g)(x) = (x^2) + (2x + 1)$
$(f+g)(x) = x^2 + 2x + 1$
2. Difference of functions $(f-g)(x)$:
The difference of two functions $f$ and $g$, denoted by $(f-g)(x)$, is defined as:
$(f-g)(x) = f(x) - g(x)$
Substitute the given expressions for $f(x)$ and $g(x)$:
$(f-g)(x) = (x^2) - (2x + 1)$
$(f-g)(x) = x^2 - 2x - 1$
3. Product of functions $(fg)(x)$:
The product of two functions $f$ and $g$, denoted by $(fg)(x)$, is defined as:
$(fg)(x) = f(x) \cdot g(x)$
Substitute the given expressions for $f(x)$ and $g(x)$:
$(fg)(x) = (x^2) \cdot (2x + 1)$
Now, distribute $x^2$ to each term inside the parenthesis:
$(fg)(x) = x^2 \cdot (2x) + x^2 \cdot (1)$
$(fg)(x) = 2x^{2+1} + x^2$
$(fg)(x) = 2x^3 + x^2$
Question 9. Find the domain of the real function $f(x) = \frac{x^2 + 2x + 1}{x^2 - 8x + 12}$.
Answer:
Given:
A real function $f(x) = \frac{x^2 + 2x + 1}{x^2 - 8x + 12}$.
To Find:
The domain of the function $f(x)$.
Solution:
The function $f(x)$ is a real function defined by a rational expression (a fraction of polynomials). For a real function to be defined, the expression must yield a real number. In the case of a rational function, the denominator cannot be equal to zero, as division by zero is undefined.
The domain of $f(x)$ is the set of all real numbers $x$ for which the denominator $x^2 - 8x + 12$ is not equal to zero.
To find the values of $x$ that are excluded from the domain, we set the denominator equal to zero and solve for $x$:
$x^2 - 8x + 12 = 0$
We can solve this quadratic equation by factoring. We need to find two numbers that multiply to 12 and add up to -8. These numbers are -2 and -6.
So, we can factor the quadratic expression as:
$(x - 2)(x - 6) = 0$
This equation is true if either factor is zero:
$(x - 2) = 0$
... (1)
$(x - 6) = 0$
... (2)
From equation (1):
$x = 2$
From equation (2):
$x = 6$
Thus, the denominator is zero when $x = 2$ or $x = 6$. These are the values that are not in the domain of the function.
The domain of $f(x)$ is the set of all real numbers except 2 and 6.
Domain($f$) = $\{x \in \mathbb{R} \mid x \neq 2 \text{ and } x \neq 6\}$
In interval notation, the domain is $(-\infty, 2) \cup (2, 6) \cup (6, \infty)$.
Question 10. Let A = $\{1, 2, 3, 4, 5\}$. Define a relation R on A as $R = \{(x, y) : y = x+1\}$. Represent R using an arrow diagram. Find the domain and range of R.
Answer:
Given:
Set A = $\{1, 2, 3, 4, 5\}$.
Relation R on A defined as $R = \{(x, y) : y = x+1\}$, where $x \in A$ and $y \in A$.
To Represent:
R using an arrow diagram.
To Find:
The domain of R.
The range of R.
Solution:
The relation R consists of all ordered pairs $(x, y)$ such that $y$ is one more than $x$, and both $x$ and $y$ are elements of the set A = $\{1, 2, 3, 4, 5\}$.
We find the ordered pairs $(x, y)$ that satisfy the condition $y = x+1$ with $x, y \in A$:
- If $x = 1$, then $y = 1 + 1 = 2$. Since $2 \in A$, the ordered pair $(1, 2)$ is in R.
- If $x = 2$, then $y = 2 + 1 = 3$. Since $3 \in A$, the ordered pair $(2, 3)$ is in R.
- If $x = 3$, then $y = 3 + 1 = 4$. Since $4 \in A$, the ordered pair $(3, 4)$ is in R.
- If $x = 4$, then $y = 4 + 1 = 5$. Since $5 \in A$, the ordered pair $(4, 5)$ is in R.
- If $x = 5$, then $y = 5 + 1 = 6$. Since $6 \notin A$, the ordered pair $(5, 6)$ is not in R.
So, the relation R in roster form is: $R = \{(1, 2), (2, 3), (3, 4), (4, 5)\}$.
Arrow Diagram:
To represent R using an arrow diagram, we list the elements of set A in two columns (representing the domain and codomain, both being set A in this case) and draw arrows from an element $x$ in the first column to an element $y$ in the second column if $(x, y) \in R$.
Element in Domain (from A) Element in Codomain (to A)
1 $\longrightarrow$ 2
2 $\longrightarrow$ 3
3 $\longrightarrow$ 4
4 $\longrightarrow$ 5
5
The element 5 in the domain column does not have an arrow pointing to any element in the codomain column because $(5, 6) \notin R$. The element 1 in the codomain column does not have any arrow pointing to it because there is no $x \in A$ such that $x+1=1$ (i.e., $x=0 \notin A$).
Domain of R:
The domain of R is the set of all first components of the ordered pairs in R.
Domain(R) = $\{1, 2, 3, 4\}$
Range of R:
The range of R is the set of all second components of the ordered pairs in R.
Range(R) = $\{2, 3, 4, 5\}$
Question 11. Determine whether each of the following relations is a function:
(i) $R_1 = \{(2, 1), (3, 1), (4, 2)\}$
(ii) $R_2 = \{(2, 2), (2, 4), (3, 3), (4, 4)\}$
Justify your answer in each case.
Answer:
To determine if a relation is a function, we need to check if every element in the domain of the relation is associated with exactly one element in the range (or codomain).
(i) Relation $R_1 = \{(2, 1), (3, 1), (4, 2)\}$
Given: $R_1 = \{(2, 1), (3, 1), (4, 2)\}$.
The ordered pairs in $R_1$ are $(2, 1)$, $(3, 1)$, and $(4, 2)$.
The first components (elements from the domain) are 2, 3, and 4.
Let's check the mapping for each element in the domain:
- For the element 2, the ordered pair is $(2, 1)$. 2 is associated only with 1.
- For the element 3, the ordered pair is $(3, 1)$. 3 is associated only with 1.
- For the element 4, the ordered pair is $(4, 2)$. 4 is associated only with 2.
In this relation, no two distinct ordered pairs have the same first component. Each element in the domain $\{2, 3, 4\}$ is associated with a unique element in the range $\{1, 2\}$. Note that different elements in the domain can be mapped to the same element in the range (as seen with 2 and 3 both mapping to 1), and the relation still be a function. What matters is that each element in the domain has only one output.
Conclusion: The relation $R_1$ is a function.
Justification: Every element in the domain of $R_1$ (which is $\{2, 3, 4\}$) is paired with exactly one element in the range (which is $\{1, 2\}$). There are no two distinct ordered pairs with the same first component but different second components.
(ii) Relation $R_2 = \{(2, 2), (2, 4), (3, 3), (4, 4)\}$
Given: $R_2 = \{(2, 2), (2, 4), (3, 3), (4, 4)\}$.
The ordered pairs in $R_2$ are $(2, 2)$, $(2, 4)$, $(3, 3)$, and $(4, 4)$.
The first components (elements from the domain) are 2, 3, and 4 (the value 2 appears more than once).
Let's check the mapping for each element in the domain:
- For the element 2, we have two ordered pairs: $(2, 2)$ and $(2, 4)$. This means the element 2 is associated with two different elements (2 and 4) in the range.
- For the element 3, the ordered pair is $(3, 3)$. 3 is associated only with 3.
- For the element 4, the ordered pair is $(4, 4)$. 4 is associated only with 4.
Since the element 2 in the domain is associated with more than one element (specifically, with both 2 and 4) in the set of second components, the relation $R_2$ does not satisfy the condition for being a function.
Conclusion: The relation $R_2$ is not a function.
Justification: The element 2 in the domain of $R_2$ is associated with two different values (2 and 4) in the range. This is evident from the presence of ordered pairs $(2, 2)$ and $(2, 4)$ in $R_2$. A function requires each element in the domain to be associated with exactly one element in the range.
Question 12. Find the range of the function $f(x) = |x-3|$.
Answer:
Given:
The real function $f(x) = |x-3|$.
To Find:
The range of the function $f(x)$.
Solution:
The function is defined by $f(x) = |x-3|$. This is a modulus function.
The modulus (or absolute value) of any real number is always non-negative.
For any real number $z$, we know that $|z| \ge 0$.
In our case, $z = x-3$. Since $x$ can be any real number (the domain of this function is $\mathbb{R}$), the expression $x-3$ can take any real value.
However, the absolute value of $x-3$, which is $|x-3|$, will always be greater than or equal to 0.
So, $|x-3| \ge 0$ for all $x \in \mathbb{R}$.
Since $f(x) = |x-3|$, this means $f(x) \ge 0$ for all $x \in \mathbb{R}$.
The smallest value that $|x-3|$ can take is 0, which occurs when $x-3 = 0$, i.e., when $x = 3$. At $x=3$, $f(3) = |3-3| = |0| = 0$.
As $x$ takes values further away from 3 (either greater than 3 or less than 3), the value of $|x-3|$ increases, and thus $f(x)$ increases.
The function $f(x) = |x-3|$ can take on any non-negative real value.
Therefore, the range of the function is the set of all non-negative real numbers.
Range($f$) = $\{y \in \mathbb{R} \mid y \ge 0\}$
In interval notation, the range is $[0, \infty)$.
Question 13. Define the Signum function. Write its domain and range. Sketch its graph.
Answer:
Definition:
The Signum function (or Sign function), denoted by $\text{sgn}(x)$, is a real function defined for every real number $x$. It gives the sign of a real number.
The definition is given by:
$\text{sgn}(x) = \begin{cases} 1 & , \text{ if } x > 0 \\ 0 & , \text{ if } x = 0 \\ -1 & , \text{ if } x < 0 \end{cases}$
...
Alternatively, it can be defined using the modulus function (for $x \neq 0$): $\text{sgn}(x) = \frac{|x|}{x} = \frac{x}{|x|}$ for $x \neq 0$, and $\text{sgn}(0) = 0$.
Domain:
The domain of the Signum function is the set of all real numbers, because the function is defined for every real number $x$ (whether $x > 0$, $x = 0$, or $x < 0$).
Domain($\text{sgn}$) = $\mathbb{R}$
In interval notation, the domain is $(-\infty, \infty)$.
Range:
The range of the Signum function is the set of all possible output values. According to the definition, the only possible values for $\text{sgn}(x)$ are 1, 0, or -1.
If $x$ is positive, $\text{sgn}(x) = 1$.
If $x$ is zero, $\text{sgn}(x) = 0$.
If $x$ is negative, $\text{sgn}(x) = -1$.
Therefore, the range of the Signum function is the set containing only these three values.
Range($\text{sgn}$) = $\{-1, 0, 1\}$
Sketch of the Graph:
The graph of the Signum function consists of three horizontal line segments:
- For $x > 0$, the graph is a horizontal line at $y = 1$. There is an open circle at $(0, 1)$ because this value is not included at $x=0$.
- For $x = 0$, the graph is a single point at $(0, 0)$.
- For $x < 0$, the graph is a horizontal line at $y = -1$. There is an open circle at $(0, -1)$ because this value is not included at $x=0$.
The graph shows a "jump discontinuity" at $x=0$.
Question 14. Let $f(x) = x^2$ and $g(x) = \sqrt{x}$. Find the domain of $(\frac{f}{g})(x)$.
Answer:
Given:
Function $f(x) = x^2$
Function $g(x) = \sqrt{x}$
To Find:
The domain of the function $(\frac{f}{g})(x)$.
Solution:
The quotient of two functions $f$ and $g$, denoted by $(\frac{f}{g})(x)$, is defined as:
$(\frac{f}{g})(x) = \frac{f(x)}{g(x)}$
Substitute the given expressions for $f(x)$ and $g(x)$:
$(\frac{f}{g})(x) = \frac{x^2}{\sqrt{x}}$
The domain of the quotient function $(\frac{f}{g})(x)$ consists of all values of $x$ that are in both the domain of $f(x)$ and the domain of $g(x)$, with the additional condition that $g(x)$ cannot be zero.
First, let's find the domain of $f(x) = x^2$.
The function $f(x) = x^2$ is a polynomial function. Polynomials are defined for all real numbers.
Domain($f$) = $\mathbb{R} = (-\infty, \infty)$.
Next, let's find the domain of $g(x) = \sqrt{x}$.
For a square root of a real number to be a real number, the expression under the square root must be non-negative.
$x \ge 0$
So, the domain of $g(x)$ is all real numbers greater than or equal to 0.
Domain($g$) = $\{x \in \mathbb{R} \mid x \ge 0\} = [0, \infty)$.
Now, we need to find the values of $x$ where $g(x) = 0$.
$g(x) = \sqrt{x}$
Set $g(x) = 0$:
$\sqrt{x} = 0$
Squaring both sides gives $x = 0$.
So, $g(x) = 0$ when $x = 0$.
The domain of $(\frac{f}{g})(x)$ is the intersection of the domains of $f$ and $g$, excluding the values of $x$ where $g(x) = 0$.
Domain($\frac{f}{g}$) = Domain($f$) $\cap$ Domain($g$) $\setminus \{x \mid g(x) = 0\}$
Domain($\frac{f}{g}$) = $(-\infty, \infty) \cap [0, \infty) \setminus \{0\}$
The intersection of $(-\infty, \infty)$ and $[0, \infty)$ is $[0, \infty)$.
$[0, \infty) \setminus \{0\}$ means we take the set $[0, \infty)$ and remove the element 0.
This results in the set of all real numbers strictly greater than 0.
Domain($\frac{f}{g}$) = $\{x \in \mathbb{R} \mid x > 0\}$
In interval notation, the domain is $(0, \infty)$.
Question 15. If $A = \{1, 2, 3\}$, find $A \times A \times A$. What is the number of elements in $A \times A \times A$?
Answer:
Given:
Set A = $\{1, 2, 3\}$.
To Find:
The Cartesian product $A \times A \times A$.
The number of elements in $A \times A \times A$.
Solution:
The Cartesian product of three sets A, B, and C is the set of all ordered triples $(a, b, c)$ where $a \in A$, $b \in B$, and $c \in C$.
In this case, we are finding the Cartesian product of set A with itself three times, which is $A \times A \times A$. This set consists of all ordered triples $(a, b, c)$ where $a \in A$, $b \in A$, and $c \in A$.
Given $A = \{1, 2, 3\}$, the elements $a$, $b$, and $c$ can each be 1, 2, or 3.
We can list all possible ordered triples $(a, b, c)$ by systematically choosing elements from A for the first, second, and third positions:
For $a=1$: $(1, 1, 1), (1, 1, 2), (1, 1, 3), (1, 2, 1), (1, 2, 2), (1, 2, 3), (1, 3, 1), (1, 3, 2), (1, 3, 3)$
For $a=2$: $(2, 1, 1), (2, 1, 2), (2, 1, 3), (2, 2, 1), (2, 2, 2), (2, 2, 3), (2, 3, 1), (2, 3, 2), (2, 3, 3)$
For $a=3$: $(3, 1, 1), (3, 1, 2), (3, 1, 3), (3, 2, 1), (3, 2, 2), (3, 2, 3), (3, 3, 1), (3, 3, 2), (3, 3, 3)$
Combining all these triples, we get the set $A \times A \times A$:
$A \times A \times A = \{(1, 1, 1), (1, 1, 2), (1, 1, 3), (1, 2, 1), (1, 2, 2), (1, 2, 3), (1, 3, 1), (1, 3, 2), (1, 3, 3), (2, 1, 1), (2, 1, 2), (2, 1, 3), (2, 2, 1), (2, 2, 2), (2, 2, 3), (2, 3, 1), (2, 3, 2), (2, 3, 3), (3, 1, 1), (3, 1, 2), (3, 1, 3), (3, 2, 1), (3, 2, 2), (3, 2, 3), (3, 3, 1), (3, 3, 2), (3, 3, 3)\}$
Number of elements in $A \times A \times A$:
The number of elements in a Cartesian product of sets is the product of the number of elements in each set.
The number of elements in set A is $|A| = 3$.
The number of elements in $A \times A \times A$ is given by:
$|A \times A \times A| = |A| \times |A| \times |A| = |A|^3$
Substitute the value of $|A|$:
$|A \times A \times A| = 3 \times 3 \times 3 = 3^3$
$3^3 = 27$
So, the total number of elements in $A \times A \times A$ is 27.
Question 16. Define the Greatest Integer function. Evaluate $\lfloor 3.7 \rfloor$, $\lfloor -1.2 \rfloor$, and $\lfloor 5 \rfloor$.
Answer:
Definition:
The Greatest Integer function, also known as the floor function, is a real function denoted by $\lfloor x \rfloor$ or $[x]$. It is defined for every real number $x$ and gives the greatest integer less than or equal to $x$.
Mathematically, $\lfloor x \rfloor = n$, where $n$ is an integer such that $n \le x < n+1$.
Domain:
The domain of the Greatest Integer function is the set of all real numbers, as the greatest integer less than or equal to $x$ exists for any real number $x$.
Domain($\lfloor x \rfloor$) = $\mathbb{R} = (-\infty, \infty)$.
Range:
The range of the Greatest Integer function is the set of all integers, as the output of the function is always an integer.
Range($\lfloor x \rfloor$) = $\mathbb{Z} = \{..., -2, -1, 0, 1, 2, ...\}$.
Evaluation:
We need to evaluate the function for the given values:
$\lfloor 3.7 \rfloor$:
We need to find the greatest integer less than or equal to 3.7. The integers less than or equal to 3.7 are ..., 0, 1, 2, 3. The greatest among these integers is 3.
So, $\lfloor 3.7 \rfloor = 3$.
(Using the definition: $3 \le 3.7 < 3+1 = 4$, so $n=3$)
$\lfloor -1.2 \rfloor$:
We need to find the greatest integer less than or equal to -1.2. The integers less than or equal to -1.2 are ..., -4, -3, -2. Note that -1 is not less than or equal to -1.2. The greatest among these integers is -2.
So, $\lfloor -1.2 \rfloor = -2$.
(Using the definition: $-2 \le -1.2 < -2+1 = -1$, so $n=-2$)
$\lfloor 5 \rfloor$:
We need to find the greatest integer less than or equal to 5. The integers less than or equal to 5 are ..., 3, 4, 5. The greatest among these integers is 5.
So, $\lfloor 5 \rfloor = 5$.
(Using the definition: $5 \le 5 < 5+1 = 6$, so $n=5$)
Question 17. Find the domain of the real function $f(x) = \sqrt{x - 5}$.
Answer:
Given:
A real function $f(x) = \sqrt{x - 5}$.
To Find:
The domain of the function $f(x)$.
Solution:
The function $f(x)$ involves a square root. For a real function defined by a square root, the expression under the square root symbol must be non-negative (greater than or equal to zero) for the function to produce a real number.
In this case, the expression under the square root is $x - 5$.
So, we must have:
$x - 5 \ge 0$
...
To find the values of $x$ for which this inequality holds, we add 5 to both sides:
$x - 5 + 5 \ge 0 + 5$
$x \ge 5$
Thus, the function $f(x) = \sqrt{x - 5}$ is defined for all real numbers $x$ that are greater than or equal to 5.
The domain of $f(x)$ is the set of all real numbers $x$ such that $x \ge 5$.
Domain($f$) = $\{x \in \mathbb{R} \mid x \ge 5\}$
In interval notation, the domain is $[5, \infty)$.
Question 18. Let R be the relation from a set A = {1, 2, 3, 4} to a set B = {1, 4, 5} defined by $R = \{(x, y) : x < y, x \in A, y \in B\}$. Write R in roster form.
Answer:
Given:
Set A = $\{1, 2, 3, 4\}$
Set B = $\{1, 4, 5\}$
Relation R from A to B defined by $R = \{(x, y) : x < y, x \in A, y \in B\}$.
To Find:
The relation R in roster form.
Solution:
The relation R consists of all ordered pairs $(x, y)$ such that $x$ is an element from set A, $y$ is an element from set B, and $x$ is strictly less than $y$.
We need to check every possible pair $(x, y)$ where $x \in A$ and $y \in B$ and see if the condition $x < y$ is satisfied.
- Consider $x = 1$ (from set A):
- Is $1 < 1$? No.
- Is $1 < 4$? Yes. So, $(1, 4) \in R$.
- Is $1 < 5$? Yes. So, $(1, 5) \in R$.
- Consider $x = 2$ (from set A):
- Is $2 < 1$? No.
- Is $2 < 4$? Yes. So, $(2, 4) \in R$.
- Is $2 < 5$? Yes. So, $(2, 5) \in R$.
- Consider $x = 3$ (from set A):
- Is $3 < 1$? No.
- Is $3 < 4$? Yes. So, $(3, 4) \in R$.
- Is $3 < 5$? Yes. So, $(3, 5) \in R$.
- Consider $x = 4$ (from set A):
- Is $4 < 1$? No.
- Is $4 < 4$? No.
- Is $4 < 5$? Yes. So, $(4, 5) \in R$.
The ordered pairs that satisfy the condition $x < y$ are $(1, 4), (1, 5), (2, 4), (2, 5), (3, 4), (3, 5), (4, 5)$.
The roster form of the relation R is the set of these ordered pairs:
$R = \{(1, 4), (1, 5), (2, 4), (2, 5), (3, 4), (3, 5), (4, 5)\}$
Question 19. Define a polynomial function. Give an example of a quadratic polynomial function and sketch its graph.
Answer:
Definition:
A polynomial function is a real function $f: \mathbb{R} \to \mathbb{R}$ defined by $f(x) = a_n x^n + a_{n-1} x^{n-1} + ... + a_1 x + a_0$, where $n$ is a non-negative integer called the degree of the polynomial, and $a_0, a_1, ..., a_n$ are real numbers called the coefficients, with $a_n \neq 0$ if $n > 0$. If $n=0$, the function is a constant function $f(x) = a_0$ (a polynomial of degree 0, provided $a_0 \neq 0$). The term $a_n x^n$ is the leading term and $a_0$ is the constant term.
The domain of a polynomial function is the set of all real numbers, $\mathbb{R}$.
Example of a Quadratic Polynomial Function:
A quadratic polynomial function is a polynomial function of degree 2. Its general form is $f(x) = ax^2 + bx + c$, where $a, b, c$ are real numbers and $a \neq 0$.
An example of a quadratic polynomial function is:
$f(x) = x^2 - 4x + 3$
Here, $a = 1$, $b = -4$, and $c = 3$. The degree is 2.
Sketch of the Graph of $f(x) = x^2 - 4x + 3$:
The graph of a quadratic function is a parabola. Since the leading coefficient ($a=1$) is positive, the parabola opens upwards, and it has a minimum point.
To sketch the graph, we can find key features:
1. Vertex: The x-coordinate of the vertex is given by $x = -\frac{b}{2a}$.
$x = -\frac{-4}{2(1)} = \frac{4}{2} = 2$
The y-coordinate of the vertex is $f(2) = (2)^2 - 4(2) + 3 = 4 - 8 + 3 = -1$.
The vertex is at $(2, -1)$. This is the minimum point of the graph.
2. Y-intercept: This is the point where the graph crosses the y-axis, which occurs when $x = 0$.
$f(0) = (0)^2 - 4(0) + 3 = 0 - 0 + 3 = 3$.
The y-intercept is at $(0, 3)$.
3. X-intercepts (Roots): These are the points where the graph crosses the x-axis, which occur when $f(x) = 0$.
$x^2 - 4x + 3 = 0$
Factor the quadratic: $(x - 1)(x - 3) = 0$.
So, $x - 1 = 0$ or $x - 3 = 0$.
$x = 1$ or $x = 3$.
The x-intercepts are at $(1, 0)$ and $(3, 0)$.
4. Symmetry: The parabola is symmetric about the vertical line passing through the vertex, i.e., the line $x = 2$.
Using these points (vertex $(2, -1)$, y-intercept $(0, 3)$, x-intercepts $(1, 0)$ and $(3, 0)$), we can sketch the parabola.
The graph starts from the top left, decreases to the vertex at $(2, -1)$, and then increases towards the top right, passing through the x-intercepts $(1, 0)$ and $(3, 0)$ and the y-intercept $(0, 3)$.
Question 20. If the domain of a function $f(x) = x^2$ is $\{-2, -1, 0, 1, 2\}$, find the range of $f$.
Answer:
Given:
Function $f(x) = x^2$.
Domain of $f$ = $\{-2, -1, 0, 1, 2\}$.
To Find:
The range of the function $f(x)$.
Solution:
The range of a function is the set of all possible output values ($f(x)$) that result from substituting the values from the domain into the function.
The domain is given as the set $\{-2, -1, 0, 1, 2\}$. We need to find the value of $f(x)$ for each element in the domain.
- When $x = -2$: $f(-2) = (-2)^2 = 4$.
- When $x = -1$: $f(-1) = (-1)^2 = 1$.
- When $x = 0$: $f(0) = (0)^2 = 0$.
- When $x = 1$: $f(1) = (1)^2 = 1$.
- When $x = 2$: $f(2) = (2)^2 = 4$.
The set of output values we obtained is $\{4, 1, 0, 1, 4\}$.
The range is the set of unique values in this collection.
Arranging the unique values in ascending order, the range is $\{0, 1, 4\}$.
Therefore, the range of the function $f(x) = x^2$ with the given domain $\{-2, -1, 0, 1, 2\}$ is $\{0, 1, 4\}$.
Range($f$) = $\{0, 1, 4\}$.
Question 21. Let $f = \{(1, 1), (2, 3), (0, -1), (-1, -3)\}$ be a linear function from $\mathbb{Z}$ to $\mathbb{Z}$ defined by $f(x) = ax + b$ for some integers $a$ and $b$. Determine $a$ and $b$.
Answer:
Given:
A linear function $f: \mathbb{Z} \to \mathbb{Z}$ defined by $f(x) = ax + b$, where $a$ and $b$ are integers.
The function is also given as the set of ordered pairs $f = \{(1, 1), (2, 3), (0, -1), (-1, -3)\}$.
To Determine:
The values of the integers $a$ and $b$.
Solution:
Since the given set of ordered pairs represents the function $f(x) = ax + b$, each ordered pair $(x, y)$ in the set satisfies the equation $y = ax + b$, where $y = f(x)$.
We can use any two of the given ordered pairs to form a system of linear equations in terms of $a$ and $b$. However, one of the ordered pairs provides a value directly.
Using the ordered pair $(0, -1)$:
Here $x = 0$ and $f(x) = -1$. Substitute these values into the equation $f(x) = ax + b$:
$-1 = a(0) + b$
$-1 = 0 + b$
$b = -1$
...
Now that we have the value of $b$, we can use any other ordered pair from the set to find the value of $a$. Let's use the ordered pair $(1, 1)$.
Here $x = 1$ and $f(x) = 1$. Substitute these values into the equation $f(x) = ax + b$, along with the value of $b$ we just found:
$1 = a(1) + (-1)$
$1 = a - 1$
Add 1 to both sides of the equation:
$1 + 1 = a$
$a = 2$
...
So, we have found $a = 2$ and $b = -1$. Both are integers as required.
The linear function is $f(x) = 2x - 1$.
Let's verify these values using the remaining ordered pairs:
- For $(2, 3)$: $f(2) = 2(2) + (-1) = 4 - 1 = 3$. This matches the given pair $(2, 3)$.
- For $(-1, -3)$: $f(-1) = 2(-1) + (-1) = -2 - 1 = -3$. This matches the given pair $(-1, -3)$.
The values $a = 2$ and $b = -1$ are consistent with all given points.
Therefore, the values of $a$ and $b$ are 2 and -1 respectively.
Question 22. Find the domain of the real function $f(x) = \frac{1}{\sqrt{x - 2}}$.
Answer:
Given:
A real function $f(x) = \frac{1}{\sqrt{x - 2}}$.
To Find:
The domain of the function $f(x)$.
Solution:
The function $f(x)$ is a real function involving a fraction and a square root in the denominator. For $f(x)$ to be a real number, two conditions must be met:
1. The expression under the square root must be non-negative (greater than or equal to zero) for the square root to be a real number.
2. The denominator cannot be equal to zero, as division by zero is undefined.
The expression under the square root is $x - 2$. For the square root to be real, we require:
$x - 2 \ge 0$
Adding 2 to both sides gives:
$x \ge 2$
... (1)
The denominator is $\sqrt{x - 2}$. For the denominator not to be zero, we require:
$\sqrt{x - 2} \neq 0$
This means the expression under the square root must not be zero:
$x - 2 \neq 0$
Adding 2 to both sides gives:
$x \neq 2$
... (2)
For the function $f(x)$ to be defined, both conditions (1) and (2) must be satisfied simultaneously.
From (1), we have $x \ge 2$.
From (2), we have $x \neq 2$.
Combining these two conditions, we need $x$ to be greater than or equal to 2, but not equal to 2. This means $x$ must be strictly greater than 2.
$x > 2$
The domain of $f(x)$ is the set of all real numbers $x$ such that $x > 2$.
Domain($f$) = $\{x \in \mathbb{R} \mid x > 2\}$
In interval notation, the domain is $(2, \infty)$.
Long Answer Type Questions
Question 1. Let A = $\{x : x \in \mathbb{R}, -1 \leq x \leq 2\}$ and B = $\{y : y \in \mathbb{R}, 0 \leq y \leq 3\}$. Illustrate $A \times B$ geometrically in the Cartesian plane. Define a relation R from A to B as $R = \{(x, y) : y = x^2\}$. Illustrate this relation in the Cartesian plane and find its domain and range.
Answer:
Given:
Set A = $\{x \in \mathbb{R} \mid -1 \leq x \leq 2\}$
Set B = $\{y \in \mathbb{R} \mid 0 \leq y \leq 3\}$
Relation R from A to B defined as $R = \{(x, y) : y = x^2, x \in A, y \in B\}$.
To Illustrate:
$A \times B$ geometrically in the Cartesian plane.
Relation R geometrically in the Cartesian plane.
To Find:
The domain of R.
The range of R.
Solution:
The Cartesian product $A \times B$ is the set of all ordered pairs $(x, y)$ such that $x \in A$ and $y \in B$.
$A \times B = \{(x, y) \mid x \in \mathbb{R}, -1 \leq x \leq 2 \text{ and } y \in \mathbb{R}, 0 \leq y \leq 3\}$
Geometric Illustration of $A \times B$:
In the Cartesian plane, the set A represents the interval $[-1, 2]$ on the x-axis, and the set B represents the interval $[0, 3]$ on the y-axis. The Cartesian product $A \times B$ corresponds to a rectangular region in the plane defined by these intervals.
The boundaries of this rectangular region are the vertical lines $x = -1$ and $x = 2$, and the horizontal lines $y = 0$ and $y = 3$. The region includes its boundaries because the inequalities are inclusive ($\leq$).
The four corners of this rectangle are the points $(-1, 0)$, $(2, 0)$, $(-1, 3)$, and $(2, 3)$.
So, $A \times B$ is the closed rectangular region with vertices at these points.
(Note: A sketch would typically be provided here, showing the rectangle bounded by $x=-1$, $x=2$, $y=0$, and $y=3$.)
Geometric Illustration of Relation R:
The relation R is the subset of $A \times B$ where the ordered pairs $(x, y)$ also satisfy the condition $y = x^2$. This means the graph of the relation R is the portion of the parabola $y = x^2$ that lies within the rectangular region defined by $A \times B$.
We need to consider the graph of $y = x^2$ only for $x$ values in the interval $[-1, 2]$ and $y$ values in the interval $[0, 3]$.
Let's examine the values of $y=x^2$ for $x \in [-1, 2]$:
- When $x = -1$, $y = (-1)^2 = 1$. The point $(-1, 1)$ is in $A \times B$ (since $-1 \in A$ and $1 \in B$). So, $(-1, 1) \in R$.
- When $x = 0$, $y = (0)^2 = 0$. The point $(0, 0)$ is in $A \times B$ (since $0 \in A$ and $0 \in B$). So, $(0, 0) \in R$.
- When $x = 1$, $y = (1)^2 = 1$. The point $(1, 1)$ is in $A \times B$ (since $1 \in A$ and $1 \in B$). So, $(1, 1) \in R$.
- When $x = 2$, $y = (2)^2 = 4$. The point $(2, 4)$ is NOT in $A \times B$ because $4 \notin B$ (since $y$ must be $\leq 3$). So, $(2, 4) \notin R$.
The graph of $y=x^2$ for $x \in [-1, 2]$ ranges from $y=(-1)^2=1$ at $x=-1$ up to $y=(2)^2=4$ at $x=2$. The minimum value is $y=0$ at $x=0$.
However, the relation R is restricted such that $y$ must be in the interval $[0, 3]$.
So, we are looking for points $(x, x^2)$ such that $-1 \le x \le 2$ and $0 \le x^2 \le 3$.
The condition $0 \le x^2 \le 3$ implies two separate conditions:
$x^2 \ge 0$: This is true for all real numbers $x$, and hence true for all $x \in [-1, 2]$.
$x^2 \le 3$: Taking the square root of both sides, $|x| \le \sqrt{3}$, which means $-\sqrt{3} \le x \le \sqrt{3}$.
We need to satisfy both the original domain condition for R ($x \in A$, i.e., $-1 \le x \le 2$) and the condition from the range restriction ($-\sqrt{3} \le x \le \sqrt{3}$).
The intersection of the intervals $[-1, 2]$ and $[-\sqrt{3}, \sqrt{3}]$ is $[-1, \sqrt{3}]$, since $\sqrt{3} \approx 1.732$ and $-1$ is within $[-\sqrt{3}, \sqrt{3}]$.
So, the x-values in R must be in the interval $[-1, \sqrt{3}]$.
The graph of the relation R is the part of the parabola $y=x^2$ where $x$ is in the interval $[-1, \sqrt{3}]$. This segment of the parabola starts at $(-1, (-1)^2) = (-1, 1)$ and ends at $(\sqrt{3}, (\sqrt{3})^2) = (\sqrt{3}, 3)$. The vertex $(0, 0)$ is also on this segment.
(Note: A sketch would typically show the parabola $y=x^2$ and highlight the segment from $x=-1$ to $x=\sqrt{3}$ that lies within the rectangle $A \times B$. The endpoints $(-1, 1)$ and $(\sqrt{3}, 3)$ would be shown on the boundary of the rectangle.)
Domain of R:
The domain of R is the set of all first components (x-values) of the ordered pairs in R. Based on our analysis above, the x-values must satisfy $x \in A$ and the corresponding $y=x^2$ must be in B. This led to the condition $x \in [-1, \sqrt{3}]$.
Domain(R) = $\{x \in \mathbb{R} \mid -1 \leq x \leq \sqrt{3}\}$
In interval notation, Domain(R) = $[-1, \sqrt{3}]$.
Range of R:
The range of R is the set of all second components (y-values) of the ordered pairs in R. The y-values are given by $y = x^2$, where $x \in [-1, \sqrt{3}]$.
For $x \in [-1, \sqrt{3}]$, the values of $y=x^2$ are as follows:
- The minimum value of $x^2$ in this interval occurs at $x=0$, where $y=0^2=0$.
- The maximum value of $x^2$ in this interval occurs at $x=\sqrt{3}$, where $y=(\sqrt{3})^2=3$. (At $x=-1$, $y=(-1)^2=1$).
As $x$ varies from -1 to $\sqrt{3}$, $x^2$ starts at 1, decreases to 0 at $x=0$, and then increases to 3 at $x=\sqrt{3}$. The set of all $x^2$ values for $x \in [-1, \sqrt{3}]$ is the interval $[0, 3]$.
We also know that the y-values must be in set B, which is $[0, 3]$.
The set of $y$ values generated by $y=x^2$ for $x \in [-1, \sqrt{3}]$ is $[0, 3]$. The intersection of this set with set B ($[0, 3]$) is simply $[0, 3]$.
Range(R) = $\{y \in \mathbb{R} \mid 0 \leq y \leq 3\}$
In interval notation, Range(R) = $[0, 3]$.
Question 2. Define a function. Given the relation $R = \{(x, x^2) : x \text{ is a prime number less than 10}\}$. (i) Write R in roster form. (ii) Determine if R is a function. Justify your answer. (iii) If R is a function, find its domain and range.
Answer:
Definition of a Function:
A function is a special type of relation where every element in the domain is associated with exactly one element in the codomain (or range). If $f$ is a function from a set A to a set B, denoted by $f: A \to B$, then for every element $a \in A$, there exists a unique element $b \in B$ such that the ordered pair $(a, b)$ is in the function $f$. In the context of ordered pairs, a relation $R$ is a function if and only if no two distinct ordered pairs in $R$ have the same first component.
Given:
The relation $R = \{(x, x^2) : x \text{ is a prime number less than 10}\}$.
(i) Roster form of R:
First, we need to identify the prime numbers less than 10.
A prime number is a natural number greater than 1 that has no positive divisors other than 1 and itself.
The natural numbers less than 10 are 1, 2, 3, 4, 5, 6, 7, 8, 9.
Among these, the prime numbers are 2, 3, 5, and 7.
So, the possible values for $x$ in the relation are 2, 3, 5, and 7.
Now, we find the corresponding $y$ values using the rule $y = x^2$ for each of these $x$ values:
- When $x = 2$, $y = 2^2 = 4$. The ordered pair is $(2, 4)$.
- When $x = 3$, $y = 3^2 = 9$. The ordered pair is $(3, 9)$.
- When $x = 5$, $y = 5^2 = 25$. The ordered pair is $(5, 25)$.
- When $x = 7$, $y = 7^2 = 49$. The ordered pair is $(7, 49)$.
The roster form of the relation R is the set of these ordered pairs:
$R = \{(2, 4), (3, 9), (5, 25), (7, 49)\}$
(ii) Determine if R is a function. Justify your answer:
We examine the ordered pairs in R to see if any two distinct pairs have the same first component.
The first components of the ordered pairs in R are 2, 3, 5, and 7.
Each of these first components is unique: 2 is paired only with 4; 3 is paired only with 9; 5 is paired only with 25; and 7 is paired only with 49.
Since every element in the set of first components $\{2, 3, 5, 7\}$ is associated with exactly one element in the set of second components $\{4, 9, 25, 49\}$, the relation R is a function.
Conclusion: The relation R is a function.
Justification: For each distinct input value (prime number less than 10), there is exactly one output value (its square). There are no two distinct ordered pairs in R that have the same first component but different second components.
(iii) Domain and range of R:
Since R is a function, we can find its domain and range.
The domain of the function R is the set of all first components of the ordered pairs in R.
Domain(R) = $\{2, 3, 5, 7\}$
This is the set of prime numbers less than 10, which is consistent with the definition of the relation.
The range of the function R is the set of all second components of the ordered pairs in R.
Range(R) = $\{4, 9, 25, 49\}$
This is the set of squares of the prime numbers less than 10.
Question 3. Find the domain and range of the real function $f(x) = \sqrt{9 - x^2}$. Illustrate the domain and range on the real number line.
Answer:
Given:
A real function $f(x) = \sqrt{9 - x^2}$.
To Find:
The domain of $f(x)$.
The range of $f(x)$.
Illustration of domain and range on the real number line.
Solution:
Domain of the function:
For the function $f(x) = \sqrt{9 - x^2}$ to be defined as a real number, the expression under the square root must be non-negative.
$9 - x^2 \ge 0$
Rearrange the inequality:
$-x^2 \ge -9$
Multiply by -1 and reverse the inequality sign:
$x^2 \le 9$
Taking the square root of both sides (and considering both positive and negative roots), we get:
$\sqrt{x^2} \le \sqrt{9}$
$|x| \le 3$
The inequality $|x| \le 3$ means that $x$ must be between -3 and 3, inclusive.
$-3 \le x \le 3$
...
The domain of $f(x)$ is the set of all real numbers $x$ such that $-3 \le x \le 3$.
Domain($f$) = $\{x \in \mathbb{R} \mid -3 \le x \le 3\}$
In interval notation, the domain is $[-3, 3]$.
Range of the function:
The range of $f(x) = \sqrt{9 - x^2}$ is the set of all possible values of $f(x)$ for $x$ in the domain $[-3, 3]$.
We know that for any real number $y = \sqrt{a}$, where $a \ge 0$, $y$ must be non-negative. Thus, $f(x) = \sqrt{9 - x^2} \ge 0$.
So the lower bound for the range is 0.
To find the maximum value of $f(x)$, we consider the values of $9 - x^2$ for $x \in [-3, 3]$.
Since $-3 \le x \le 3$, we have $0 \le x^2 \le 9$.
Multiplying by -1 reverses the inequality signs: $-9 \le -x^2 \le 0$.
Now, add 9 to all parts of the inequality:
$9 - 9 \le 9 - x^2 \le 9 + 0$
$0 \le 9 - x^2 \le 9$
So, the expression under the square root, $9 - x^2$, ranges from 0 to 9.
Now, take the square root of the inequality:
$\sqrt{0} \le \sqrt{9 - x^2} \le \sqrt{9}$
$0 \le f(x) \le 3$
The function $f(x)$ takes values between 0 and 3, inclusive.
The minimum value $f(x) = 0$ occurs when $9 - x^2 = 0$, i.e., $x = \pm 3$.
The maximum value $f(x) = 3$ occurs when $9 - x^2 = 9$, i.e., $x^2 = 0$, which means $x = 0$.
The range of $f(x)$ is the set of all real numbers $y$ such that $0 \le y \le 3$.
Range($f$) = $\{y \in \mathbb{R} \mid 0 \le y \le 3\}$
In interval notation, the range is $[0, 3]$.
Illustration on the real number line:
Domain: The domain $[-3, 3]$ is represented on the x-axis. It is a closed interval from -3 to 3, including the endpoints.
(Note: A diagram showing a number line with a closed segment marked from -3 to 3 would be placed here.)
Range: The range $[0, 3]$ is represented on the y-axis (or any other real number line for illustration). It is a closed interval from 0 to 3, including the endpoints.
(Note: A diagram showing a number line with a closed segment marked from 0 to 3 would be placed here.)
Alternatively, if illustrating on a single number line, one would mark the interval [-3, 3] and the interval [0, 3].
Question 4. Define the Greatest Integer function (Floor function), $f(x) = \lfloor x \rfloor$. Sketch the graph of this function for the interval $[-3, 3)$. State its domain and range. Explain why it is called the step function.
Answer:
Definition:
The Greatest Integer function, also known as the Floor function, is a real function denoted by $f(x) = \lfloor x \rfloor$. For any real number $x$, $\lfloor x \rfloor$ is defined as the largest integer that is less than or equal to $x$.
Mathematically, $f(x) = \lfloor x \rfloor = n$, where $n$ is an integer such that $n \le x < n+1$.
Domain:
The domain of the Greatest Integer function $f(x) = \lfloor x \rfloor$ is the set of all real numbers, $\mathbb{R}$, because for any real number $x$, there exists a greatest integer less than or equal to $x$.
Domain($f$) = $\mathbb{R} = (-\infty, \infty)$.
For the specific sketch requested over the interval $[-3, 3)$, the domain of consideration is the interval itself.
Range:
The range of the Greatest Integer function $f(x) = \lfloor x \rfloor$ is the set of all integers, $\mathbb{Z}$, because the output of the function is always an integer.
Range($f$) = $\mathbb{Z} = \{..., -2, -1, 0, 1, 2, ...\}$.
For the interval $[-3, 3)$, the values of $\lfloor x \rfloor$ will be the integers $-3, -2, -1, 0, 1, 2$. The range over this specific interval is $\{-3, -2, -1, 0, 1, 2\}$.
Sketch of the Graph for $[-3, 3)$:
The graph of the Greatest Integer function consists of horizontal line segments. For any interval $[n, n+1)$, where $n$ is an integer, the value of $\lfloor x \rfloor$ is constant and equal to $n$. The graph has jumps at integer values of $x$.
For the interval $[-3, 3)$:
- For $-3 \le x < -2$, $\lfloor x \rfloor = -3$. The graph is a horizontal line segment at $y=-3$ from $x=-3$ (inclusive) to $x=-2$ (exclusive).
- For $-2 \le x < -1$, $\lfloor x \rfloor = -2$. The graph is a horizontal line segment at $y=-2$ from $x=-2$ (inclusive) to $x=-1$ (exclusive).
- For $-1 \le x < 0$, $\lfloor x \rfloor = -1$. The graph is a horizontal line segment at $y=-1$ from $x=-1$ (inclusive) to $x=0$ (exclusive).
- For $0 \le x < 1$, $\lfloor x \rfloor = 0$. The graph is a horizontal line segment at $y=0$ from $x=0$ (inclusive) to $x=1$ (exclusive).
- For $1 \le x < 2$, $\lfloor x \rfloor = 1$. The graph is a horizontal line segment at $y=1$ from $x=1$ (inclusive) to $x=2$ (exclusive).
- For $2 \le x < 3$, $\lfloor x \rfloor = 2$. The graph is a horizontal line segment at $y=2$ from $x=2$ (inclusive) to $x=3$ (exclusive).
At each integer value on the left end of an interval $[n, n+1)$, the point $(n, n)$ is included (represented by a closed circle). At each integer value on the right end of an interval $[n, n+1)$, the point $(n+1, n)$ is not included (represented by an open circle). For the interval $[-3, 3)$, this means a closed circle at $(-3, -3)$, an open circle at $(-2, -3)$, a closed circle at $(-2, -2)$, an open circle at $(-1, -2)$, and so on, up to a closed circle at $(2, 2)$ and an open circle at $(3, 2)$.
Why it is called the step function:
The graph of the Greatest Integer function resembles a series of steps. It consists of horizontal line segments of length 1, which jump vertically upwards by 1 unit at each integer value of $x$. This staircase-like appearance is why it is commonly referred to as a step function.
Question 5. Let $f(x) = x^2 + 1$ and $g(x) = |x|$. Define the functions $(f+g)(x)$, $(f-g)(x)$, $(fg)(x)$, and $(\frac{f}{g})(x)$. Find the domain of each of these resulting functions.
Answer:
Given:
Function $f(x) = x^2 + 1$
Function $g(x) = |x|$
Domain of the individual functions:
The function $f(x) = x^2 + 1$ is a polynomial function. The domain of any polynomial function is the set of all real numbers.
Domain($f$) = $\mathbb{R} = (-\infty, \infty)$.
The function $g(x) = |x|$ is the modulus function. The domain of the modulus function is the set of all real numbers.
Domain($g$) = $\mathbb{R} = (-\infty, \infty)$.
1. Sum of functions $(f+g)(x)$:
Definition: $(f+g)(x) = f(x) + g(x)$
$(f+g)(x) = (x^2 + 1) + |x|$
$(f+g)(x) = x^2 + 1 + |x|$
Domain: The domain of $(f+g)(x)$ is the intersection of the domains of $f(x)$ and $g(x)$.
Domain($f+g$) = Domain($f$) $\cap$ Domain($g$)
Domain($f+g$) = $\mathbb{R} \cap \mathbb{R} = \mathbb{R}$
In interval notation, the domain is $(-\infty, \infty)$.
2. Difference of functions $(f-g)(x)$:
Definition: $(f-g)(x) = f(x) - g(x)$
$(f-g)(x) = (x^2 + 1) - |x|$
$(f-g)(x) = x^2 + 1 - |x|$
Domain: The domain of $(f-g)(x)$ is the intersection of the domains of $f(x)$ and $g(x)$.
Domain($f-g$) = Domain($f$) $\cap$ Domain($g$)
Domain($f-g$) = $\mathbb{R} \cap \mathbb{R} = \mathbb{R}$
In interval notation, the domain is $(-\infty, \infty)$.
3. Product of functions $(fg)(x)$:
Definition: $(fg)(x) = f(x) \cdot g(x)$
$(fg)(x) = (x^2 + 1) \cdot |x|$
$(fg)(x) = |x|(x^2 + 1)$
Domain: The domain of $(fg)(x)$ is the intersection of the domains of $f(x)$ and $g(x)$.
Domain($fg$) = Domain($f$) $\cap$ Domain($g$)
Domain($fg$) = $\mathbb{R} \cap \mathbb{R} = \mathbb{R}$
In interval notation, the domain is $(-\infty, \infty)$.
4. Quotient of functions $(\frac{f}{g})(x)$:
Definition: $(\frac{f}{g})(x) = \frac{f(x)}{g(x)}$
$(\frac{f}{g})(x) = \frac{x^2 + 1}{|x|}$
Domain: The domain of $(\frac{f}{g})(x)$ is the intersection of the domains of $f(x)$ and $g(x)$, excluding the values of $x$ where $g(x) = 0$.
Domain($\frac{f}{g}$) = Domain($f$) $\cap$ Domain($g$) $\setminus \{x \mid g(x) = 0\}$
We know Domain($f$) = $\mathbb{R}$ and Domain($g$) = $\mathbb{R}$. Their intersection is $\mathbb{R}$.
Now, we find the values of $x$ where $g(x) = 0$.
$g(x) = |x|$
Set $g(x) = 0$:
$|x| = 0$
This implies $x = 0$.
So, we must exclude $x = 0$ from the domain.
Domain($\frac{f}{g}$) = $\mathbb{R} \setminus \{0\}$
In interval notation, the domain is $(-\infty, 0) \cup (0, \infty)$.
Question 6. Find the domain and range of the real function $f(x) = \frac{x-2}{3-x}$.
Answer:
Given:
A real function $f(x) = \frac{x-2}{3-x}$.
To Find:
The domain of $f(x)$.
The range of $f(x)$.
Solution:
The function $f(x)$ is a real function defined by a rational expression. For $f(x)$ to be defined, the denominator cannot be equal to zero.
Domain of the function:
The denominator is $3-x$. We must ensure that $3-x \neq 0$.
$3 - x \neq 0$
Add $x$ to both sides:
$3 \neq x$
Or, $x \neq 3$.
The domain of $f(x)$ is the set of all real numbers except 3.
Domain($f$) = $\{x \in \mathbb{R} \mid x \neq 3\}$
In interval notation, the domain is $(-\infty, 3) \cup (3, \infty)$.
Range of the function:
The range of $f(x)$ is the set of all possible output values ($y = f(x)$) for $x$ in the domain.
Let $y = f(x)$. We have:
$y = \frac{x-2}{3-x}$
To find the range, we try to express $x$ in terms of $y$. Multiply both sides by $(3-x)$:
$y(3-x) = x-2$
Distribute $y$ on the left side:
$3y - xy = x - 2$
We want to isolate $x$. Move all terms containing $x$ to one side and terms without $x$ to the other side. Let's move $-xy$ to the right side and $-2$ to the left side:
$3y + 2 = x + xy$
Factor out $x$ on the right side:
$3y + 2 = x(1 + y)$
Now, divide by $(1+y)$ to solve for $x$, assuming $1+y \neq 0$:
$x = \frac{3y + 2}{1 + y}$
...
For $x$ to be a real number, the denominator $(1+y)$ cannot be zero.
$1 + y \neq 0$
$y \neq -1$
This means that the function $f(x)$ can take any real value except -1.
The range of $f(x)$ is the set of all real numbers except -1.
Range($f$) = $\{y \in \mathbb{R} \mid y \neq -1\}$
In interval notation, the range is $(-\infty, -1) \cup (-1, \infty)$.
Question 7. Let $f$ be the relation from $\mathbb{R}$ to $\mathbb{R}$ defined by $f(x) = \{(x, x^2) : x \in \mathbb{R}\}$. Is $f$ a function? Justify. Sketch the graph of $f(x) = x^2$ for $x \in [-3, 3]$. State its domain and range over this interval.
Answer:
Given:
A relation $f$ from $\mathbb{R}$ to $\mathbb{R}$ defined by $f = \{(x, x^2) : x \in \mathbb{R}\}$. This means the relation consists of all ordered pairs where the second component is the square of the first component, and the first component is any real number.
To Determine:
Whether the relation $f$ is a function, with justification.
To Sketch the graph of $f(x) = x^2$ for $x \in [-3, 3]$.
To State the domain and range of $f(x) = x^2$ over the interval $[-3, 3]$.
Solution - Is $f$ a function?
A function is a relation where every element in the domain is associated with exactly one element in the codomain. In terms of ordered pairs $(x, y)$, a relation is a function if for any given value of $x$, there is only one possible value of $y$.
The given relation is $f = \{(x, x^2) : x \in \mathbb{R}\}$. For any real number $x$, its square, $x^2$, is a unique real number.
For example:
- If $x = 1$, $x^2 = 1$. The ordered pair is $(1, 1)$.
- If $x = -1$, $x^2 = 1$. The ordered pair is $(-1, 1)$.
- If $x = 2$, $x^2 = 4$. The ordered pair is $(2, 4)$.
- If $x = -2$, $x^2 = 4$. The ordered pair is $(-2, 4)$.
Even though different x-values (-1 and 1, or -2 and 2) can result in the same y-value (1 or 4), this does not violate the definition of a function. The crucial point is that for a *single* x-value, there is only *one* corresponding y-value.
In the relation $f = \{(x, x^2) : x \in \mathbb{R}\}$, for any chosen real number $x$, the value of $x^2$ is uniquely determined. Therefore, there are no two distinct ordered pairs in this relation that have the same first component but different second components.
Conclusion: The relation $f$ is a function.
Justification: For every real number $x$ in the domain ($\mathbb{R}$), there is a unique value $x^2$ in the codomain ($\mathbb{R}$). Thus, the definition of a function is satisfied.
Sketch of the graph of $f(x) = x^2$ for $x \in [-3, 3]$:
The function $f(x) = x^2$ is a quadratic function, and its graph is a parabola that opens upwards with its vertex at the origin $(0, 0)$.
We are asked to sketch the graph for the interval $[-3, 3]$. This means we consider only the part of the parabola where the x-values are between -3 and 3, inclusive.
Let's find the y-values at the endpoints of the interval:
- At $x = -3$, $f(-3) = (-3)^2 = 9$. The point is $(-3, 9)$.
- At $x = 3$, $f(3) = (3)^2 = 9$. The point is $(3, 9)$.
The vertex is at $x=0$, $f(0) = 0^2 = 0$. The point is $(0, 0)$.
The graph starts at $(-3, 9)$, curves downwards through the vertex at $(0, 0)$, and then curves upwards to end at $(3, 9)$. The graph is symmetric about the y-axis.
Since the interval $[-3, 3]$ is closed (inclusive of endpoints), the points $(-3, 9)$ and $(3, 9)$ are included on the graph.
(Note: A sketch would typically be provided here, showing the parabolic segment between $x=-3$ and $x=3$, with the vertex at $(0,0)$ and endpoints at $(-3,9)$ and $(3,9)$. The y-axis would be labeled, and the x-axis marked with -3, 0, and 3.)
Domain and Range of $f(x) = x^2$ over $[-3, 3]$:
The domain of the function, restricted to the given interval for the sketch, is explicitly stated in the question.
Domain: The domain of $f(x) = x^2$ over the interval $[-3, 3]$ is the set of all possible input values $x$, which is $[-3, 3]$.
Domain($f$) for this interval = $[-3, 3]$.
Range: The range is the set of all output values $f(x)$ corresponding to the input values in the domain $[-3, 3]$.
For $x \in [-3, 3]$, the value of $x^2$ ranges from $0^2=0$ (at $x=0$) to $3^2=9$ (at $x=\pm 3$).
Since $x^2$ is always non-negative, the minimum value is 0.
The maximum value occurs at the endpoints of the interval $[-3, 3]$, where $x^2 = (\pm 3)^2 = 9$.
As $x$ varies from -3 to 3, $x^2$ covers all values from 0 up to 9.
So, the range of $f(x) = x^2$ for $x \in [-3, 3]$ is the interval $[0, 9]$.
Range($f$) for this interval = $[0, 9]$.
Question 8. Define the identity function and the constant function. Sketch the graphs of $f(x) = x$ and $g(x) = c$ (for some constant c). State their domains and ranges.
Answer:
Definition of Identity Function:
The Identity function is a real function, denoted by $f(x) = x$. It is defined for every real number $x$. The function maps each element in its domain to itself in the codomain. In other words, the output value is always equal to the input value.
$f: \mathbb{R} \to \mathbb{R}$ defined by $f(x) = x$.
Graph of Identity Function $f(x) = x$:
The graph of the identity function $f(x) = x$ is a straight line that passes through the origin $(0, 0)$. It has a slope of 1 and makes an angle of $45^\circ$ with the positive x-axis.
Key points on the graph include $(0, 0)$, $(1, 1)$, $(2, 2)$, $(-1, -1)$, etc. For any point $(x, y)$ on the graph, $y = x$.
(Note: A sketch would typically show a straight line passing through the origin with a slope of 1.)
Domain and Range of Identity Function $f(x) = x$:
Domain: The identity function $f(x) = x$ is defined for all real numbers $x$.
Domain($f$) = $\mathbb{R} = (-\infty, \infty)$.
Range: For every real number $x$ as input, the output $f(x) = x$ is also a real number. Every real number can be an output.
Range($f$) = $\mathbb{R} = (-\infty, \infty)$.
Definition of Constant Function:
The Constant function is a real function, denoted by $g(x) = c$, where $c$ is a fixed real number. It is defined for every real number $x$. The function maps every element in its domain to the same single value $c$ in the codomain.
$g: \mathbb{R} \to \mathbb{R}$ defined by $g(x) = c$, where $c \in \mathbb{R}$.
Sketch of the Graph of Constant Function $g(x) = c$:
The graph of the constant function $g(x) = c$ is a horizontal straight line. For any value of $x$, the value of $g(x)$ is always equal to the constant $c$. The line is parallel to the x-axis and passes through the point $(0, c)$ on the y-axis.
If $c = 0$, the graph is the x-axis itself.
(Note: A sketch would typically show a horizontal line crossing the y-axis at $y=c$.) For example, if $c=2$, the line is $y=2$.
Domain and Range of Constant Function $g(x) = c$:
Domain: The constant function $g(x) = c$ is defined for all real numbers $x$.
Domain($g$) = $\mathbb{R} = (-\infty, \infty)$.
Range: For every real number $x$ as input, the output $g(x)$ is always the single value $c$. The range is the set containing only the value $c$.
Range($g$) = $\{c\}$.
Question 9. Find the domain of the real function $f(x) = \frac{1}{\sqrt{(x-1)(x-3)}}$.
Answer:
Given:
A real function $f(x) = \frac{1}{\sqrt{(x-1)(x-3)}}$.
To Find:
The domain of the function $f(x)$.
Solution:
The function $f(x)$ is a real function involving a fraction and a square root in the denominator. For $f(x)$ to be defined as a real number, two conditions must be met:
1. The expression under the square root must be non-negative (greater than or equal to zero).
2. The denominator cannot be equal to zero.
The expression under the square root is $(x-1)(x-3)$. For the square root to be real, we require:
$(x-1)(x-3) \ge 0$
... (1)
The denominator is $\sqrt{(x-1)(x-3)}$. For the denominator not to be zero, we require:
$\sqrt{(x-1)(x-3)} \neq 0$
This means the expression under the square root must not be zero:
$(x-1)(x-3) \neq 0$
... (2)
Combining conditions (1) and (2), we need the expression under the square root to be strictly positive:
$(x-1)(x-3) > 0$
...
We need to solve the inequality $(x-1)(x-3) > 0$. The roots of the quadratic $x^2 - 4x + 3$ are $x = 1$ and $x = 3$. These points divide the real number line into three intervals: $(-\infty, 1)$, $(1, 3)$, and $(3, \infty)$.
We test the sign of the expression $(x-1)(x-3)$ in each interval:
- Interval $(-\infty, 1)$: Choose a test value, say $x = 0$. $(0-1)(0-3) = (-1)(-3) = 3 > 0$. The inequality holds in this interval.
- Interval $(1, 3)$: Choose a test value, say $x = 2$. $(2-1)(2-3) = (1)(-1) = -1 < 0$. The inequality does not hold in this interval.
- Interval $(3, \infty)$: Choose a test value, say $x = 4$. $(4-1)(4-3) = (3)(1) = 3 > 0$. The inequality holds in this interval.
Alternatively, since the quadratic $x^2 - 4x + 3$ has a positive leading coefficient (the coefficient of $x^2$ is 1), the parabola opens upwards. The expression $(x-1)(x-3)$ is positive outside its roots (1 and 3) and negative between its roots.
The inequality $(x-1)(x-3) > 0$ is satisfied when $x < 1$ or $x > 3$.
The domain of $f(x)$ is the set of all real numbers $x$ such that $x < 1$ or $x > 3$.
Domain($f$) = $\{x \in \mathbb{R} \mid x < 1 \text{ or } x > 3\}$
In interval notation, the domain is $(-\infty, 1) \cup (3, \infty)$.
Question 10. Let $f(x) = \begin{cases} x^2 & , & x \leq 0 \\ x & , & x > 0 \end{cases}$. Sketch the graph of this piecewise function. Determine its domain and range.
Answer:
Given:
The piecewise function $f(x) = \begin{cases} x^2 & , & x \leq 0 \\ x & , & x > 0 \end{cases}$.
To Sketch:
The graph of the function.
To Determine:
The domain and range of the function.
Solution:
The function $f(x)$ is defined by different rules depending on the value of $x$. This is a piecewise function.
The function is defined in two parts:
Part 1: $f(x) = x^2$ for all real numbers $x$ such that $x \leq 0$.
Part 2: $f(x) = x$ for all real numbers $x$ such that $x > 0$.
Domain:
The domain of the function is the set of all input values ($x$) for which the function is defined.
The first part of the function ($f(x) = x^2$) is defined for the interval $(-\infty, 0]$.
The second part of the function ($f(x) = x$) is defined for the interval $(0, \infty)$.
Since every real number $x$ falls into either the category $x \leq 0$ or $x > 0$, the function is defined for all real numbers.
The domain is the union of these two intervals: $(-\infty, 0] \cup (0, \infty) = (-\infty, \infty)$.
Domain($f$) = $\mathbb{R}$
In interval notation, the domain is $(-\infty, \infty)$.
Range:
The range of the function is the set of all possible output values ($f(x)$).
For the first part, $f(x) = x^2$ for $x \leq 0$. The square of any non-positive real number is always non-negative. As $x$ takes values from $-\infty$ up to 0, $x^2$ takes values from arbitrarily large positive numbers down to $0^2 = 0$. The range for this part is $[0, \infty)$.
For the second part, $f(x) = x$ for $x > 0$. The output values are the same as the input values. As $x$ takes values strictly greater than 0, $f(x)$ takes values strictly greater than 0. The range for this part is $(0, \infty)$.
The range of the entire function is the union of the ranges of the two parts:
Range($f$) = $[0, \infty) \cup (0, \infty)$.
The union of the set of non-negative numbers $[0, \infty)$ and the set of strictly positive numbers $(0, \infty)$ is the set of all non-negative numbers.
Range($f$) = $[0, \infty)$.
In interval notation, the range is $[0, \infty)$.
Sketch of the Graph:
To sketch the graph of this piecewise function, we combine the graphs of the two definitions over their respective intervals.
1. For $x \leq 0$, the graph is $y = x^2$. This is the left half of the standard parabola $y=x^2$. It starts from the left and ends at the point where $x=0$, which is $(0, 0^2) = (0, 0)$. The point $(0, 0)$ is included because the condition is $x \leq 0$.
2. For $x > 0$, the graph is $y = x$. This is a straight line passing through the origin with a slope of 1. This part starts just after the point where $x=0$. The point $(0, 0)$ is not included in this part because the condition is $x > 0$. However, as $x$ approaches 0 from the right, $y=x$ approaches 0, so the line segment starts "infinitesimally" to the right of $(0,0)$ and moves upwards to the right.
Combining these two parts, the graph is the left half of the parabola $y=x^2$ for $x \le 0$, joined smoothly at the origin $(0,0)$ with the line $y=x$ for $x > 0$.
Key points to plot:
- For $x \le 0$: $(-2, (-2)^2=4)$, $(-1, (-1)^2=1)$, $(0, 0^2=0)$.
- For $x > 0$: $(0.5, 0.5)$, $(1, 1)$, $(2, 2)$.
The graph starts as a parabola from the left, hits the origin, and then becomes a straight line going towards the top right.
Question 11. Let A and B be two finite sets with $\text{n}(A) = p$ and $\text{n}(B) = q$. Find the number of relations from A to B and the number of functions from A to B. If $p=3$ and $q=2$, calculate these numbers.
Answer:
Given:
A and B are finite sets.
Number of elements in set A, $\text{n}(A) = p$.
Number of elements in set B, $\text{n}(B) = q$.
To Find:
The number of relations from A to B.
The number of functions from A to B.
Calculate these numbers for $p=3$ and $q=2$.
Solution:
Number of Relations from A to B:
A relation from set A to set B is defined as any subset of the Cartesian product $A \times B$.
The number of elements in the Cartesian product $A \times B$ is given by $\text{n}(A \times B) = \text{n}(A) \times \text{n}(B) = p \times q = pq$.
The total number of subsets of a set with $N$ elements is $2^N$. Since a relation is any subset of $A \times B$, the total number of relations from A to B is the number of subsets of $A \times B$.
Total number of relations from A to B = $2^{\text{n}(A \times B)} = 2^{pq}$.
Number of Functions from A to B:
A function from set A to set B is a special type of relation where every element in the domain A is associated with exactly one element in the codomain B.
Let the elements of set A be $a_1, a_2, ..., a_p$. For each element $a_i \in A$, there must be exactly one element in B that is its image under the function.
For the element $a_1 \in A$, there are $q$ possible choices in set B to which it can be mapped.
For the element $a_2 \in A$, there are also $q$ possible choices in set B to which it can be mapped.
... and so on.
For the element $a_p \in A$, there are $q$ possible choices in set B to which it can be mapped.
Since the choice of mapping for each element in A is independent, the total number of ways to define such a mapping (which is the number of functions) is the product of the number of choices for each element in A.
Total number of functions from A to B = (Number of choices for $a_1$) $\times$ (Number of choices for $a_2$) $\times$ ... $\times$ (Number of choices for $a_p$)
Total number of functions from A to B = $q \times q \times ... \times q$ ($p$ times)
Total number of functions from A to B = $q^p = (\text{n}(B))^{\text{n}(A)}$.
Calculation for $p=3$ and $q=2$:
Given $p = 3$ and $q = 2$.
Number of relations from A to B = $2^{pq} = 2^{3 \times 2} = 2^6$.
$2^6 = 2 \times 2 \times 2 \times 2 \times 2 \times 2 = 64$.
Number of relations from A to B = 64.
Number of functions from A to B = $q^p = 2^3$.
$2^3 = 2 \times 2 \times 2 = 8$.
Number of functions from A to B = 8.
Question 12. Find the domain and range of the function $f(x) = \frac{1}{\sqrt{|x|-x}}$.
Answer:
Given:
A real function $f(x) = \frac{1}{\sqrt{|x|-x}}$.
To Find:
The domain of $f(x)$.
The range of $f(x)$.
Solution:
For the function $f(x) = \frac{1}{\sqrt{|x|-x}}$ to be defined as a real number, two conditions must be satisfied:
1. The expression under the square root must be non-negative: $|x| - x \ge 0$.
2. The denominator must not be zero: $\sqrt{|x|-x} \neq 0$, which means $|x|-x \neq 0$.
Combining these conditions, we require the expression under the square root to be strictly positive:
$|x| - x > 0$
We analyze this inequality based on the definition of the modulus function:
$|x| = \begin{cases} x & , \text{ if } x \ge 0 \\ -x & , \text{ if } x < 0 \end{cases}$
Case 1: $x \ge 0$
If $x \ge 0$, then $|x| = x$. The inequality becomes $x - x > 0$, which simplifies to $0 > 0$. This is a false statement. Thus, the inequality $|x| - x > 0$ is never true for $x \ge 0$.
Case 2: $x < 0$
If $x < 0$, then $|x| = -x$. The inequality becomes $-x - x > 0$, which simplifies to $-2x > 0$. Dividing by -2 and reversing the inequality, we get $x < 0$. This is consistent with the condition for this case ($x < 0$). Thus, the inequality $|x| - x > 0$ is true for all $x < 0$.
Combining both cases, the inequality $|x| - x > 0$ holds if and only if $x < 0$.
The domain of $f(x)$ is the set of all real numbers $x$ such that $x < 0$.
Domain($f$) = $\{x \in \mathbb{R} \mid x < 0\}$
In interval notation, the domain is $(-\infty, 0)$.
Range of the function:
We need to find the set of all possible output values $y = f(x)$ for $x$ in the domain $(-\infty, 0)$.
For $x < 0$, we have $|x| = -x$. Substituting this into the function definition:
$f(x) = \frac{1}{\sqrt{(-x) - x}} = \frac{1}{\sqrt{-2x}}$ for $x < 0$.
Let $y = f(x)$. We analyze the possible values of $y$ as $x$ varies over the domain $(-\infty, 0)$.
For $x \in (-\infty, 0)$, the expression $-2x$ is always positive. As $x \to -\infty$, $-2x \to \infty$. As $x \to 0$ from the left ($x \to 0^-$), $-2x \to 0$ from the right ($-2x \to 0^+$).
Therefore, $\sqrt{-2x}$ takes on all positive real values. As $x \to -\infty$, $\sqrt{-2x} \to \infty$. As $x \to 0^-$, $\sqrt{-2x} \to 0^+$.
The reciprocal $\frac{1}{\sqrt{-2x}}$ will be positive for all $x < 0$.
As $\sqrt{-2x} \to \infty$ (when $x \to -\infty$), $\frac{1}{\sqrt{-2x}} \to 0$ from the positive side ($0^+$).
As $\sqrt{-2x} \to 0^+$ (when $x \to 0^-$), $\frac{1}{\sqrt{-2x}} \to \infty$.
The function $f(x)$ can take any strictly positive real value.
The range of $f(x)$ is the set of all strictly positive real numbers.
Range($f$) = $\{y \in \mathbb{R} \mid y > 0\}$
In interval notation, the range is $(0, \infty)$.